ims 500

January 26, 2017 | Author: Prashant Srivastava | Category: N/A
Share Embed Donate


Short Description

Download ims 500...

Description

Dear Student, The wait is finally coming to an end!! As you already know, the final frontier: is scheduled to be conducted starting 27 th October 2010. We are sure that all the and efforts that you have put in the last few months will pay rich dividends on Being anxious about what the test may have in store for you is natural at this do not let this anxiety affect how you will finally perform in the CAT.

CAT 2010 hard work the D-day. stage. But

“CAT 500 series” is a special booklet from IMS to support you in your final lap before the CAT. This booklet comprises 500 “must-solve questions” that have been statistically chosen from select SimCATs* of 2009 and 2010. If you can solve these questions correctly and in the target time (average 2.5 min), then you can be rest assured that you are on the right path to acing CAT 2010. * SimCATs have historically and statistically proven to be closest to the actual CAT. Refer Chart on the left to view the statistical representations for CAT and SimCAT data. To ensure an “effective preparation”, the 500 Questions are classified into two levels: 1. 2.

Score Enhancers: Questions that have been solved correctly by a majority of the Top 10 percentilers who attempted them (These are questions that are must attempts to break into the Top 10 percentile) Score Maximisers: Questions that have been attempted by a majority of the Top 10 percentilers but solved incorrectly (These are questions that will differentiate the best from the rest)

The questions have been classified into 10 sections on an area-wise basis: Arithmetic, Algebra, Geometry, Modern Math, Verbal Ability, Verbal reasoning, Reading Comprehension, Data Interpretation, Logical Reasoning and Data Sufficiency. How to use this booklet: 1. Plan your schedule for solving the questions in this booklet in a systematic manner. And more importantly, stick to it. a. To attempt every question and analyse how to solve it in the most effective way, you should be spending about 8 minutes on an average on each of the questions – this translates to a total time of about 4000 min, i.e. approx 65 hours b. We recommend that you spend 3-4 hours daily for this purpose – this means you should complete the booklet in around 20 days c. If you do not have enough time on hand, first focus on completing questions from your areas of weakness and then, time permitting, move to those from your areas of strength. 2. If you need any help in solving the questions in this booklet, refer to the explanatory answers provided at the end of the booklet (refer Pg 132) 3. For further assistance/guidance on how to solve these questions in the most efficient manner, feel free to contact our centres and register for “CAT 500 Helpline” sessions. Wish you all the best for Success in the CAT and other tests that you will be taking this season. Stay Focussed. Vinayak KUDVA National Product Manager, Test prep, IMS

1

The Last Mile All endeavor calls for the ability to tramp the last mile, shape the last plan, endure the last hour’s toil. Step 1: Believe that you can tramp the last mile!! Life’s races aren’t won by the biggest, the fastest or the strongest. It’s inevitable that one day it will be won by the person WHO BELIEVES HE CAN!! Step 2: Have a fresh outlook: Rejuvenate and Focus on your goal – acing CAT 2010 -

Imagine you have just started your prep. Imagine the last comprehensive test you took was your first test. Remember that at this stage, you have nothing to lose and everything to gain.

Step 3: For every test you take: -

Set the desired target score to get the coveted IIM call. Take the test to find your actual score. Analyse how you can bridge the gap between your actual score and desired score

Step 4: Bridge the Gap -

List all areas that require improvement. Rate the impact each of the areas can have on the score and prioritise accordingly areas with a higher weightage in the test must be tackled first. Spend about 3-4 hours everyday and work on two to three critical areas. Ensure that you can solve the “Score enhancer” and “Score maximiser” questions that have been identified.

Finally, a few tips for the D-Day: 1. 2. 3. 4. 5. 6.

Stay calm and focused. Do not stress yourself by worrying. On the eve the CAT, sleep early and ensure that you are fresh the next day. Ensure that you have the Admit Card and any other document(s) as specified in your admit card. Before the test, read the instruction page carefully - see if there are any major changes in the test structure and if the changes warrant a change in your test strategy. While taking the test, read the questions and the directions for every question very carefully. Also be careful while clicking the right option. Mentally break the test into shorter tests-carry the success in one mini-test to the next, but leave any failure behind.

DO NOT GIVE UP AT ANY STAGE DURING THE TEST. For all you know, you may perform relatively better than the others.

2

1. PROBLEM SOLVING a. ARITHMETIC Score Enhancer ............................................................................................................ 5 Score Maximiser .......................................................................................................... 8 b. ALGEBRA Score Enhancer ............................................................................................................ 11 Score Maximiser .......................................................................................................... 12 c. GEOMETRY Score Enhancer ............................................................................................................ 13 Score Maximiser .......................................................................................................... 15 d. MODERN MATHEMATICS Score Enhancer ............................................................................................................ 17 Score Maximiser .......................................................................................................... 19 2. DATA INTERPRETATION * .................................................................................... 21 3. DATA SUFFICIENCY Score Enhancer ................................................................................................................ 39 Score Maximiser .............................................................................................................. 40 4. LOGICAL REASONING * ......................................................................................... 45 5. VERBAL ABILITY Score Enhancer ................................................................................................................ 53 Score Maximiser .............................................................................................................. 63 6. READING COMPREHENSION * ............................................................................ 80 Explanatory Answers ............................................................................................................ 132 · *Questions will be classified as “Score Enhancer” and “Score Maximiser” as per the set.

3

PROBLEM SOLVING Arithmetic ªScore Enhancerª DIRECTIONS for questions 1 to 19: Choose the correct alternative. 1.

6.

What is the remainder when ((55) 15!) 188 is divided by 17? 1) 1 3) 3 5) 15

2) 16 4) 5

An ant travels along the edges of a cube shown below. It travels along the longest path from A to F at 2 cm/s and travels back to A at 1 cm/s taking the shortest route. It does not cross any vertex more than once and completes the journey in 120 seconds. What is the length of each side of the cube? H

2.

Let n = 999 ... 99 be an integer consisting of a string of 2009 nines. Find the sum of digits of n2. 1) 18072 3) 18090 5) 18073

F

E

2) 18081 4) 18080

A group of 10 workers can plough a field in 20 days. This group starts the work and after every 2 days, 2 additional workers join the group. The capacity of each worker is the same. In how many days will the field be ploughed? 1) 11 3) 14 5) 13

4.

7.

8.

2) 11 4) 6.6

bers: 1)

−1 55

3)

−1 77

,

−1 66

,

−1 77

,

−1 88

2) 4)

−1 88

In a computer program, the current values of a and b are 5 and 8 respectively. It then executes the following code 100 times. Step 1: New value of a = Sum of the present values of a and b; Step 2: New value of a= The difference between the present values of a and b;

? −1 66

2) 3 : 11 4) 1 : 5

{

Which is the largest of the following num−1 55

2) 15 cm. 4) 20 cm.

A mixture of liquids A and B contains 70% of B by weight. Liquid C is added till the final solution contains 12% by weight of A. What is the ratio (by weight) of B to C in the final mixture? 1) 7 : 15 3) 7 : 25

11 11 11 11 + + + + Find x, where x = 8 15 24 3 11 11 11 11 11 + + + + . 35 48 63 80 99

5.

B

1) 12 cm. 3) 24 cm.

2) 12 4) 15

1) 7.2 3) 14.4

C

D A

3.

G

Step 3: New value of b= The difference between the present values of a and b; } What are the final values of a and b? 1) a = 5, b = 8 3) a = 5, b = 2

5

2) a = 8, b = 5 4) a = 5, b = 3

PROBLEM SOLVING 9.

Considering all the natural numbers which lie between 1000 and 7770 (not including either), in which place does the digit 7 appear the most? 1) Units 3) Hundreds

10.

2) Tens 4) Thousands

16.

2) 12 4) 18

If k = [ ] ×[ ] × [ ] × [ ] . . . × [

]× [

],

where [ ] is the greatest integer d” a, then

12.

13.

17.

2) 15 4) 24 18.

Distance between two points P and Q is 1200 meters. Car A starts from P and travels on a straight line at a speed of 15m/s to reach Q. Then, it reverses its direction immediately to travel back to P. If car B starts from P towards Q, four seconds later than car A at a speed of 10m/s, what distance from Q will these two cars meet? 2) 991 m 4) 209 m

Find the value of the following expression:

2 × 4n +1 + 42n −1

2) 1 4) 65

2n

4×2

− 8× 4

1) 22n+1 + 2 3) –1

A mixture of liquids P, Q and R, contains the three in the ratio 2 : 4 : 7 respectively. P and Q are added till the ratio becomes 7 : 4 : 2. Find the ratio of the amounts of P and Q added. 1) 49 : 16 3) 5 : 2

The cruise liner “Queen Alice” is 380 m long and travels at a speed of 32 kmph in still water. The frigate “Lord Harry” is 180 m long and travels at 40 kmph in still water. The two ships pass each other in the Atlantic ocean, traveling in opposite directions, in a region where there is current of 8m/s. How long will it take them to pass each other?

1) 936 m 3) 264 m

Find the remainder when (3! + 6! + 12! + 24! + 48! + 96!) is divided by 66. 1) 0 3) 33

14.

2) 32 4) 18

On buying a camera, the shopkeeper gives three rolls of film free. On buying a camera and six rolls of film, the shopkeeper gives additional four rolls of film free. If the equivalent discount is the same in both cases, then how many rolls will be equal in value to a camera? 1) 12 3) 18

2) 8.82 % 4) 11.61%

1) 20 sec 2) 28 sec 3) 42 sec 4) Cannot be determined

the number of zeroes at the end of k is 1) 21 3) 33

A mutual fund gives 21% per annum compound interest. Another investment gives the same earning in 5 years under simple interest as the mutual fund gives in two years. What is the rate of interest of simple interest? 1) 8.4% 3) 9.28 %

Find the sum of the last two digits of (2 3 + 3 3 + 4 3) 1) 9 3) 1

11.

15.

19.

n −1

×

1 2(n −2)+1

4+2

2) 1 4) 2n

What is the remainder when by 79? 1) 0 3) 47

2) 9 : 4 4) 7 : 4

6

2) 1 4) 67

9 24 is divided

PROBLEM SOLVING DIRECTIONS for questions 20 and 21: Answer the questions on the basis of the data given below.

23.

If f(n) < 2n, then the number ‘n’ is called a deficient number.

The factorial of a natural number ‘n’ (or n!) is defined as the product of all natural numbers less than or equal to ‘n’. Given: m = 1! + 2! + 3! + 4! + …. + 99! + 100! 20.

21.

f(n) is defined as the sum of all the divisors of a natural number ‘n’.

If f(n) > 2n, then the number ‘n’ is called an abundant number.

Find the last two digits of ‘m’.

Which of the following pairs of numbers comprises one abundant and one deficient number?

1) 3 3) 13

1) 7, 28 3) 42, 84

2) 9 4) 19

Find the remainder, when ‘m’ is divided by 168. 1) 33 3) 153

2) 6, 64 4) 32, 42

DIRECTIONS for question 24: Answer the questions on the basis of the data given below.

2) 129 4) 67

DIRECTIONS for questions 22 and 23: Answer the questions independently of each other. 22.

Alok started one hour after Bimol from city P towards city Q and crossed Bimol at a distance of 10 km from P. After reaching city Q, Alok immediately started moving back to city P along the same route. On the way back, he again met Bimol, who still needed 1 hour and 12 minutes to reach city Q. Find the distance between city P and city Q if Alok and Bimol travelled at a constant speed throughout the journey and Alok’s speed was 5 km/hr.

ACEG is a square park divided into two parts by its diagonal. The only paths available for cycling in this park are the boundary of the park, the diagonal and two identical circles placed in the two halves. Each of the circular paths has a radius of 1 km and they touch the side of the square at B, D, F and H and the diagonal of the square at point K. Amit cycles

1) 30 km 3) 34 km

24.

along these paths at a constant speed of

2) 28 km 4) 20 km

Find the ratio of time taken by Amit to cycle along the path B-K-F to the time taken to cycle along the path B-H-K-D-F. 1) 2 : 3 3) 3 : 5

7

2 km/hr.

2) 1 : 2 4) 4 : 5

PROBLEM SOLVING Arithmetic ªScore Maximiserª DIRECTIONS for questions 25 and 26: Choose the correct alternative.

DIRECTIONS for questions 28 to35: Choose the correct alternative.

25.

28.

Shakuntala challenges Dushyant - “Identify the ages of my three younger brothers. No two are of the same age and the sum of their ages is 35.” “This is certainly not enough information. Give me some more clues”, said Dushyant. “Okay”, she replied, “The age of each one is a prime number and if I tell you the age of the middle one, then you would easily get the ages of the others.” What is the age of the eldest brother? 1) 17 years 3) 23 years 5) Cannot be determined

26.

1) 20 3) 16 29.

2) 19 years 4) 29 years

2) 42 4) 14

30.

M children (1 to M) are standing in a circle facing each other wearing different caps. In the first round each passes his cap to the child on his left. In the next round each one passes the cap he now has to the second child on his left. In the next round each one passes the cap he has to the third child on his left and so on. They stop when the first child gets his original cap back for the first time.

in

the

set

2) 17 4) 21

It is known that: “Integer n is not prime if k is an odd number divisible by 3”. Which of the following can be logically concluded from this?

The time for a pendulum’s swing is directly proportional to the square root of its length. A pendulum 21 cm long is found to swing 30 times per minute. How many swings per minute will be made by a pendulum 28/3 cm long? 1) 45

2) 20

40 3

4) 80

3) 31.

If M = 44 then after how many rounds will they stop? 1) 12 3) 33

numbers

1) If n is prime then k is an even number not divisible by 3. 2) If k is not an odd number divisible by 3 then n is prime. 3) If n is prime and k is divisible by 3 then k must be divisible by 6. 4) If k is divisible by 3 and n is not prime then k is not divisible by 6.

DIRECTIONS for question 27: Refer to the data below and answer the questions that follow.

27.

many

6! 3!   99! 96! 93! , , ,... , ,  are divisible by 24?  3! 0!   96! 93! 90!

A function f is defined as f(n) = 6 n + 8n for all integers n. Find the remainder when f(83) is divided by 49. 1) 7 3) 0 5) 35

How

How many pairs of integers (x, y) exist such that x2 + 4y2 < 100? 1) 95 3) 159

2) 22 4) 32 32.

How many numbers between 300 and 400 are such that the number equals the sum of the cubes of its digits? 1) 0 3) 2

8

2) 90 4) 180

2) 1 4) 3

PROBLEM SOLVING 34.

2 for n > 1, then find the value n −1 of u 2 + u 3 + … + u 100.

If un

=

1) 1.15 3) 1.36 33.

35.

ACEG is a square park divided into two parts by its diagonal. The only paths available for cycling in this park are the boundary of the park, the diagonal and two identical circles placed in the two halves. Each of the circular paths has a radius of 1 km and they touch the side of the square at B, D, F and H and the diagonal of the square at point K. Amit cycles along these paths at a constant speed of

2) 1.24 4) 1.48

In the last summer vacation, Akshay was given an assignment of writing down numbers from 100 to 1000. Despite all his brilliance and intelligence, Akshay always gets confused between the digits ‘6’ and ‘9’. As a result, he ends up interchanging them. How many numbers did he write correctly in his assignment? 1) 343 3) 448

2 km/hr.

How much extra time will Amit take to cycle along the path B-C-K-G-F than to cycle along the path B-C-D-E-F? 1) 2 hrs 3) 0.5 hrs

2

2) 3.5 hrs 4) 0.2 hrs

9

2) 353 4) 449

PROBLEM SOLVING Algebra ªScore Enhancerª DIRECTIONS for questions 6 and 7: Refer to the data below and answer the questions that follow. DIRECTIONS for question 36 and 37: Refer to the data below and answer the questions that follow.

41.

The value of the quadratic function f(x) = ax 2 + bx + c is minimum for x = –2.5. Also, it is known that f(–2) = –2f(3). 36.

Find the product of the roots of f(x).

1) 2 5 3) 6

Determine the minimum value of f(x).

42.

DIRECTIONS for questions 38 to 41: Choose the correct alternative. Three positive real numbers x, y, z are such that x + y + z = 1. Which of the following inequalities best describes the relation between xy, yz and zx?

40.

2 3

A two-digit number x, when increased by x% becomes a perfect square 'y'. Find y. 1) 324 3) 256 5) 144

2) 289 4) 196

43.

2) 4 4) more than 5

Two people took the test, attempting 11 questions each. Which of the following cannot be the sum of their scores? 2) 23 4) 88

Train fares on the Northern Railway are calculated as follows: for each journey, a fixed charge of Rs. k is levied for the first 100 km, and Rs. 1.5 per km thereafter. Mr. Pant travels from A to B (a distance of over 300 km) and pays Rs 1100. Mr. Khot travels from A to C (which is exactly midway between A and B) and then takes a second train from C to B, spending Rs. 1150 in total. 44.

What is the coefficient of a2c2e2 in the expansion of (a)(a + b)(a + b + c)… (a + b + … + f)? 1) 1 3) 8 5) 0

1) 3 3) 5

Answer questions 44 and 45 based on the following information:

1 2 3) xy + yz + zx < 4) xy + yz + zx < 3 3

39.

A group of friends took the test. If all of them scored exactly 15 marks, but each of them attempted a different number of questions, what is the maximum number of people who could be in the group?

1) 22 3) 33

1 1 1) xy + yz + zx > 2) xy + yz + zx < 3 3

5) xy + yz + zx >

4) Cannot be determined

A test has 20 questions, with 4 marks for a correct answer, –1 mark for a wrong answer, and no marks for an unattempted question.

1) –60.75 2) –91 3) –2.5 4) –121.5 5) Cannot be determined

38.

2) 4

DIRECTIONS for questions 199 and 200: Refer to the data below and answer the questions that follow.

1) 14 2) –14 3) –9 4) 9 5) Cannot be determined 37.

The quadratic polynomial f(x) = x 2 + bx + c takes a minimum value of –5 and f(0) = 4. What is the difference between the roots of f(x)?

2) 2 4) 90

11

What is the value of k? 1) Rs. 100 3) Rs. 200

45.

2) Rs. 150 4) Rs. 250

What is the distance from A to C? 1) 350 km 3) 600 km

2) 450 km 4) 700 km

PROBLEM SOLVING DIRECTIONS for questions 46 to 50: Choose the correct alternative. 46.

A two-digit number is reversed and twice the resultant number is subtracted from the original number. If the result obtained is 13, what can be said about the original number? 1) 2) 3) 4)

47.

48.

1) 14 3) 15

It is a perfect square It is a prime It is an even number None of the above

When x =

A number is said to be a ‘zeroth number’ if the sum of the squares of its digits ends in a zero. How many two-digit ‘zeroth numbers’ are there?

49.

2) 13 4) 17

p 1 q +1 1 r +2 3 s + 4 3 = , = , = If q = 3 , and r +1 2 s +2 2 t + 4 4 p+6 2 p+q+r = , the value of is: q+6 3 s+t

3 , find the product xy such that 2

y satisfies the following equation:

1)

7 5

2)

8 7

18y 2 +24y +14 3 4× x

3)

17 20

4)

21 20

9y 2 +7+12y 3 − 12 × x

1) 1 3)

+ 9 = 0

2) –1

2 3

4)

−3 2

Algebra ªScore Maximiserª 50.

5An+1 + 1 = 5An + n. Given A 5 = 55, find A55. 1) 341.2 3) 350 5) 342.8

DIRECTIONS for questions 53 and 54: Choose the correct alternative.

2) 340 4) 352

53.

DIRECTIONS for questions 51 and 52: Refer to the data below and answer the questions that follow. The quadratic polynomial f(x) = x2 + bx + c takes a minimum value of –5 and f(0) = 4. 51.

What is the value of

1)

2 3

3) 3 52.

1) m < 62 3) 63 < m < 64 54.

1) 11 3) 20

2) 62 < m < 63 4) m > 64

a, b, c are 3 natural numbers such that

a 2 + b 2 = c 2 . If b and c are two consecutive

3 2

numbers and c < 51, find the number of triplets (a, b, c) that are possible.

4) Cannotbedetermined

1) 3 3) 5

How many integers x with |x|< 100 can be expressed as x =

, then what can be said

about the value of m?

b ? c

2)

If m =

4 − y2 for some integer y? 4

2) 10 4) 21

12

2) 4 4) 7

PROBLEM SOLVING Geometry ªScore Enhancerª DIRECTIONS for questions 55 to 57: Choose the correct alternative. 55.

Answer Questions 58 and 59: Answer the following questions on the basis of given information:

E B

C

G

F

A

D

A

Three ants are standing at position A, B and C on a large circular clock, at the marks indicating 3 o’clock, 5 o’clock and 7 o’clock respectively, as shown in the adjoining figure.

What is the area of square ABCD, if the area of kite AECF = area of the kite DEBG = 100 sq. units? 1) 200 sq. units 2) 150 sq. units 3) 400 sq. units

58.

4) 100 2 sq. units 5) Cannot be determined 56.

A rectangular tank, 8 m deep, is filled with water upto a certain level. If 50% of the water is removed from the tank, the water level falls by 30% of the total depth. Up to what level from the base was the tank filled initially? 1) 2.4 m 3) 4.8 m C

B

D

In the circle alongside, chords AB and CD of equal lengths intersect at right angles at point M. Two smaller circles of radius 2 cm and 4 cm are drawn touching the larger circle such that AB and CD are tangents to both the circles. Find the radius of the outer circle. 2) 3 2 cm

3) 3(2 2 – 1) cm

4) 3( 2 + 1) cm

13

2) 3 π : 2 3 4) 2 π : 3

If ∠BAC = Y, then which of the following is true? 1) 2) 3) 4)

57.

1) 6 cm

: 3 2

3) p : 2

2) 3 m 4) 7.2 m

M

If the ant at A walks in a straight line (i.e. crossing the face of the clock) to the 7 o’clock mark while the ant at C walks along the rim of the clock to the 3 o’clock mark in anticlockwise direction, and both take the same time to traverse their respective paths, what is the ratio of their speeds? 1) 2 π

59.

A

B

C

0 < Y < 20 20 < Y < 40 40 < Y < 60 Cannot be determined

PROBLEM SOLVING DIRECTIONS for questions 60 to 70: Choose the correct alternative

62.

A E

O B 60.

F

C

In the figure, the radius of the circle with centre ‘O’ is 7 cm. Two circles are drawn inside a third circle, such that they touch each other externally and the large circle internally, such that the points of contact with the outer circle are diametrically opposite. The area of the larger circle is thus divided into 4 parts with areas p, q, r and s as shown in the figure. Then which of the following is true?

If AC = BC and OE = 3 cm, then find the length of AF. 1) 9 cm 3) 10 cm

2) 8 cm 4) 11 cm

63.

1) p + q = r + s 2) pr = qs 3) p + r = q + s 4) 61.

A triangle ABC is isosceles with the two equal sides measuring 5 cm and the third side being 8 cm in length. Find its circumradius.

In the given figure, m ∠ ACD = 20°, m ∠ ABD = 40° and BC = BD = AC.

1) 4 cm 3) 4.25 cm

Find m ∠ BAC. 1) 40° 3) 60°

2) 4.17 cm 4) 4.33 cm

14

2) 70° 4) 55°

PROBLEM SOLVING Geometry ªScore Maximiserª 64.

How many distinct triangles have all three sides in integer units, none of which is a part of a pythagorean triplet? 1) 0 3) 3 5) Infinitely many

65.

2) 1 4) 4

∠ ACD

2) x = 1 4) x > 1.1

A kite-shaped quadrilateral is cut from a circular sheet of paper such that the vertices of the kite lie on the circumference of the circle. If the lengths of the sides of the kite are in the ratio 3 : 3 : 4 : 4, then what percentage of the area of the circular sheet of paper remains after the kite has been cut out? 1) 53% 3) 39%

2) 47% 4) 42%

70.

In ∆ ABC, AB is extended to point D such that AB = BD. Which of the following state-

I.

to be a

∆ ABC has two sides equal

II. ∠ CBD = 120 o 1) Only I 3) Both I and II 67.

69.

128 cu. m 96 cu. m 54 cu. m 72 cu. m Cannot be determined

ments is/are necessary for right angle?

If x is the side of the largest equilateral triangle that can be drawn inside a square of side 1, what can be said about x? 1) 0.7 < x < 1 3) 1 < x < 1.1

From a 4m x 5m x 6m cuboid, two cubes are to be cut out such that both of them have faces parallel to those of the cuboid. What is the maximum possible total volume of the cubes? 1) 2) 3) 4) 5)

66.

68.

In the given figure, find the side of square ABCD if the radius of the circle is 5 m and the length of EF is 8 m. 1) 3.5 m 2) 5 m 3) 4.2 m 4) 6.7 m

2) Only II 4) Neither I nor II

A lecturer uses a microphone while teaching. The 4.8 m × 7.2 m classroom has speakers mounted at the four corners. Whenever the lecturer ventures within 3 m distance of the corners, there is a disturbance on the speakers. What is the maximum distance that he can walk in a single direction, parallel to the longer sides of the room, without any disturbance? 1) 6 m 3) 3.6 m

2) 4.2 m 4) 1.2 m

15

PROBLEM SOLVING Modern Mathematics ªScore Enhancerª DIRECTIONS for questions 71 to 80: Choose the correct alternative 71.

75.

In a box, there are some red balls, some blue balls and two yellow balls. The probability

At how many points do the graphs of the functions y = 2 log x and y = log 2x intersect? 1) Zero 3) Exactly 2

1 . Also, 2 the probability that two balls picked simul-

2) Exactly 1 4) More than 2

that a ball picked at random is red is

76.

1 . How many balls 11 are present in the box?

taneously are blue is

1) 22 3) 12 5) 20

A function f is defined for all whole numbers as follows: f(0) = 1, f(1) = 2 and f(n+2) = f(n) + 2n + 3. Then which of the following is not always true about f(n)? 1) 2f(n) = (n2 + n + 2) 2) f(n +4) = f(n +2) + f(n + 3) 3) 3f(4) = 3f(3) × 3f(2)

2) 18 4) 16

4) f(2n) = f(n) + 72.

In a series of positive numbers, (n + 1) th term is given as t (n+1) = tn + t(n–1) (n > 1) where tn is the nth term of the series. If t 8 = 124 and t 11 = 430, find t 12. 1) 694 3) 277 5) 643

77.

78. 73.

If log b a +

1 log a b +

= 1, then what

1 log b a +

A function f is defined on all whole numbers a, b as f(a+b) = f(a) + f(b) + 2 ab. Also it is known that f(3) = 15 and f(5) = 35. 1) 2 3) 4

1) 8

2) 16

3) 3

4)

79.

1 2

If k =

2) 3 4) 5 , what can be said about

the value of k? 1) 15 < k < 21 3) 27 < k < 33

5) 64 In an arithmetic progression, the sum of the

80.

1 first n terms is (3n 2 + 7n) for any natural 2

2) 21 < K < 27 4) 33 < k < 39

The function F is defined as F(k) = 2k3 – 3k2 – 5k + 7 and the function G is defined as G(k) = 2k3 + k2 +7k + 15. Find the product of all

number n. What is the difference between the 10 th and 15 th terms of the sequence? 1) 15 3) 32

2) 0.66 4) 0.70

What is the value of f(1)?

1 log a b

is the value of a if b = 32?

74.

If log 4= 0.6, find log 5. 1) 0.64 3) 0.69

2) 707 4) 153

n (3n + 1) 2

values of k for which F(k) and G(k) are equal.

2) 25 4) Cannotbedetermined

1) 2 3) –1

17

2) 1 4) –2

PROBLEM SOLVING Answer questions 81 and 82 based on the following information: 81.

How much time would the event occupy, if organized efficiently (i.e. each guy moves immediately on to the next girl when the time is up)?

The functions P and Q are defined as follows (where x, y > 0 and m, n are natural numbers):

1) 2) 3) 4)

P (x, m) = 1 + x + x 2 + x 3 + . . . + x m Q (y, n) = 1 – y + y2 – y3 + . . . + (– 1)n yn If J = P(3, 2)ÎQ(2, 3), what can be said about the value of J? 1) 2) 3) 4) 82.

85.

J is a perfect square 24 < J < 33 J < 24 J is divisible by 6

1) P(x, m) × Q(y, n) will always be positive if x > 1 2) P(x, m) × Q(y, n) will always be negative if y > 1 3) P(x, m) × Q(y, n) will always be negative if m is odd 4) P(x, m) × Q(y, n) will always be positive if n is even

86.

87.

2) 354 4) 420

A florist makes 25 bouquets, such that the first bouquet has 5 roses and each subsequent bouquet has ‘n’ more roses than the previous one. He uses 3125 roses in all for these 25 bouquets. How many more bouquets can he make, if he uses 6125 roses in all? 1) 35 3) 15

DIRECTIONS for questions 83 to 88: Choose the correct alternative. 83. There are ‘n’ children standing in a circle. A teacher has three types of chalk (pink, green and blue) and marks a triangle joining every set of three children, using only one colour of chalk for a given triangle. Once he has finished all the triangles, he notices that there are an equal number of triangles of all three colours. Which of the following could not be the number of children?

84.

Out of 15 points that lie in a plane, 3 points lie on a line and another 4 points lie on a line parallel to it. If no other 3 points are collinear, find the number of triangles that can be formed using these points. 1) 450 3) 455

Which of the following is true?

1) 18 3) 20

4m minutes 4n minutes 4mn minutes 4(m + n) minutes

2) 10 4) 20

In an arithmetic progression, the p th term is 1 1 and the qth term is p . Find the sum of q

the first ‘pq’ terms of the progression. 1) pq + 1

2)

pq + 1 2

2pq + 1 2

4)

pq + 2 2

3)

2) 19 4) 21

88.

In a Blind Dating event, the participants comprise ‘m’ guys and ‘n’ girls (m > n). Each of the m guys gets to spend exactly 4 minutes with each of the n girls (any excess unpaired guys during any round await their turn). At the end of the event, each female has to submit a list of the males that she would like to meet again (which could even be all of the males, or none of them).

Using only 3, 4, 5, 7 and 9 as digits, how many distinct natural numbers, can be made if the last digit is a perfect square and the first digit is a prime? All digits may not be used but no digit can be repeated. 1) 48 3) 96

18

2) 49 4) 98

PROBLEM SOLVING DIRECTIONS for question 89: Answer the questions on the basis of the data given below.

89.

What could have been the total number of respondents surveyed? 1) 55 3) 70

Recently, a survey was conducted in Mumbai to determine the most commonly used mode of transport among buses, trains and cabs. It was found that 32 of the respondents travel by bus while 24 of the respondents travel by train. Only four respondents use all three modes of transport. The number of respondents travelling by bus only, train only and cab only is 21, 12 and 14 respectively. All the respondents use at least one of these modes of transport.

2) 65 4) 85

Modern Mathematics ªScore Maximiserª DIRECTIONS for questions 90 and 91: Choose the correct alternative. 90. If it is true that |x2 – 9| is less than K for all real numbers x such that |x – 3| < 0.5, then what is the smallest value that K can take? 1) 0.25 3) –2.75 91.

3) F(4m) = F(2m) × 4 4) F(m + 1) – F(m) > F(n + 1) – F(n) if m > n DIRECTIONS for questions 93 to 102: Choose the correct alternative. 93.

2) 2.75 4) 3.25

1) 9 3) 22

If from the digits 1–9, five distinct digits are used to form a pair of numbers, using all five digits exactly once; how many such pairs of numbers can be made? 1) 60480 3) 15120

94.

2) 1890 4) 30240

95.

A function F(n) is defined for all natural numbers by: F(n + 1) = F(n) + 4n if n is odd = F(n) if n is even and F(1) = 0

1) F(n + 2) – F(n) is never divisible by 8 2) F(n) has an odd number of factors for n > 2

19

2) 36.25 4) 53.75

A man standing at the bottom of a staircase starts tossing a coin. Every time it shows Heads, he climbs two steps, while every time it shows Tails he climbs one step. After a while, he finds that he has climbed 8 steps. How many possible sequences of Heads and Tails could he have thrown? 1) 256 3) 36

Which of the following is not always true for F(n)?

2) 15 4) 33

If the value of x (x + 3) lies between 28 and 40, where x is a real number, then the value of x 2 cannot be: 1) 21.75 3) 49.25

DIRECTIONS for question 92: Refer to the data below and answer the questions that follow.

92.

Which of the following will not divide F(36) – F(31)?

2) 28 4) 34

PROBLEM SOLVING 96.

The functions P and Q are defined as follows (where x, y > 0 and m, n are natural numbers):

100.

P (x, m) = 1 + x + x 2 + x 3 + . . . + x m

which of the following is/are possible values of y?

Q (y, n) = 1 – y + y2 – y3 + . . . + (– 1)n yn What is the value of P(x, 8) / Q(x, 8) 1)

2)

3)

4)

1 64 3) 8

1)

101. 97.

4) Both [1) and [3)

… (n terms) 1) 1.5 log (n!) 2) n log {(n+1)!}

1) 1 3) 3

Find log 2

4)

+ log 3

+ log 4

+ log 5

102.

log {(n+1)!} 3

In a Blind Dating event, the participants comprise ‘m’ guys and ‘n’ girls (m > n). Each of the m guys gets to spend exactly 4 minutes with each of the n girls (any excess unpaired guys during any round await their turn). At the end of the event, each female has to submit a list of the males that she would like to meet again (which could even be all of the males,

3)

In how many different ways could the final lists be made by the girls? 1) mn 3) 2mn

2) nm+1 4) (m+1)n

Two functions, f and g, are such that 4g(x)2 – 2f(x)f(–x) = f(x)2 + f(–x)2. If g(4) = 20, what is the value of g(–4)? 1) 2) 3) 4)

20 –20 Either (1) or (2) Neither (1) nor (2)

20

2) 2 4) 4

Raju is a movie freak. Last Sunday, he watched five movie shows one after the other in his favourite multiplex. This multiplex has six screens, each screen showing a different movie. On each screen, a movie is repeated five times a day. If Raju watched exactly one movie twice and the other three movies only once, then in how many ways could he have watched the movies? 1) 6 ×

or none of them).

99.

2) 1

If the number of respondents travelling by cab is the least, then find the number of possibilities for the total number of respondents.

3) log {(3n/2)!}

98.

12 If (logxy)2 – log x = 45 and y > x > 0, then y

5! 2!

2) 60 ×

6! 2!

6! 2!

4) 60 ×

5! 2!



Data Interpretation Data Interpretation ªScore Enhancerª DIRECTIONS for questions 103 and 104: Refer to the data below and answer the questions that follow.

In League 2, each team was involved in exactly one 'upset'. Both the Semi-Final games as well as the Final were 'upsets'. [In each of the questions from 27 to 30 you can use data obtained from earlier question/s, if necessary].

12 countries are participating in the World Hockey Tournament being played in Australia. The tournament format comprises two league rounds followed by the semi-finals and final. Some information about the tournament is given below: League 1 In this round the 12 teams were divided across four groups (A, B, C, D) as per their WHR (World Hockey Ranking). WHR 1 8 9

Group A Netherlands Argentina S. Korea

WHR 2 7 10

Group B Spain India New Zealand

WHR 3 6 11

103.

S. Korea and Pakistan were the Runner Up teams from their respective groups in League 1. Which of the following were the winners from their respective groups? 1) 2) 3) 4) 5)

Group C WHR Group D Australia 4 Germany Pakistan 5 France England 12 China

League 2 In this round the 8 teams that advanced from League 1 were divided across 2 groups E and F as follows: Group E: Winners of Group A, C and Runners Up of Group B, D Group F: Winners of Group B, D and Runners Up of Group A, C Semi-final The two Semi-final games were played between the Winner of Group E and the Runner Up of Group F and the Winner of Group F and the Runner Up of Group E. Final The Winners of the two Semi-Final games played the Final game that decided the Winner of the tournament. *In both League 1 and League 2, each team played one game against every other team in its group. Every game in these rounds had a single winner with the winner getting 1 point and the loser getting 0 points. * In each group, in each round, the team with the higher number of points was placed higher in the group. If 2 teams had the same number of points, the team with the higher WHR was placed higher. In league 1 & 2, the 2 teams placed highest in each group advanced to the next round (with the one placed higher declared as the winner). The points from one round were not carried to the next round. *An 'upset' is a game in which a lower ranked team defeats a higher ranked team. In League 1, there was exactly one 'upset' in each of the groups.

104.

Netherlands, Australia Netherlands, England Argentina, Australia Argentina, England Cannot be determined

New Zealand and France lost both their matches in League 1. Which of the following teams proceeded to Group E in League 2? 1) 2) 3) 4) 5)

Spain, China Spain, Germany India, China India, Germany Cannot be determined

DIRECTIONS for question 105: Refer to the data below and answer the questions that follow. Electro India is an electronic chip producing company. There are 5 machines which manufacture 5 types of chips (not necessarily all types). Different types of devices use one or more of these chips. Efficiency of a machine is given by number of functional (i.e. non-defective) chips manufactured to the total number of chips manufactured. The following table gives the data for number of chips produced by each machine in a day with their efficiencies. Machine M1 M2 M3 M4 M5

Chip 1 120

Chip 2

Chip 3 250

200 100 160

Chip 4

Chip 5

160 230

300

200 170

Efficiency (in %) 90 95 90 70 80

Note: Each machine produces only one type of chips in a day. Device A requires one unit of Chip 1 and Chip 3 Device B requires one unit of Chip 2 and Chip 5 Device C requires one unit of Chip 4 and Chip 5

21

Data Interpretation Device D requires a unit of Chip 2 and two units of Chip 4 Answer the questions independent of each other. 105.

DIRECTIONS for questions 107 to 110: Refer to the data below and answer the questions that follow.

Electro India gets a contract to produce chips for 1000 units of device C. The contract states that they need to ship 100 devices daily. For how many days will the production run?

The runs scored by top 5 run scorers Mathew Hayden (MH), SK Raina (SKR), JP Duminy (JPD), Yuvraj Singh (YS), JH Kallis (JHK) after all five matches in IPL – 2009 is given in the tabular form.

1) 10 days 3) 8 days 5) 6 days

The pie-chart shows the contribution of singles, doubles, triples, fours and sixes to the total runs scored by these top 5 players in the five matches.

2) 9 days 4) 7 days

Player Name MH SKR JPD YS JHK

DIRECTIONS for question 106: Refer to the data below and answer the questions that follow.

A B

A

B



15 – –

C D

C

D

Total 40 –

Diff

Win-Loss

+5

2–1

15–10

Runs Scored

– 15 –

–3 –

40–41

1–2

Runs scored in fours

–1 Runs scored in sixes

In a badminton tournament, four players A, B, C and D played with each other, exactly once. The player, who reached the score of 15 was declared as the winner of the game. Each game had a single winner. The table above gives some data of the six games played between these players. E.g., in the game between A and B, A got 15 points, while the data about the points earned by B is unknown. In the column of Total, the first number is the number of points earned by the player (row-wise), while the second number is the number of points scored (in all) against the player. In the column of Difference, the number indicates the total points scored by the player minus the points scored against him. In the Win-Loss column, the first number indicates the number of wins, while the second number indicates the number of losses. 106.

Runs scored in singles

36° 90°

54°

108° Runs scored in triples

72° Runs scored in doubles

Some more information about them is given as follows: 1. SKR scored all his runs in doubles, triples and fours. He scored an equal number of doubles and triples. 2. JPD did not hit a single four, whereas an equal number of fours were hit by the remaining four batsmen. 3. JHK scored 160 runs in doubles and the remaining runs in singles and fours. 4. Sixes were hit in the ratio 2 : 1 : 1 by three batsmen and YS hit the maximum number of sixes.

How many points did A score in match lost by him? 1) 10 3) 12 5) Less than 10

Runs Scored 424 310 296 298 272

2) 11 4) 10 or 12

107.

How many singles did JHK score in IPL – 2009? 1) 152 3) 72 5) Cannot be determined

22

2) 192 4) 174

Data Interpretation 108.

Who scored the maximum number of doubles in IPL – 2009? 1) JHK 3) MH 5) Cannot be determined

109.

DIRECTIONS for questions 112 and 113: Refer to the data below and answer the questions that follow.

2) SKR 4) JPD

ABC college has Junior, Senior and Post Graduate (PG) sections. The total number of students studying in the college is 1000.

What was the number of runs scored by MH in singles, if he scored 10 doubles and 10 triples in each of the 5 matches in IPL – 2009? 1) 72 3) 76 5) Cannot be determined

110.

2) 74 4) 78

Junior Senior PG Percentage

If YS scored it an equal number of singles, doubles and triples, then find the number of singles that he scored in IPL – 2009. 1) 21 3) 23 5) Cannot be determined

112.

2) 27 4) 29

Team

Won

Loss

1

A

2

B

2

3

C

1

4

D

5

E

6

F

Draw

Girls

113.

Points 9

Percentage

10 40

If the number of boys in Senior and PG sections combined is not less than the number of girls in these two sections combined, then what is the maximum possible number of students in the senior section? 1) 199 3) 200 5) 201

DIRECTIONS for question 111: Refer to the data below and answer the questions that follow. Position

Boys 450

2) 210 4) 196

In the month of April, exams were conducted. The number of boys who passed the exam exceeds the number of girls who passed the exam by 60. x% of boys pass the exam and (x + 15)% of girls passed the exam. How many students in all failed the exam? 1) 220 2) 660 3) 300 4) 340 5) Cannot be determined

5 0 2

A local football league was held in Kolkata comprising 6 teams A, B, C, D, E and F. Each team played against all other teams only once. For every win the team registered, it gained two points. For a draw, both teams got 1 point each and no points were added or subtracted for a loss. At the end of the league, the team standings were displayed on the basis of the points. When the points of two teams were equal, the team with the maximum number of wins was placed at a higher position. No two teams had the same position in the league. The above incomplete table represents the positions and points of the teams when the league was over.

DIRECTIONS for questions 114 and 115: Refer to the data below and answer the questions that follow. A school conducted a cycling competition during the sports week. There were in all 12 students who participated in the competition. The competition was conducted in two rounds. The difference between the ranks of Akshay, Saif, Bobby, Abhay, Sanjay & Kunal as compared to the ranks of Salman, Aamir, Hrithik, Sunny, Shahrukh & Shahid, after round 1, are given in Graph 1. For example, the difference of ranks of Akshay and Salman is '4'. It is known that Sunny's rank is better than Shahid's. Graph 1

111.

How many points did C score in all? 1) 8 3) 6 5) Cannot be determined

12

2) 7 4) 5

6

0 Akshay

Saif Salman

23

Bobby Aamir

Hrithik

Abhay Sunny

Sanjay Shahrukh

Shahid

Kunal

Data Interpretation DIRECTIONS for questions 118 and 119: Refer to the data and answer the questions that follow.

Graph 2 shows the difference between the ranks of all 12 students in round 2 as compared to their respective ranks in round 1.

Joshi, Lele, Deshmukh and Rao are four families who buy products from same store of same brand. The following graph shows their monthly expenditure on Soaps, Detergents, Shampoos and Hair oils. The cost of one unit of each product is Rs. 5, Rs. 14, Rs. 9 and Rs. 7 respectively. It is observed that each family buys exactly three kinds of products every month and at most 2 units of any one product.

Graph 2 9

9

5

5 3 2

2

2

2

2

1

K un al

Sa nj ay

A bh ay

Sa if Bo bb y

Su nn y Sh ah ru kh Sh ah id A ks ha y

ir H rit hi k

A am

Sa lm

an

0

55

Note: In a round, no two students were given the same rank. 114.

45 40

What was Sanjay's rank in round 1? 1) 1 3) 10 5) 12

115.

50

Lele 35

2) 2 4) 11

Saif Shahid I, II, IV, III II, IV, III, I I, II, III, IV

20 Jan

II. Salman IV. Shahrukh 2) IV, II, I, III 4) IV, III, I, II

118.

117.

119.

Mar

Apr

In which month did the Joshi family definitely not buy detergent?

If the Rao family buys detergent and hair oil every month, then in the given period, they do not buy 2 units of which of the following products? 1) Soap 2) Detergent 3) Shampoo 4) Hair Oil 5) Cannot be determined

DIRECTIONS for questions 120 to 122: Refer to the data and answer the questions that follow. An estate agent is an agent who sells a property to a customer on behalf of the owner / proprietor of that property. He gets a dual commission for selling the property i.e. from the owner / proprietor and the customer. A builder constructed three buildings - A, B and C, in three different areas. Based on the location of these buildings, the rate (in rupees) per square feet (RPSF) of a flat is fixed by the builder. In each building as the floor increases, the RPSF increases by Rs. 50/- per floor.

If Aman, Ajit and Alok scored equal marks in the test, then what could be the maximum marks they could have scored? 2) 68 4) 60

What can be the maximum number of students who got a total score of 20? 1) 1 3) 4 5) 6

Feb

1) Jan 2) Feb 3) Mar 4) Apr 5) Cannot be determined

CST Company conducted an aptitude test for recruitment of trainee managers. The test had 25 questions with 4 marks awarded for every correct answer and 1 mark deducted for every wrong answer. It is known that if two or more students got equal marks then they did not attempt the same number of questions.

1) 70 3) 55 5) 63

Rao

25

DIRECTIONS for questions 116 and 117: Refer to the data below and answer the questions that follow.

116.

Deshmukh

30

Arrange the following students from the best ranked to the worst ranked, after round 1. I. III. 1) 3) 5)

Joshi

2) 3 4) 5

24

Data Interpretation DIRECTIONS for questions 123 to 125: Refer to the data and answer the questions that follow.

For every flat sold, the estate agent gets 2% (i.e. 1% from each party) of the price of the flat. The following table gives the RPSF of the ground floor and the number of floors in a building along with the area (in sq. ft.) of each flat in that building.

120.

No. of Floors 8 9 10

45 40 Percentage Increase

RPSF for Building Ground floor Area of each flat A 4000 1600 B 4350 1200 C 4700 1400

Percentage increase in the sales compared to the sales in the previous year of two companies A and B in the given period 2003 to 2007 is shown in the graph below:-

How much does a customer pay for a flat on the sixth floor of building 'C'? 1) Rs. 70 lakh 3) Rs. 70.7 lakh 5) Rs. 69.993 lakh

2) Rs. 69.3 lakh 4) Rs. 68.8 lakh

35 30 25

B

20

A

15 10 5

121.

A person buys flats on the sixth, seventh and ninth floor of building 'B'. He gets 20% discount on RPSF of any one of the flats. He decides to use the discount to minimize the total cost. Approximately how much does he pay to the estate agent for all the three flats? 1) Rs. 1.42 lakhs 3) Rs. 1.27 lakhs 5) Rs. 1.65 lakhs

122.

0 2003

1) One of the flats is located on the 2 nd floor of building 'A' while the other is on the 4 th floor of building 'C'. 2) One of the flats is located on the 2 nd floor of building 'C' while the other is on the 4 th floor of building 'B'. 3) One of the flats is located on the 2 nd floor of building 'A' while the other is on the 4 th floor of building 'B'. 4) One of the flats is located on the 2 nd floor of building 'B' while the other is on the 4 th floor of building 'A'. 5) One of the flats is located on the 2 nd floor of building 'C' while the other is on the 4 th floor of building 'A'.

2006

2007

For which year, increase in the sales of company A with respect to the previous year is highest? 1) 2003 2) 2004 3) 2005 4) 2006 5) 2007

124.

If the ratio of sales of companies A and B was 1 : 2 in 2002, then in 2004 sales of A forms approximately what percentage of sales of B? 1) 39 2) 72 3) 49 4) 59 5) 69

125.

In 2006 total sales of A and B were 500 and 900 units, respectively. In 2008, the percentage increase in sales of A as compared to the previous year is equal to that of B in 2008. The total sales of both A and B taken together is 1908 units, in 2008. Find the percentage increase in sales in 2008. 1) 22 3) 20 5) 21

25

2005 Year

123.

2) Rs. 1.78 lakhs 4) Rs. 1.58 lakhs

The estate agent gets same commission on selling two different flats. One of the flats is located on the 2 nd floor in some building while the other is located on the 4 th floor on some other building. Which of the following options is true about these flats?

2004

2) 19 4) 17

Data Interpretation DIRECTIONS for questions 126 to 128: Refer to the data and answer the questions that follow.

ªScore Enhancerª DIRECTIONS for question 129: Refer to the data below and answer the questions that follow.

A company shortlisted five candidates for various positions at its new branch. A panel gave them points out of 10 (10 being the highest and 1 being the lowest) for each of the skill sets as shown in the table (1) below. Table (2) gives the data about the weight-age for each position according to the required skill set. Lakshmi

Mohan

Academics

6

9

8

5

6

Experience

7

6

4

6

5

Team Player

9

8

6

5

8

Leadership

10

5

3

8

10

Communication

4

5

10

8

5

12 countries are participating in the World Hockey Tournament being played in Australia. The tournament format comprises two league rounds followed by the semi-finals and final. Some information about the tournament is given below: League 1 In this round the 12 teams were divided across four groups (A, B, C, D) as per their WHR (World Hockey Ranking).

Nandini Ojas Pawan

WHR 1 8 9

Academics

Experience

Team Player

Leadership

Communication

5

1

3

2

4

Executive

4

2

5

1

3

Team Leader

1

3

2

4

5

Project Leader

2

3

1

5

4

Table (2) Total points for each candidate are calculated as the sum of the products of the points and the weightage of the respective skill set. A candidate is said to be 'fit' for a position if he/she has maximum total points for that position. 126.

127.

128.

2) Mohan 4) Ojas

2) Executive 4) Project Leader

If the company decided to select only one ‘fit’ candidate for a post, then who won’t be selected for any of the posts? 1) 2) 3) 4) 5)

WHR 3 6 11

Group C WHR Group D Australia 4 Germany Pakistan 5 France England 12 China

Final The Winners of the two Semi-Final games played the Final game that decided the Winner of the tournament. *In both League 1 and League 2, each team played one game against every other team in its group. Every game in these rounds had a single winner with the winner getting 1 point and the loser getting 0 points. * In each group, in each round, the team with the higher number of points was placed higher in the group. If 2 teams had the same number of points, the team with the higher WHR was placed higher. In league 1 & 2, the 2 teams placed highest in each group advanced to the next round (with the one placed higher declared as the winner). The points from one round were not carried to the next round. *An 'upset' is a game in which a lower ranked team defeats a higher ranked team. In League 1, there was exactly one 'upset' in each of the groups. In League 2, each team was involved in exactly one 'upset'. Both the Semi-Final games as well as the Final were 'upsets'.

Nandini is fit for which of the given posts? 1) Trainee 3) Team Leader 5) None of these

Group B Spain India New Zealand

Semi-final The two Semi-final games were played between the Winner of Group E and the Runner Up of Group F and the Winner of Group F and the Runner Up of Group E.

Who is fit for the position of Executive? 1) Lakshmi 3) Nandini 5) Pawan

WHR 2 7 10

League 2 In this round the 8 teams that advanced from League 1 were divided across 2 groups E and F as follows: Group E: Winners of Group A, C and Runners Up of Group B, D Group F: Winners of Group B, D and Runners Up of Group A, C

Table (1) Trainee

Group A Netherlands Argentina S. Korea

Lakshmi Ojas Pawan Either Ojas or Pawan N one of these

26

Data Interpretation [In each of the questions from 27 to 30 you can use data obtained from earlier question/s, if necessary]. 129.

DIRECTIONS for questions 131 to 133: Refer to the data below and answer the questions that follow.

Only one team in Group E did not win a single match in League 2. Which of the following teams proceeded to the Semi-finals from Group E? 1) 2) 3) 4) 5)

Team

1

A

2

B

2

3

C

1

4

D

5

E

6

F

Australia, China Spain, Netherlands Netherlands, Australia India, Spain Cannot be determined

Electro India is an electronic chip producing company. There are 5 machines which manufacture 5 types of chips (not necessarily all types). Different types of devices use one or more of these chips. Efficiency of a machine is given by number of functional (i.e. non-defective) chips manufactured to the total number of chips manufactured. The following table gives the data for number of chips produced by each machine in a day with their efficiencies. Chip 1 120

Chip 2

Chip 3 250

200 100 160

Chip 4

Chip 5

160 230

300

200 170

Efficiency (in %) 90 95 90 70 80

131.

132.

Device A requires one unit of Chip 1 and Chip 3 Device B requires one unit of Chip 2 and Chip 5 Device C requires one unit of Chip 4 and Chip 5 Device D requires a unit of Chip 2 and two units of Chip 4 Answer the questions independent of each other.

Draw

Points 9

5 0 2

How many matches in the league ended in a draw?

133.

27

C won F won Either C won or F won [not a draw] Draw Cannot be determined

If the match played between A and D resulted in a draw, then what was a possible result of the match between B and C? 1) 2) 3) 4) 5)

2) 43 days 4) 60 days

2) 6 4) 8

What was the result of the match played between C and F? 1) 2) 3) 4) 5)

Electro India gets a contract to produce Chips for device A. The contract is to produce 10000 devices. A minimum of how many days are required to fulfill the contract? 1) 50 days 3) 48 days 5) 57 days

Loss

1) 3 3) 4 5) Cannot be determined

Note: Each machine produces only one type of chips in a day.

130.

Won

A local football league was held in Kolkata comprising 6 teams A, B, C, D, E and F. Each team played against all other teams only once. For every win the team registered, it gained two points. For a draw, both teams got 1 point each and no points were added or subtracted for a loss. At the end of the league, the team standings were displayed on the basis of the points. When the points of two teams were equal, the team with the maximum number of wins was placed at a higher position. No two teams had the same position in the league. The above incomplete table represents the positions and points of the teams when the league was over.

DIRECTIONS for question 130: Refer to the data below and answer the questions that follow.

Machine M1 M2 M3 M4 M5

Position

B won C won Both [1) and [2) Draw All the above are possible

Data Interpretation DIRECTIONS for question 134: Refer to the data below and answer the questions that follow.

136.

CST Company conducted an aptitude test for recruitment of trainee managers. The test had 25 questions with 4 marks awarded for every correct answer and 1 mark deducted for every wrong answer. It is known that if two or more students got equal marks then they did not attempt the same number of questions. 134.

1) Soap 2) Detergent 3) Shampoo 4) Hair Oil 5) Cannot be determined DIRECTIONS for questions 137 and 138: Refer to the data below and answer the questions that follow.

Ram and Raju scored equal marks in the test. Raju said to Ram "If you would have attempted one more question then your score would be unique". Which of the following can be their initial score? 1) 83 3) 68 5) 65

In the month of April every family bought a particular product. Which of the following could be that product?

In a library, on a particular day, the following statistics for 5 different books were noted: Table–1 shows the number of readers of each of the five books. Table-2 shows the number of persons to read different combinations of the books. The readers have been categorized gender wise. Every visitor reads atleast 1 book and no more than 2 books.

2) 70 4) 63

DIRECTIONS for questions 135 and 136: Refer to the data and answer the questions that follow. Joshi, Lele, Deshmukh and Rao are four families who buy products from same store of same brand. The following graph shows their monthly expenditure on Soaps, Detergents, Shampoos and Hair oils. The cost of one unit of each product is Rs. 5, Rs. 14, Rs. 9 and Rs. 7 respectively. It is observed that each family buys exactly three kinds of products every month and at most 2 units of any one product.

Male Female

Book 1 5 9

Book 2 4 6

Book 3 11 8

Book 4 6 12

Book 5 9 15

Table 1 Book 2 10

Book 1 Book 2 Book 3 Book 4

55

Book 3 3 0

Book 4 0 0 11

Book 5 1 0 1 5

Table 2 Also, it was known that the person who read books 1 and 5 was male.

50 45 40

Joshi Lele

35

137.

Deshmukh

30

How many readers visited the library? 1) 23 3) 85

Rao

2) 31 4) 54

25

ªScore Maximiserª

20 Jan

135.

Feb

Mar

Apr

138.

In March, which family did not buy Hair Oil? 1) Joshi 3) Deshmukh 5) Cannot be determined

How many females read both book 1 and book 3? 1) 2) 3) 4)

2) Lele 4) Rao

28

Three Four Five Cannot be Determined

Data Interpretation 140.

ªScore Enhancerª DIRECTIONS for question 139 to 141: Refer to the data below and answer the questions that follow. QRS Ltd. produces six different types of drinks. These drinks are sealed in bottles. The manufacturing of these drinks is carried out in 6 different factories. Chart 1 shows the contribution of the factories as a percentage of the total production. The production of each type of drink in a factory, as a percentage of the total production of that factory, is given in chart 2. [ Use data from previous questions wherever necessary]

1) 3% 3) 6% 141.

Factory 1 Factory 4 10%

Student

Chart 2: Product-wise distribution in each factory

Rachit Nanu Astha Manav Kartik

Orange 15%

Mango 10%

2) 30,000 4) 60,000

Two tests of 8 marks each were conducted in a class. All scores were positive integers. The table below gives the ranks allotted on the basis of the scores in each test. When final ranks were allotted on the basis of the total score in both the tests, Rachit stood first and there were no ties.

Factory 2 20%

Apple 20%

The total production of QRS Ltd. in a particular month was 4 lakh bottles. What was the number of guava bottles produced by factory 3 in that month?

DIRECTIONS for question 142 to 146: Refer to the data below and answer the questions that follow.

Factory 6

Factory 3

2) 5% 4) 8%

1) 15,000 3) 33,333

Chart 1: Factory-wise Distribution Factory 5 10%

If the number of bottles of the grape flavored drink produced by factory 1 is equal to the number of bottles of the guava flavored drink produced by factory 6, over a given period of time, then find the percentage contribution of factory 6 towards the total production of QRS Ltd.

Pineapple 20%

Rank in Test 1 Rank in Test 2 1 2 3 4 5

4 1 5 2 3

Grapes 5%

142.

Guava 30%

139.

1) 3 3) 5

Factory 1 produces 300 bottles of the grape flavored drink in a week. If the total production of QRS Ltd. in that week was 20,000 bottles, find the percentage contribution of factory 1 towards the total production of QRS Ltd. 1) 25% 3) 35%

How much did Astha score in Test 1?

143.

What is the difference in Kartik and Nanu’s scores in Test 2? 1) 2 3) 4

2) 30% 4) 40%

29

2) 4 4) Cannotbedetermined

2) 3 4) Cannotbedetermined

Data Interpretation 144.

By how much is Rachit’s total score more than Manav’s? 1) 3 3) 5

145.

1) Shubham 3) Ninad

2) 4 4) Cannotbedetermined

Kartik scored 3 marks in test 2 Rachit scored 10 marks in all Astha scored 4 marks in test 2 None of the above

DIRECTIONS for questions 147 to 150: Refer to the data below and answer the questions that follow. Five students applied for a position of 'research fellow' at the IIT. They were all alumni of the same institute with Grade Point Average (GPA) scores of 2, 3, 4, 5 and 6. Their work experience was 0, 1, 2, 4 and 5 years, not necessarily in the same order. Ankit's GPA score was twice that of Shubham but he was not selected. The institute did not select the student who had a GPA score half as much as Karan's but work experience twice as much as his. Rohit had a work experience of 3 years more than Ninad's but a GPA of 3 less than Ninad's. The only one who got selected had the highest total of GPA score + work experience (in years). 147.

148.

Revenue(in lakhs) 6000

3000

2750

3200

1000 0 2005

2006

2007

2008

2009

Graph 1

100% 90% 80% 70%

2) Ankit 4) Cannot be deter-

60% 50% 40% 30% 20% 10%

2) Karan 4) Ninad

0% 2005

What was Karan's total of GPA score + work experience (in yrs)? 1) 5 3) 7

2500

2000

LMVs

149.

4000

4000

Who did not have more than a year of work experience? 1) Shubham 3) Rohit

5100

5000

Who had a GPA score of 6? 1) Karan 3) Ninad mined

2) Rohit 4) Cannotbedetermined

DIRECTIONS for questions 151 and 152: Refer to the data below and answer the questions that follow. Graph 1 shows the revenue (in lakhs) generated by an automobile manufacturing company over a period of five years. The company manufactures automobiles that can be categorized into LMV’s, Motorcycles, Buses, Transport vehicles and Other vehicles. The revenue of the company is generated by the sales of these vehicles. Graph 2 shows the contribution of these vehicles in the total revenue generation for that year.

Which of the following is possible? 1) 2) 3) 4)

Who was selected?

2) 4 4) 6

What is Kartik’s final rank? 1) 3 3) 5

146.

150.

Motorcycles

2006 Buses

2007

Graph 2

2) 6 4) 8

30

2008

Transport vehicles

2009 Other vehicles

Data Interpretation In which year did buses generate the least revenue? 1) 2005 3) 2008 152.

Salesof HairClearShampoo

2) 2006 4) 2009

4 3.5 3

What was the approximate percentage contribution of the LMVs in the total revenue generated by the company over the period 2005 - 2009? 1) 42% 3) 30%

Sales(Rs. Crore)

151.

2.5

Jan Feb March

2 1.5 1

2) 33% 4) 38%

0.5 0 Gujarat

Data for Questions 153 and 154: Abhay and Bhayya conducted a survey in their department to find out how many employees were affected by the company’s new HR policies. Abhay selected 50% of the employees and found that of the employees out of which 40% were affected. 40% of the employees selected by Bhayya were also selected by Abhay. Four-sevenths of the employees not selected by either of them were affected but three-eights of the affected employees were not selected by either of them.

155.

153.

156.

What is the minimum possible number of employees in the department? 2) 42 4) 84

2) 32.33 4) 106.7

Find the rate of growth in the volume (in ml) of HairClear shampoo sold in Madhya Pradesh from January to February and February to March. 1) 20%, 15% 3) 32%, 20%

What percentage of the employees selected by both Abhay and Bhayya were affected? 1) 50% 3) 33.33%

MadhyaPradesh

Find the volume of HairClear Shampoo sold in the month of March (approximately, in crore ml). 1) 3.23 3) 35.57

ªScore Maximiserª 154.

Goa

The promotional campaign was launched only in Goa for the month of January, only in Madhya Pradesh for the month of February and in all the four states for the month of March.

53 1 3 % of them were affected. Bhayya selected 25%

1) 30 3) 60

Maharashtra

2) 25%, 16.67% 4) 32%, 16.67%

Answer questions 157 to 160 based on the information given:

2) 66.66% 4) 16.66%

An airline service rents out planes. They have three models of planes, the A340, the B727 and the C225, with several planes of each model. For each model of plane, there is a fixed cost per trip; in addition, there is a fuel cost per km and a variable cost per passenger. These costs (in US Dollars) are summarised in the table below:

ªScore Enhancerª DIRECTIONS for questions 155 and 156: Refer to the data below and answer the questions that follow. HairClear shampoo is priced at Rs. 3 for a 10ml sachet. During promotions, 10% extra volume is offered on every sachet for free. The chart shows month-wise sales in the four states of western region.

31

Data Interpretation

Fixed cost per aircraft per trip Fuel cost per aircraft per km Cost per passenger Capacity

157.

A P Q R S T

161.

2) $ 1000 4) $ 1300

C

23% 28%

D

E 23%

27% 28% 30%

23% 20%

24%

If S was in third position in state C, P was fourth in state D, and R was last in state E, then which of the following could not be the percentage of valid votes garnered by Q in state D? 2) 9 % 4) 17%

Answer questions 162 and 163 based on the information given: Hardvar Institute of Management (HIM) was established in 2002 with 275 students, increasing the intake by 8 in 2003. Every student compulsorily undergoes a “Final Exam” after completing the 2year course, on passing which the degree is awarded. A student who fails, is given one more chance and allowed to re-take the examination a year later (i.e. along with his immediate juniors). A student who fails the second time is expelled, (i.e. compelled to leave without a degree).

If a group of N people want to travel 500km and want all the planes to be of the same type, then for which of the following values of N will the cheapest plane be the B727? 2) 30 4) 60

The following chart shows some statistics for HIM over the years 2004 to 2010:

If 100 people choose to travel a distance of 500 km at $100 per head, what is the maximum profit the airline can realize from the transaction, given that only 1 type of plane is used? 1) 31.31 % 3) 51.51 %

B 25%

1) 5 % 3) 13%

Put all 35 people in C225s Put all 35 people in B727s Put 30 people in a C225 and 5 in an A340 Put 15 people in a C225 and 20 in an A340

1) 12 3) 45 160.

C225 700 1.5 10 30

If 35 people wish to hire aircraft for a 100 km journey for a fixed amount, which would be the most cost-effective option for the airline? 1) 2) 3) 4)

159.

B727 600 1.1 20 25

What is the cost of sending 10 people for a distance of 200 km in an A340? 1) $ 900 3) $ 1100

158.

A340 500 1 30 20

350 300

2) 34 % 4) 66.66 %

250

315 287

299301

280

304 282

2

5

8

2006

2007

306 298

321

316

295

299

5

6

2009

2010

245

200 150

Answer questions 161: based on the information given:

100 50

Five parties P, Q, R, S and T are contesting elections in 5 states A, B, C, D and E. The table below shows the percentage of valid votes obtained by the top two parties in each state. In each state, it was observed that each of the five parties received a distinct number of votes. It was also observed that no party ended up in the same position in two different states.

0

0 2004

2005

Joined

162.

2008

Awarded degree

Expelled

How many students failed the Final Exam for the first time in 2006? 1) 22 3) 31

32

12

2) 30 4) Cannotbedetermined

Data Interpretation 163.

In which year did the maximum number of students appear for the Final Exam? 1) 2007 3) 2008

166.

2) 2009 4) 2006

1) Kenya achieved a score of 100 runs faster than Bangladesh. 2) Bangladesh achieved a score of 100 runs faster than Kenya. 3) Kenya and Bangladesh achieved a score of 100 runs in the same over in their respective innings. 4) None of these.

7 6.5 6 5.5 5 4.5 4 3.5 3 2.5 2 1.5 1 0.5 0 1

5

10

15

20

25

30

35

40

45

Directions for Questions 167 to 169: Answer the questions on the basis of the data given below.

Run Rate

Total Runs

Directions for Questions 164 to 166: Answer the questions on the basis of the data given below. 325 300 275 250 225 200 175 150 125 100 75 50 25 0

The U.S. Department of Transportation prepared a report on “Highway Traffic” for the year 2006. In this report, they analyzed the fatalities in the major states of the country. The following table gives the total fatalities reported in these sates. It also gives the number of fatalities related to three major factors: Motorcyclists, Speeding and Alcohol.

50

Total Overs Total Runs

Run Rate

A One-Day International (ODI) match is being played between Bangladesh and Kenya. Kenya won the toss and decided to bat first. The line graph gives the cumulative runs scored by Kenya in 50 overs and the run rate of Bangladesh upto 40 overs. The Run Rate, at any point, is calculated as the ratio of Total Runs to the Total Overs Played. 164.

165.

Speeding: Fatal traffic crash being speeding-related, i.e., exceeding the posted speed limit or driving too fast for conditions, which is one of the most prevalent factors contributing to traffic crashes. Alcohol: Fatal traffic crash being alcohol-related, i.e., if either a driver or a non-occupant (e.g., pedestrian) had a blood alcohol concentration (BAC) greater than the legal limit of intoxication.

What is the difference in the scores of the two teams at the end of the 40 th over? 1) 15 3) 30

2) 75 4) 35

Motorcyclists: Fatal traffic crash being motorcyclistrelated, i.e., if motorcycles have a fatal collision with a fixed object or any another vehicle. Also, if the motorcyclist was involved in any other fatal accidents.

In order to win the match, what should be Bangladesh’s minimum Run Rate for the last 10 overs? 1) 9.8 3) 7.7

Which of the following statements is definitely true?

2) 7 4) None of these

33

Data Interpretation Sr. No.

State

Total Fatalities

1 2 3 4 5 6 7 8 9 10 11 12 13 14 15 16 17 18 19 20

Alabama Arizona California Colorado Florida Georgia Ilinois Kentucky Louisiana Michigan Mississippi Missouri New York Ohio Oklahoma Pennsylvania South Carolina Texas Virginia Washington

995 1036 3753 681 2999 1541 1418 820 937 1382 949 1157 1458 1351 652 1520 1065 3769 930 632

Directions for Question 170 and 171: Answer the questions on the basis of the data given below.

Fatalities Related To Motorcyclists Alcohol Speeding 43 326 369 90 354 354 276 1061 1331 73 198 281 259 930 525 61 438 281 126 489 492 38 203 169 57 352 111 86 397 276 22 289 221 44 387 456 119 293 434 126 411 318 25 169 245 149 511 582 57 329 312 227 1450 1446 43 257 166 37 217 242

A shopkeeper holds a stock of commodities at the beginning of a month and sells that entire stock by the end of that month. The pie chart below gives the distribution of the quantities of the commodities in his stock at the beginning of May 2007.

Moong 8% Pigeonpea 5%

All the three types of fatalities need not add up to Total Fatalities, as there can be more factors involved other than these three major factors. In the report, the states were ranked on the basis of the total fatalities reported. The state in which the highest number of fatalities was reported is ranked 1, the state with the second highest fatalities is ranked 2, and so on. If any two states have the same number of fatalities reported, they share the same rank. Then, the next states in order will be ranked consecutively. Thus, no rank will be skipped. 167.

168.

Sugar 20%

Pigeonpea 4%

2) 4 4) 8

Moong 4% Rice 32%

Chickpea 15%

2) 6th 4) 8th

Sugar 25%

For how many states are fatalities related to Alcohol more than those related to Speeding? 1) 10 3) 13

Wheat 22%

The pie chart below gives the distribution of the profit earned by selling the different commodities by the end of May 2007.

What is the rank of New York when the states are ranked on the basis of the Total Fatalities? 1) 5th 3) 7th

169.

Chickpea 10%

If the states are ranked on the basis of fatalities related to Motorcyclists, how many ranks are shared by two or more states? 1) 3 3) 6

Rice 35%

2) 12 4) 14

170.

If the overall profit was Rs.6 per kg, then the profit earned per kg by selling Rice is: 1) Rs.5.5 3) Rs.3.5

171.

2) Rs.4 4) Rs.2.25

How much more profit can be earned by selling one kg of Chickpea over one kg of Moong? 1) 20% 3) 50%

34

Wheat 20%

2) 30% 4) None of these

Data Interpretation DIRECTIONS for questions 172 and 173: Refer to the data below and answer the questions that follow. Mr. Samuel plans an official 1-day trip to 3 countries – A, B and C from India. Following table gives Mr. Samuel’s flight departures/arrival timings in these countries in their respective standard times.

Directions for questions 174 to 176: Answer the question on basis of the following graphs The graph shows the number of fans (in hundred thousands) who either read comic books or watch the TV shows of different superheroes. The ones who watch the superheroes’ TV shows are further divided into three groups on the basis of their ages; below 15, 15 to 18 and above 18. No two superheroes have common fans.

Local Flight Timings in Countries India A B C (IST) (AST) (BST) (CST) Trip 1 From India to A 5:00 AM 6:00 AM Trip 2 From A to C 8:00 AM 9:00 AM Trip 3 From C to B 10:00 AM 12:00 PM Trip 4 From B to C 12:00 PM 5:00 PM Trip 5 From C to A 9:00 PM 6:00 PM Trip 6 From A to India 12:00 AM 10:00 PM -

Where IST: Indian AST: Country BST: Country CST: Country

Comic book 370 290 250

210

Spiderman

above 18

43

174.

175.

6 hrs. before returning back to India, Mr. Samuel was: In In In In

below 15

35

109 112

103

Batman

44

Superman

Flash

2. 0.26 4. 0.19

If it is known that 573 people like Batman, how many people like Batman TV shows as well as comic books? 1. 77 3. 496

country C a flight from country B to country C country A a flight from country C to country A

Flash

What is the approximate probability that a kid picked from TV show fans is below 18 and likes Batman or Superman? 1. 0.28 3. 0.22

2) 9:30 AM 4) 10:30 AM

295

74

60

What was the time (IST) when Mr. Samuel reached country C from A?

1) 2) 3) 40

15 to 18

93

Spiderman

173.

Superman

265

142

140

Standard Time A’s Standard Time B’s Standard Time C’s Standard Time

1) 9:00 AM 3) 10:00 AM

Batman

240

175

170

Time required by any flight for a forward journey is the same as the time required for the return journey. 172.

Television

360

2. 87 4. 650

Data Interpretation 176.

x Superman comic book fans convert to Spiderman comic book fans, y to Batman comic book fans and z to Flash comic book fans. Given that x, y and z are in AP and that the decrease in the number of superman comic book fans is

Time per week Dramatics 20%

Music 25%

of the total comic book fans,

what is the increase in the number of Batman fans in the comic book fans is what fraction of the total comic book fans? 1.

3 145

7 2. 650

3.

1 145

4.

Dance 25% Literary Arts 30%

7 290

177.

Directions for questions 177 to 179: The questions are based on the pie charts given below. The pie chart shows the distribution of students among various clubs in a school and the second pie chart shows the time for which these clubs operate per week. Assume that every student is a member of only one club.

1. 41 3. 52 178.

Students Dramatics 23%

Dance 32%

The total number of students remaining the same, if the number of students in literary arts club doubles and the relative ratios of students in other three clubs remain same, what is the approximate angle in degrees subtended by music club in the pie chart?

Literary Arts 27%

Music 18%

There are a total of 400 students in all the clubs combined. If 90 students from other clubs joined the literary arts club, the number of students in literary arts club becomes twice that of dance club and thrice that of dramatics club. What is the total number of student hours (product of number of students and the hours per day) in the music club if the total operating time of all the clubs combined is 70 hours a week? 1) 96 3) 78.5

179.

2) 88 4) 92.5

One of the conveners of the dramatics club falls ill as a result of which the average time spent in the dramatics club meetings falls by 25%. If this time is used by the literary arts club and the dance club in the ratio 3:4 respectively. What is the percentage increase in the time for which the dance club operates? (Assume data from the previous question if required). 1) 3 3) 23

36

2. 11 4. 72

2) 11.5 4) 34.5

Data Interpretation DIRECTIONS for questions 180 to 182: Refer to the data below and answer the questions that follow. Mr. Tanna had invested an amount of Rs.2,80,000 comprising five stocks A, B, C, D and E at the beginning of the year. The amount of his investments in these stocks is Rs.20,000, Rs.30,000, Rs.50,000, Rs.80,000 and Rs.1,00,000 not necessarily in that order. He

181.

1) Rs.278000. 3) Rs.200000. 182.

by 80%, C declined by 50% and E declined by 33.33 %. The amount received back at the end of the year on selling the four of the stocks is Rs.10,000, Rs.20,000, Rs.40,000 and Rs.2,00,000. Which of the following statements is true? 1) A priced at Rs.50,000 declined by to Rs.20,000 2) A priced at Rs.20,000 declined by to Rs.8,000 3) D priced at Rs.1,00,000 declined by to Rs.40,000 4) D priced at Rs.1,00,000 declined by to Rs.20,000

60% 60% 60% 80%

37

2) Rs.270000. 4) Rs.150000.

Stock D _____________. 1) 2) 3) 4)

5 saw his portfolio’s value decline by only % during 7 the calendar year. A declined by 60%, B declined

180.

The total value of the portfolio at the end of the year was _______.

Declined by 16.66%. Increased by 100%. Increased by 150%. Increased by 200%.

Data Sufficiency Data Sufficiency ªScore Enhancerª DIRECTIONS for question 52: Mark [1), if the question can be answered by statement I alone. Mark [2), if the question can be answered by statement II alone. Mark [3), if the question can be answered by using either of the statements alone. Mark [4), if the question can be answered by using both statements together but not by either statement alone. Mark [5), if the question cannot be answered, even by using both statements together.

Mark (4) If the question cannot be answered even by using both the statements together. 184.

I. By selling twelve chocolates, he makes a profit of the cost price of 1 chocolate II. By selling thirteen chocolates, he makes a profit of the selling price of 1 chocolate 185.

What is the ratio of speeds of A and B? I.

Answer question 183 based on the following information: In a ‘10-10’ tournament (each team plays a maximum of 10 overs), India played four matches. In all the four matches Sachin, Rahul, Saurabh and Yuvraj got the opportunity to bat. A reporter had made some notes, however, almost all of these were lost and only one page is available on which the following two rows are seen. These rows provide some information about the matches in order; e.g. 90 and 80 give some information about the 1st match.

183.

What profit percentage does the shopkeeper make on chocolates?

When A takes 7 steps, B takes 5 steps

II. The distance covered by A in a step is 2 3 of that covered by B in a step 186.

rd

A group of people consists of smokers and non-smokers. How many non-smokers are there? I.

The number of smokers exceeds the number of non-smokers by 3. II. The total number of people exceeds the number of smokers by 12. 187.

What is the total contribution of Sachin, Rahul, Saurabh and Yuvraj (considering all the matches) in terms of runs?

How many brothers and sisters are there in the Adams family? I.

Each boy in the family has twice as many brothers as he has sisters. II. Each girl in the family has thrice as many brothers as she has sisters.

I.

One of the rows represents only the runs scored by India in each match. II. One of the rows represents only the com bined percentage contribution of the four batsmen, in each match.

188.

DIRECTIONS for questions 184 to 192: Mark (1) If the question can be answered using one of the statements alone but not using the other alone.

In a running race, P beats Q by 50 m. When P completes the race, Q is 100 m ahead of R. If all, P, Q and R, run at a uniform speed, what is the length of the racetrack? I.

In the same race, P completes the race 10 seconds before R. II. R takes, in all, 120 seconds to complete the same race.

Mark (2) If the question can be answered using any of the statements alone . Mark (3) If the question can be answered using both the statements together but not using either statement alone.

39

Data Sufficiency 189.

191.

Is a = b? I.

I. Last digit of m is 7 II. Last digit of n is 9

(a + b) = 9

II. (a – 4)2 – (b – 4)2 = 0 192. 190.

What is the last digit of m n ?

What is the area of a rectangle with sides p and q?

Peter bought buns (at Rs. 7 each) and cakes (at Rs. 11 each) from a shop. How many buns did he buy?

I. p 3 – 3p 2q + 3pq 2 – q 3 = 8 II. (p + q) 2 = (p – q) 2 + 96

I. Peter spent a total of Rs. 115 II. Peter spent a total of Rs. 116

Data Sufficiency DIRECTIONS for question 53: Each question is followed by two statements. ªScore Maximiserª DIRECTIONS for question 193 to 197: Mark [1), if the question can be answered by statement I alone. Mark [2), if the question can be answered by statement II alone. Mark [3), if the question can be answered by using either of the statements alone. Mark [4), if the question can be answered by using both statements together but not by either statement alone. Mark [5), if the question cannot be answered, even by using both statements together. 193.

Is

194.

What is the maximum margin by which India won a match against its opponent? I. India did not win more than 2 matches. II. One of the rows represents India’s total score in each match and the other row represents the runs scored against India in that match.

Answer questions 195 and 196 based on the following information: Following data is available about a class:

b rational?

I. b is a root of the equation 4x2 – 37x + 63 = 0. II. b is not a root of the equation x3 – 10x2 + 23x – 14 = 0.

195.

How many boys liked Statistics? I.

The ratio of the number of boys who liked Statistics to the number of girls who disliked Statistics was 4 : 5. II. The ratio of the number of boys who disliked Maths to the number of boys who disliked Physics was 4 : 5.

Answer questions based on the following information: In a ‘10-10’ tournament (each team plays a maximum of 10 overs), India played four matches. In all the four matches Sachin, Rahul, Saurabh and Yuvraj got the opportunity to bat. A reporter had made some notes, however, almost all of these were lost and only one page is available on which the following two rows are seen. These rows provide some information about the matches in order; e.g. 90 and 80 give some information about the 1 st match.

196.

How many students liked Maths? I.

The ratio of the number of girls, who liked Maths to the number of girls, who disliked Physics is same as the ratio of the number of boys, who liked Maths to the number of boys, who liked Physics. II. There were 195 students in the class.

40

Data Sufficiency DIRECTIONS for questions 197 to 216: Mark [1) if the question can be answered using statement I alone but not using II alone Mark [2) if the question can be answered using statement II alone but not using I alone Mark [3) if the question can be answered using either statement alone Mark [4) if the question can be answered using statements I and II together but not using either alone Mark [5) if the question cannot be answered even by using both statements I and II together 197.

203.

I.

II. If another card is drawn without replacement, the probability that it is not a diamond is

Aparna, Amit, Anil, Anupam, Ajit, Arun and Anasuya are standing in a row for a photograph. Aparna and Anasuya are standing one at each corner and Anil, Arun and Ajit are standing together. Who is standing at the middle position?

204.

200.

201.

B Q

S D

R

C

PQRS is a rectangle inscribed in square ABCD with PS parallel to BD as shown in the figure. What is the perimeter of PQRS?

Is integer N a prime number? I. N = p! + 1 for some prime number p II. q! + 1 < N < q! + q for some prime number q

I. II.

l (AC) = 10 cm A( ∆ APS) = 16 sq

sq cm

What will be the remainder when 15 is divided by 60 (k is a positive integer)? 15k

k > 100

P

A

206.

II. k is odd

Can three sticks of length ‘a’, ‘b’ and ‘c’ form a triangle?

cm and A( ∆ BPQ) = 1

In a chess tournament, every child in the club played exactly one match against every other child. How many children were there in the club? I.

The number of matches between two girls was 8 more than the number of matches between two boys. II. The number of matches between one boy and one girl was 72.

I. ‘a’ is the Arithmetic Mean of ‘b’ and ‘c’ II. ‘a’ is the Geometric Mean of ‘b’ and ‘c’ 202.

A rectangle is cut along a diagonal to get two triangles. What is the area of each triangle?

205.

ABCD is a kite inscribed in a circle of radius 5 cm. What is the area of ABCD? I. BD is the diameter of the circle II. l (AB) = 4 cm

I.

13 17

I. The perimeter of the rectangle is 14 cm II. Two sides of one of the triangles are 3 cm and 4 cm

Amit and Anupam are not standing together. II. The number of people on one side of Arun is same as the number of people on the other side of Anil.

199.

If another card is drawn without replacement, the probability that it is a red queen is not 2 51

I.

198.

A card is picked at random from a wellshuffled regular pack of cards. What card is it?

The cost of 22 transistors, 25 resistors and 7 LEDs is Rs. 142. What is the cost of one transistor, one LED and one resistor?

207.

I.

The cost of 7 transistors and 6 LEDs is Rs.23 II. The cost of 5 transistors and 6 resistors is Rs.31

A function g is defined as g(n) = g(n – 1) – g(n – 2). What is the value of g(2001)? I. g(999) = 11 II. g(1000) = 13

41

Data Sufficiency 208.

What is the perimeter of hexagon ABCDEF?

212.

I.

The largest circle which can be drawn inside the hexagon has a radius of 3.5 cm. II. The largest circle which can be drawn inside the hexagon touches all the 6 sides. 209.

A biased die with faces bearing the numbers from 1 to 6 is rolled. What is the probability of getting a 6?

I.

I.

The probability of getting any number n is proportional to n 2. II. The probability of getting a perfect square or a prime is

In quadrilateral ABCD, AB is parallel to DC and the diagonals AC and BD intersect at O in the interior of ABCD. From the set of all triangles formed by joining points from the set {A, B, C, D, O} find the exact number of pairs of triangles (in any order) having equal area.

1  DC 3

AB = 

II. AD is not parallel to BC

55 . 91

213.

Tina’s speed is 10 m/s. What is the ratio of speeds of Nita and Tina? I.

210.

In a single race, Nitin beats Tina by 20 m and Tina beats Nita by 20 seconds II. In a single race, Nitin beats Nita by 60 m and Tina by 2 seconds

O, P, Q, R are distinct digits. What is the value of P × Q × R? I.

P + Q + R

214.

Find the value of xyz.

P Q II.

x, y and z are three distinct natural numbers such that their sum is 12. I. xy = 15 II. xy + yz + xz = 47

P Q + Q R

215.

Is |x – 3| < 2? I.

O P

|x| > 2

II. |x – 1| < 3

Note: Two letters one after the other represent a 2-digit number with those digits and not the product of the digits. 211.

216.

Eleven students took a test. Each of them scored a different non-negative integer score. What was the highest score? I. The median score was 6 II. The mean score was 6.

What is the first term of an Arithmetic Progression consisting of 9 positive integers? I. The sum of the 9 terms is 45 II. The 9 terms are all distinct

42

Data Sufficiency Answer questions 217 to 220 based on the following information:

219.

The 10 cricket players – A, B, C, D, E, F, G, H, I and J, are to be divided into 2 teams (5 players per team) which will play against each other. A, D, E and J are batsmen, B, D, F, G and I are bowlers while C, E and H are wicketkeepers. Each team must include atleast 1 batsmen, 1 wicketkeeper and 1 bowler. D and E are the captains of team 1 and team 2 respectively. F and G will not play together in a team. B and C play in the same team. 217.

I. During the trip, Sanjay covers a distance of 14 km relative to the water II. During the trip, Sanjay travels at 7 kmph relative to the water 220.

3 cards are drawn from a normal pack of 52 cards. What is the probability that the 3 cards are all aces? I. All the three cards drawn are black II. The cards are drawn without replacement.

For which team does ‘I’ play? I.

Each team has atleast 2 bowlers and atleast 2 batsmen II. Team 1 plays with 2 wicketkeepers 218.

Sanjay covers a distance of 20 km in a stream flowing at 3 kmph. How long does Sanjay take to complete the trip?

For which team does ‘H’ play? I.

There are exactly 2 wicketkeepers in the team in which ‘C’ plays II. Each team has either exactly 3 batsmen or exactly 3 bowlers

43

Logical Reasoning Logical Reasoning ªScore Enhancerª DIRECTIONS for question 221: Refer to the data below and answer the questions that follow.

223.

Four friends Abhishek, Bipin, Chetan and Deb are playing a game “Pass it on”. Initially each of them has 24 coins. The game starts with Abhishek passing a coin to Bipin, then Bipin passing 2 coins to Chetan, then Chetan passing 3 coins to Deb and then Deb passing 4 coins to Abhishek. At this point, one round is completed. In the 2nd round, Abhishek passes 5 coins to Bipin, Bipin passes 6 coins to Chetan and so on. The game ends when one person has all the coins and he is declared the winner. 221.

1) Gryffindor 2) Hufflepuff 3) Ravenclaw 4) Slytherin 5) Cannot be determined

ªScore Maximiserª DIRECTIONS for question 224: Refer to the data below and answer the questions that follow. Four friends Abhishek, Bipin, Chetan and Deb are playing a game “Pass it on”. Initially each of them has 24 coins. The game starts with Abhishek passing a coin to Bipin, then Bipin passing 2 coins to Chetan, then Chetan passing 3 coins to Deb and then Deb passing 4 coins to Abhishek. At this point, one round is completed. In the 2nd round, Abhishek passes 5 coins to Bipin, Bipin passes 6 coins to Chetan and so on. The game ends when one person has all the coins and he is declared the winner.

Find the number of coins with Bipin in the 8th round just after he has received the coins from Abhishek. 1) 32 3) 45 5) None of these

2) 44 4) 46

DIRECTIONS for questions 222 and 223: Answer the questions based on the data below: In Hogwarts, there are 4 groups - Gryffindor (G), Hufflepuff (H), Ravenclaw (R) and Slytherin (S). Each group has 12 students - numbered G 1 to G 12, H 1 to H12, etc. Every year, the students in each group get identical caps with the group name on them. Each student's cap was kept in a box having his number on it. Filch, the meddlesome caretaker, interchanged the caps in the following manner in that order: (E.g. G1 ↔ H1 denotes that the cap in box G1 was interchanged with the cap in box H1.) i) First, G 1 ↔ H1, G2 ↔ R2, G3 ↔ S3, G4 ↔ H 4 … G 12 ↔ S 12 ii) Then H1 ↔ R1, H2 ↔ S2, H3 ↔ G3, H4 ↔ R 4 … H 12 ↔ G 12 iii) Then R1 ↔ S1, R2 ↔ G2, R3 ↔ H3, R4 ↔ S 4 … R 12 ↔ H 12 . iv) Then S1 ↔ G1, S2 ↔ H2, S3 ↔ R3, S4 ↔ G 4,… S 12 ↔ R 12. Each student then collected the box having his number on it. 222.

The cap that was originally last in the Slytherin group would now belong to a student of:

224.

Who will win the game? 1) Abhishek 2) Bipin 3) Chetan 4) Deb 5) Cannot be determined

DIRECTIONS for question 225: Refer to the data and answer the questions that follow. In the game 'Hot and Cold' there are some trenches in a row. In Raghav's turn, Raman hides a treasure beneath one of the trenches. In each step Raghav taps on a trench. As shown in the illustration below: • If it contains the treasure, Raman opens it and Raghav's turn ends. • If it is adjacent to a trench containing the treasure, Raman calls out 'hot'. • If there are exactly one or two trenches between this trench and the one containing the treasure, Raman calls out 'warm'. • If it is none of the above, Raman calls out 'cold'.

Which group's cap would the 8 th student in the Hufflepuff group get? 1) Gryffindor 2) Hufflepuff 3) Ravenclaw 4) Slytherin 5) Cannot be determined

45

Logical Reasoning Raghav tries to end his turn in the minimum number of steps.

cold

treasure

cold

hot

225.

warm

warm

warm

warm

hot

cold

cold

228.

1) Arun sits next to someone who uses same number of sachets as him. 2) Joe sits next to someone who eats the same number of cookies as him. 3) Raman sits next to someone who uses same number of sachets as him. 4) Sam sits next to someone who eats the same number of cookies as him.

treasure

cold

If Raghav played his turn intelligently and took three steps to end it, then what is the minimum possible number of trenches in that game? 1) 3 3) 6 5) 8

DIRECTIONS for questions 229 to 232: Answer the questions based on the following information:

2) 4 4) 7

Eight members of a trekking club - A, B, C, D, E, F, G and H – plan to go trekking this weekend. Two simultaneous treks – Trek I and Trek II - are planned, each of which will have exactly 4 participants. A, C and G are doctors. A, B, E and H have prior trekking experience. It is decided that each group should have at least 1 doctor and 2 members with prior experience. Also B and C must be in different groups.

ªScore Enhancerª DIRECTIONS for questions 226 to 228: Refer to the data below and answer the questions that follow. Four friends are sitting at the four sides of a square table at a Café. Eighteen cookies and many sachets of sugar, all of equal size, are kept at the table. The one opposite Sam has black coffee with twice as many sachets of sugar as him and three cookies more than him. Joe has half as many sachets of sugar but twice as many cookies as the one opposite him. Sam has the same number of cookies as one of his friends. Raman has more sugar in his coffee than Arun and also adds milk to it. No one uses a sachet partially or has more than four sachets. 226.

227.

229.

If D and F insist in going on Trek II, which of the following must also go on Trek II? 1) G 3) B

230.

Black, with 1 sachet of sugar With milk and 3 sachets of sugar Black, with 2 sachets of sugar Cannot be determined

231.

1) The one opposite Arun uses 2 sachets of sugar less than him. 2) The one opposite Raman eats 3 cookies more than him. 3) The one opposite Joe uses 1 sachet of sugar more than him. 4) The one opposite Sam eats 5 cookies.

46

1 way 2 ways 3 ways No such group possible

If A is the only doctor in his group, then which of the following is necessarily true? 1) 2) 3) 4)

Which of the following is true?

2) E 4) H

If F and C go on Trek II, then in how many ways can the group for Trek I be formed? 1) 2) 3) 4)

How does Arun have his coffee? 1) 2) 3) 4)

Which of the following is false?

B E D D

and and and and

G G F F

go go go go

on on on on

the same trek. different treks. the same trek. different treks.

Logical Reasoning 232.

If A and B go on different treks, which of the following could be true? 1) 2) 3) 4)

A D E F

and and and and

G F H H

go go go go

on on on on

the the the the

same same same same

236.

1) Mini’s pawn 2) When Mini’s got 8 heads 3) Mini’s pawn 4) When Mini’s got 6 tails

trek. trek. trek. trek.

DIRECTIONS for questions 233 and 234: Use the following data to answer the questions

234.

Ganpat is next to Bittu. Bittu is the second from right. Esha is next to Charan. Divya is at the centre.

If there is exactly one student between Bittu and Ashay, then Esha cannot be next to: 1) 2) 3) 4)

Bittu or Divya Charan or Ganpat Ganpat or Ashay Bittu or Ashay

237.

Which of the following is the last letter in the password? 1) E 3) M

Data for Questions 235 and 236: Mini is playing with a pawn on an empty chessboard. She places the pawn on the bottom-left corner square and tosses a coin. If it shows heads, she moves the pawn two squares up or down; if tails, she moves it one square to the right or left. Then she again tosses the coin and moves the pawn in the same manner and continues the process. The pawn may arrive at the same position twice. Note: (a, b) denotes the square in the a th column from left and b th row from bottom. 235.

reached (6, 7) in 11 moves pawn was at (4, 3) she had

Data for questions 237 and 238: Prathamesh gave an eight-lettered password to his personal folder on his computer using all the letters in his name exactly once. His younger brother, Rakesh, who is interested in reading the contents of this folder, collects the following information about the password. i. No two letters adjacent in his brother’s name are adjacent in the password. ii. Only two letters are at their original position and one of them is the fourth letter to the right of T in the password. iii. There are exactly three letters between R and H. iv. R, S and T are together with R and S being adjacent.

Which of the following is not possible? 1) 2) 3) 4)

reached (5, 4) in 6 moves pawn was at (6, 5) she had

ªScore Maximiserª

Seven students are sitting in a row. Bittu is closer to Charan than to Divya. There are exactly four students between Ashay and Ganpat. The only student sitting farthest from Esha is Mohit. 233.

Which of the following is possible?

238.

2) A 4) P

The password cannot start with: 1) A 3) T

2) H 4) None of these

ªScore Enhancerª Answer questions 239 to 241 based on the following information: A flower garden has 5 parallel rows of flowers. The gardener has a choice of 7 different types of flowers, Azaleas, Begonias, Camellias, Dahlias, Gardenias, Petunias and Zinnias. He observes certain rules every year while planting the garden, viz: a) No two adjacent rows contain the same type of flower b) A single type of flower can be planted in not more than two rows c) If Azaleas are planted, Dahlias also must be planted

Mini’s pawn was at (4, 5) when she stopped. Which of the following is not possible? 1) Mini had got 5 tails in all 2) Mini had got 2 more tails than heads 3) The number of heads Mini had got was 1 less than the number of tails 4) The number of heads Mini had got was 3 more than the number of tails

47

Logical Reasoning d) e) f) g)

239.

Gardenias are not planted unless Petunias are also planted Camellias can not be planted immediately next to Dahlias At least two out of Azaleas, Camellias and Zinnias must be present in a given planting Begonias and Petunias must be planted together, though not necessarily in adjacent rows

The following information on the daily schedule for the current week is known: Screen 1 Show 1 Show 2 Show 3 Show 4 Show 5

242.

Which of the following, in order, cannot be planted in a given year?

241.

2) Gardenias 4) Camellias

If Dahlias are planted in the second and fourth rows, then the other three rows, in order, can contain 1) 2) 3) 4)

243.

Genre

Die Hard Transformers

244.

Action

Action

Shutter

Exorcist Hangover

Horror

Horror

Screen 5 Hangover

Die Hard

The movie, screened as the 2nd show on screen 1, belongs to which genre? 2) Horror 4) Cannotbedetermined

If N is the 6 th speaker, which of the following slots could be allocated to R? 1) 1st 3) 7th

Azaleas, Zinnias, Begonias Azaleas, Azaleas, Zinnias Azaleas, Camellias, Zinnias Camellias, Azaleas, Gardenias

2) 3rd 4) 8th

Which of the following, in order, could be a possible schedule for the morning session? 1) Q, M, N, P 3) Q, R, S, N

DIRECTIONS for question 242: Refer to the data below and answer the questions that follow. A multiplex has 5 screens screening 5 shows each, one after the other, every day. Currently 5 movies are being shown belonging to 3 different genres – Horror, Action and Comedy. No two consecutive shows on any screen belong to the same genre. No movie is shown simultaneously in any of the screens and no movie is shown more than once on any screen in a day. All the shows start at the same time and assume that all the movies are of equal duration. Movie

Screen 4

Answer questions 243 to 245 based on the following information: (Select best 3) Eight speakers L, M, N, O, P, Q, R and S are to be scheduled to speak in eight slots, one after another, the first four in the morning session and the rest after a lunch break in the afternoon session. It is also known that: P must speak in the afternoon session. R cannot be the first speaker in a session. M and N must be awarded consecutive slots (not necessarily in that order). Q and O must not be scheduled to speak in the same session.

If the first and third row contain Azaleas and the second has Camellias then the fifth row can have 1) Zinnias 3) Petunias

Screen 3 Shutter

Transformers

1) Action 3) Comedy

1) Camellias, Zinnias, Dahlias, Azaleas, Camellias 2) Azaleas, Dahlias, Zinnias, Camellias, Azaleas 3) Azaleas, Dahlias, Azaleas, Gardenias, Camellias 4) Begonias, Petunias, Gardenias, Camellias, Zinnias 240.

Screen 2

245.

If the first two speakers are S and M, which of the following can be true? 1) 2) 3) 4)

Comedy

48

2) R, O, Q, S 4) S, M, N, R

L R Q N

can can can can

be be be be

the the the the

4 th 5 th 6 th 7 th

speaker speaker speaker speaker

Logical Reasoning DIRECTIONS for questions 249 to 251: Refer to the data below and answer the questions that follow.

Answer questions 246 to 248 based on the following information: A factory starts a certain manufacturing job on August 1, 2009. The job involves seven processes: A, B, C, D, E, F and G. Initially, only processes D and E can be started; D takes 12 days for completion, while E requires only 7 days. Process B, which requires 5 days, can be started only after both D and E are finished. Process C, which takes 8 days, can be started after E is completed. Process F, which requires 5 days, can be started only after D and C finish. Process G, which takes 2 days, may commence only after B and C are over. Process A, lasting 6 days, can commence only after all the other processes end. Processes can be carried out simultaneously. 246.

248.

Teams/Ro Points in unds Round 1 T1 3 T2 1 T3 2

Which is the earliest date on which the project could be completed? 1) August 21 3) August 26

247.

6 players are divided in 3 teams – T1, T2 & T3, with 2 players each. They are playing a game of dice in which the absolute difference between face values of the die cast by teammates is recorded & the team with highest absolute difference is declared as winner. The winner gets 3 points, runners up get 2 points & loser gets 1 point. Absolute differences of all teams in a round are always different. All players get different scores on the die in a round. The table below records performances of the teams in the two rounds

2) August 25 4) August 20

Points in Round 2 3 2 1

If there are a dozen extra workers who can be deployed for process D, thereby halving the time for that process, then which is the earliest date on which the project could now be completed?

Additional data:

• •

One of the members of T1 scored a 5.

1) August 21 3) August 26



Absolute difference of only 1 team in round 2 is same as that of a team in round 1.



The team, with an odd Absolute difference in round 2, was not the loser in that round.

2) August 25 4) August 20

If the extra workers from the above question could be deployed for exactly one process out of A, B, C, D, E, F and G, thereby halving the time for that particular process, for which process should they be deployed to maximize the reduction in project duration? 1) 2) 3) 4)

249.

Absolute difference of only 1 team was odd, in round 2.

What was the score of B if he was the highest scorer for T3 in Round 1? 1) 5 3) 4

A C D No reduction is possible

250.

Which team never cast a 6? 1) T2 3) T3

251.

2) T1 4) Cannotbedetermined

The sum of the face values of the die cast by the members of T1 is: 1) 11 3) 13

49

2) 3 4) Cannotbedetermined

2) 12 4) Cannot be determined

Logical Reasoning DIRECTIONS for questions 252: Refer to the data below and answer the questions that follow. 5 teams – Nifty Ninjas, Feral Falcons, Devil Hunters, Laughing Lamas, Rocket Rangers participated in a football competition. In Round 1, there were in all 10 games where each team played against every other team exactly once. Thus, every team played 4 matches. The table below shows the number of goals scored by a team in a match and the number of wins, losses and draws. Match 1

Match 2

Match 3

Match 4

W

L

D

0

2

2

0

1

2

1

1

2

3

2

4

0

0

1

1

2

1

2

1

1

2

1

2

0

1

1

2

1

0

1

1

0

4

0

Devil Hunters Nifty Ninjas Laughing Lamas Feral Falcons Rocket Rangers

253.

1) 2) 3) 4) 254.

255.

What was the score line of the game between Laughing Lamas and Nifty Ninjas? 1) 2) 3) 4)

(LL) 1 – 3 (NN) (LL) 1 – 2 (NN) (LL) 2 – 3 (NN) Either (2) or (3)

DIRECTIONS for questions 253 to 255: Refer to the data below and answer the questions that follow. A, B, C and D are four passengers who travel from the same place, Lincoln street to Park Street by train every day. A, B, C and D travel by the trains which start at Lincoln street at 8 A.M., 8:30 A.M., 9 A.M. and 9.30 A.M. respectively while going to Park street and they return by trains which start at Park street at 7 P.M., 7.30 P.M., 8 P.M. and 8.30 P.M. respectively. The speeds of trains starting at different times are different. The speed of the trains starting at 8 A.M. and 8 P.M. are equal and higher than all other trains. The total time spent in the to and fro journey is the same for all four people. Also the sum of time taken for all journeys from Lincoln Street to Park Street is the same as that from Park street to Lincoln Street.

8 A.M. and 8 P.M. 9.30 A.M. and 7 P.M. 9 A.M. and 8 P.M. 9 A.M. and 7 P.M.

Let t1, t2, t3, t4 be the time taken by trains starting at 8, 8.30, 9 and 9.30 A.M respectively to travel from Lincoln Street to Park Street and t5, t6, t7 and t8 be the taken by trains at 7, 7.30, 8 and 8.30 P.M respectively to travel from Park street to Lincoln Street. If the total time spent by A, B, C and D in journey is t seconds each, which of the following statements represent conditions that are not possible in the given constraints? 1) 2) 3) 4)

As the competition was held on the home ground of ‘Nifty Ninjas’, they played the first game. Also, no team played consecutive games. 252.

At what timings do the trains taking the maximum amount of time for the journey start?

t2 t5 t1 t1

= < < ×

t8. t7 < t8 t2 < t3 t5 = t7 × t3

On a certain day, in the morning, A, B and C, missed their regular trains and hence took the next immediate train. During this journey, it was observed that the time taken by A is greater than the time taken by C. Which of the following relations is true? (The symbols used imply the same as explained in the earlier question. All the other previously stated conditions hold true.) 1) 2) 3) 4)

t7 t1 t7 t4

< < < <

t8 t2 t6 t1

< < < <

t5 t4 t8 t2

< < < <

t6. t3. t5 t3.

Answer questions 256 to 259 based on the information given: In a recent inter-college competition, two students each from five colleges – P, Q, R, S and T. participated in a 10 Test Series conducted by ABC Institute of Computer Studies. The scores of the students in the 10 tests were classified into four ranges: 0 to 25; 26 to 50; 51 to 75 and 76 to 100. The range-wise breakup of scores of every student is given in table 1. The total scores in a range for students from any particular college is given in table 2.

50

Logical Reasoning Table 1 0-25 2 1 3 3 5 3 2 1 6 2

Amar Bijoy Carl Dev Elen Farhan Geet Hiten Inder Jeet

26-50 3 4 3 2 2 1 4 5 1 1

Answer questions 260 and 261 based on the information given: 51-75 4 1 1 4 2 4 1 2 2 5

76-100 1 4 3 1 1 2 3 2 1 2

A team of 3 dentists and 4 pathologists sit around a circular table. No two dentists sit together. i) B and C sit farthest from A. ii) F sits between D and a pathologist iii) E and G are of the same profession 260.

I. II. 1) 2) 3) 4)

Table 2 0-25 5 4 9 7 3

P Q R S T

256.

261.

2) Q 4) T

2) Bijoy 4) Dev

How many of the following statements are definitely false?

Directions for Questions 262 to 264: Answer the questions on the basis of the data given below. Five colleges, P, Q, R, S and T, participated in an intercollegiate tournament comprising five events: Swimming, Running, Cycling, Discus Throw and Horse Riding. In each of the events of this tournament, the top 3 colleges were awarded a gold, a silver and a bronze medal. Shamik, one of the spectators, recalled the following information. i. Each college won at least one medal but not more than two medals of the same type, i.e., no college won more than two gold medals, more than two silver medals, or more than two bronze medals. ii. College Q won gold medals in two games, in which college P won bronze medals. iii. College T won a gold and a silver medal but not a single bronze medal.

Amar & Dev belong to the same college Carl and Elen belong to the same college Geet and Hiten belong to the same college Bijoy and Dev belong to different colleges

Which of the following is the right studentcollege combination? 1) Farhan – Q 3) Hiten – T

E sits next to B B is a pathologist Only I or only II Either I or II Both I and II Neither I nor II

I. A dentist sits between E and C II. Four people sit between G and D III. F sits at an equal distance from A and one of the pathologists 1) Exactly one 2) Exactly two 3) All three 4) None of the given statements is definitely false

Which of the following statements is true? 1) 2) 3) 4)

259.

76-100 3 5 4 3 5

Which of the following students belongs to College R? 1) Amar 3) Carl

258.

51-75 8 5 3 7 3

Amar belongs to which college? 1) P 3) S

257.

26-50 4 6 4 3 9

Which of the following statements is/are true?

2) Elen – P 4) Jeet - R

51

Logical Reasoning iv. v. vi.

Only R won 4 medals, the highest number of medals won by any college. R, S and Q won a gold, silver and a bronze medal respectively in Discus Throw. P and S won a gold and a silver medal re spectively in Swimming.

262.

264.

266.

2) P 4) S

Which of the following colleges, won a silver medal in Running? (Refer to the data in previous question) 2) P 4) S

267. Directions for Questions 265 and 266: Answer the questions on the basis of the data given below.

268.

No player played against the same player twice.

269. 2 2 0 1 1 0 2 2 0

3 2 1 1 2 0 0 0 2

4 1 2 1 1 1 0 1 1

5 1 2 2 0 1 2 0 0

6 2 0 0 1 2 0 2 1

7 1 2 1 1 1 1 1 0

52

2) J is in team Z 4) C is in team Y

Which of the following can be a valid team? 1) Q, T, C, K 3) B, C, P, J

Use data from previous questions

2) T 4) Q

If B, R and S are in team Y and T, K and D are in team X, then which of the following is compulsorily true? 1) Q is in team Z 3) L is in team Y

The table shows the points scored in rounds one to seven. 1 2 2 1 1 1 1 0 0

If L and M are in team X and J, R and S are in team Z, then which of the following is not in team Y? 1) K 3) C

In an 8 player chess tournament seven rounds are played, with each player playing every other player exactly once. For a win a player gets 2 points, a loss 0 points and for a draw 1 point goes to each player.

Player Garry Anand Kramnik Topalov Shirov Carlsen Bareev Polgar

2) Two 4) Cannot be deter-

Answer questions 267 to 269 based on the following information: Three teams X, Y and Z of four students each are to be formed for a college quiz. The twelve available students are P, Q, R, S and T from Science stream, B, C and D from Arts stream and J, K, L and M from Commerce stream. P, R, S, B and M are boys and the rest are girls. Each team must have at least one student from each stream and at least one boy. P is in team X.

If college T won a gold in Running and a silver in Cycling, who won a bronze in Horse Riding?

1) R 3) Q

2) Carlsen 4) Cannotbedetermined

Garry played Polgar in Round ______. 1) One 3) Four mined

2) 1 4) 3

1) Q 3) R

If Topalov drew with Carlsen in Round 7, then whom did Anand lose to in Round 2? 1) Garry 3) Bareev

How many bronze medals did college R win in these five events? 1) 0 3) 2

263.

265.

2) P, R, C, L 4) M, B, P, S

Verbal Ability Verbal Ability ªScore Enhancerª DIRECTIONS for questions 270 to 273: In each question, there are four sentences. Each sentence has a pair of words that are italicized and highlighted. From the italicized and highlighted words, select the most appropriate words (A or B) to form correct sentences. The sentences are followed by options that indicate the words, which may be selected to correctly complete the set of sentences. From the options given, choose the most appropriate one. 270.

272.

There was subsequent outage (A) / outrage (B) from several sections of the media and public. The article was criticized (A) / critiqued (B) by several intellectuals and academicians alike. Thereafter the author was looked down upon as being an immoral (A) / amoral (B) influence upon the youth. 1) AABA 3) ABAA 5) ABAB

The students accessed (A) / assessed (B) their results through a unique PIN provided to them. Most of the students had performed exceptionably (A) / exceptionally (B) well.

273.

However, university professors adviced (A) / advised (B) the students against getting carried away by success.

271.

For someone who was a top-ranked actress, she lives a rather closeted (A) / closed (B) life.

But as to her own lifestyle (A) / livelihood (B) her friends beg to differ with the public notion.

2) BABA 4) AABB

Perhaps being in the limelight for over a decade brings about an inevitable (A) / incontrovertible (B) change in your outlook. 1) AAAB 3) BABA 5) BAAA

She spoke with an American accent (A) / assent (B). She said that after having lived 10 years in India, she had hoped to lose (A) / loose (B) it.

2) AABB 4) AAAA

DIRECTIONS for questions 274 to 275: In each of the questions, a word has been used in sentences in five different ways. Choose the option corresponding to the sentence in which the usage of the word is incorrect or inappropriate.

Unfortunately, she had unintentionally preserved (A) / persevered (B) it. She promised to work with (A) / work on (B) this problem in the future. 1) AAAB 3) ABAB 5) BBBA

2) BBAB 4) BABB

She attributes (A) / attritions (B) this to the rather debasing world of glamour and films.

In spite of this, the mood amongst the student community that day was far from sober (A) / sombre (B). 1) AAAB 3) ABBB 5) ABBA

The boldness (A) / baldness (B) of that statement shocked conservative English society.

274.

2) AAAA 4) BAAA

Draw 1) Saying so, he began to draw out a sheaf of legal papers from his right pocket. 2) After the league match that ended in a draw, Australia have little chance of making the semi-finals. 3) One should never draw a blank cheque if one does not trust the drawee. 4) Draw in the colours very neatly or you will smudge the painting. 5) With everything at stake, he foolishly decided to take part in the lucky draw to recover some money.

53

Verbal Ability 275.

Cream

278.

1) Would you like some cream in your coffee? 2) We finally decided to paint the bedroom a shade of cream. 3) The first course at the dinner consisted of cream of tomato soup. 4) She applies cream on her hands and face every night before going to bed. 5) The new management is so inept, that they fired all the good employees, leaving only the cream of the crop.

1) Intelligence certainly is not predictor of success in work or but its as strong as any. 2) Intelligence is certainly not predictor of success in work or but its as strong as any. 3) Intelligence is not certainly predictor of success in work or but its as strong as any. 4) Intelligence is not certainly predictor of success in work or but it's as strong as any. 5) Intelligence certainly is not predictor of success in work or but it's as strong as any.

DIRECTIONS for question 276: Each of the following questions has a sentence with two blanks. Given below each question are five pairs of words. Choose the pair that best completes the sentence. 276.

Nurturing young people's minds might not _______ their DNA, but it does help them achieve their intellectual _______. 1) 2) 3) 4) 5)

279.

quash … quotient eradicate … zenith quell … capacity annihilate … aptitude override … potential

280.

the only in school, the only in school, the only in school, the only in school,

Having lived in small towns her whole life, nothing about human nature ever surprised her.

Granny Weatherwax poured her cup with the tea, and picked up her cup and saucer. 1) 2) 3) 4) 5)

Further, the instruments capable of this lay still farther in the future, primarily because they are so expensive. 1) Further, the instruments capable lay still farther in the future, 2) Further, the instruments capable lie still further in the future, 3) Farther, the instruments capable lie still further in the future, 4) Farther, the instruments capable lie still farther in the future, 5) Farther, the instruments capable lay still further in the future,

the only in school,

1) nothing about human nature ever surprised her. 2) anything about human nature never surprised her. 3) human nature never surprised her. 4) she was never surprised by anything about human nature. 5) she was ever surprised by nothing about human nature.

DIRECTIONS for questions 277 to 280: In each of the following sentences, a part of the sentence is highlighted. Beneath each sentence, five different ways of phrasing the underlined part are indicated. Choose the best alternative from among the five. 277.

Intelligence certainly is not the only predictor of success in work or in school, but its as strong as any.

of this of this of this of this of this

54

poured her cup with the tea, poured the tea into her cup, poured her cup the tea, pored her cup with the tea, pored the tea into her cup,

Verbal Ability 4) It is the custom in India for weddings to be elaborate events involving all of one's family and friends. 5) She had had so little free time recently that she had to give up her custom of reading for an hour every evening.

DIRECTIONS for questions 281 to 283: Each of the following questions has a sentence with two blanks. Given below each question are five pairs of words. Choose the pair that best completes the sentence. 281.

Feeling giddy in the _____ heat, and disillusioned by the _____ treatment meted out to him, he decided to just lie down under the shade of a tree and doze off. 1) 2) 3) 4) 5)

282.

oppressive; partisan scorching, step-motherly repressive; permissive malignant; unfathomable unbearable; detrimental

285.

Driven to desperation by a desire to _____ her sister's death, she resorted to making nasty _____ about the accused, much to the consternation of all present in the court. 1) 2) 3) 4) 5)

283.

DIRECTIONS for question 285: Each of the following questions has a paragraph with one italicized word that does not make sense. Choose the most appropriate replacement for that word from the options given below the paragraph.

1) minus 3) deficient 5) inadequate

pay off; conclusions redeem; accusations revenge; pointers avenge; insinuations condemn; fabrications

maniacs; appropriate heretics; averse insane; conditioned reprobates; predestined atheists; antithetical

What is Professor Pope's basic assumption? 1) Written works throughout history would accurately record representative samples of the psychological abnormalities of human beings. 2) The human brain has a multitude of capabilities, including depression, hallucinations, etc. 3) Historical records may provide a clue regarding the causes and cure of dissociative amnesia. 4) Dissociative amnesia has so far been assumed to be a recent psychological phenomenon. 5) Dissociative amnesia is in fact not an innate capability of the human brain.

DIRECTIONS for question 284: In each of the questions, a word has been used in sentences in five different ways. Choose the option corresponding to the sentence in which the usage of the word is incorrect or inappropriate. 284.

2) scarce 4) bereft

DIRECTIONS for questions 286 to 288: Read the short passages given below and answer the question that follows each. 286. In a recent study, professor of psychiatry Harrison Pope put dissociative amnesia - i.e., 'repressed memory' - to the test of time. He reasoned that if dissociative amnesia were an innate capability of the human brain - akin to depression, hallucinations, anxiety and dementia - it would appear in written works throughout history.

During the Middle Ages, intellectuals who published works contrary to Church teachings were deemed _____ and considered _____ to damnation. 1) 2) 3) 4) 5)

But if this triumph occurs, astronomers will find themselves vurblefub of the information that they would most dearly like to obtain: what conditions exist on these planets, and are they suitable for life?

Custom 1) He is so rich, that he drives a custombuilt car. 2) He was not custom to waking up so early, but somehow he managed. 3) The small grocery shops have lost a lot of custom since the supermarket chains opened up.

55

Verbal Ability 287.

288.

London, the city often denigrated as 'the Smoke', is the dirtiest EU capital apart from Athens, according to a survey. Severe traffic congestion causing high levels of air pollution and problems with rubbish collection combine to make the city an environmental black spot. ‘These are sprawling cities with public transport problems and severe traffic congestion, producing a detrimental effect on air quality,’ said Slagin Parakatil, senior researcher at William M. Mercer, the consulting firm which produced the survey. ‘Waste disposal systems are also under pressure from the cities' dense populations, making them less efficient than in other EU cities.' Which of the following could seriously undermine the argument put forth by Mr. Parakatil, as regards London? 1) New evidence has come to light which suggests that pollution levels in London have remained consistently high due to the presence of textile industries on the outskirts of the city. 2) Pollution levels in both London and Athens have been the lowest in the past month as compared to the last six months. 3) Nearly 30% of the owners of two- as well as four-wheelers in London have installed pollution control technology into their vehicles in the last 6 months. 4) Pollution levels in London are high not because of traffic problems and inefficient waste disposal systems, but because of the increasing presence of polluting industrial units within the city itself. 5) The credibility of the method and apparatus used to measure pollution levels in London is being challenged in the courts of Britain.

The health minister recently argued for a policy of Prohibition to be implemented across the country. While making it clear that there were no plans for a nationwide implementation of this policy, he strongly opposed the growing culture of alcohol abuse amongst youngsters in the metropolises. According to a government survey, the average age of commencement of alcohol use in India has reduced from 28 years in 1990 to 19 years in 2008. He pointed out that in metropolises like Bangalore and Chennai, this culture was the most rampant. According to another survey, by a well-known NGO, in New Delhi, alcohol is available across the counter and does not require any license. Most importantly, according to the survey, youngsters as young as 15 could obtain alcohol from places like petrol pumps. From this data, the NGO concluded that alcoholism is becoming a major problem in India. Which of the following, if true, significantly weakens the conclusion stated in the above passage? 1) The agencies involved in the surveys have totally overlooked the problem of alcohol abuse prevalent in rural India, which is by some estimates far graver than the problem in the metropolises. 2) The members of the NGO and the government have not been able to reach a consenses regarding the surveys. 3) The methods used to conduct the surveys are faulty in the sense that they concentrate on the percentage of people who consume alcohol, and not the duration of the addiction or any specific health effects of it. 4) The surveys have concentrated on only a specific section of the country's populace, and drawn conclusions based on the preferences of a very select and insignificant income group in society. 5) The surveys have not received official sanction from the executive organ of the government.

56

Verbal Ability DIRECTIONS for questions 289 to 291: In each question, there are five sentences/paragraphs. The sentence/paragraph labelled A is in its correct place. The four that follow are labelled B, C, D and E, and need to be arranged in the logical order to form a coherent paragraph/ passage. From the given options, choose the most appropriate option. 289.

290.

for their food. 1) EDBC 3) EBDC 5) DBCE 291.

A. Historians of the deep human past generally consider cooking to be a recent activity. B. This is a fantastically weird way of looking at evolutionary change. If cooking pushed us across a species threshold, it means that our biology is also shaped in completely unintended ways by cultural innovations. C. By contrast, Richard Wrangham, a biological anthropologist, thinks we were cooking 1.8 million years ago - and that the activity was not an outcome of being human but that being human was an outcome of cooking. D. According to him, cooking physically transformed a creature that was more ape into the earliest version of us, Homo erectus. E. A significant number of hearths have been unearthed around the 76,000 year mark, and there is diminishing archaeological evidence of controlled fire the further back you go. The assumption is that once we became modern, we worked out how to cook. 1) BCDE 2) CDBE 3) CEDB 4) ECDB 5) ECBD

2) DEBC 4) BCDE

A. Nietzsche's unpublished writings often reveal his more tentative and speculative ideas. B. Disagreement regarding Nietzsche's notebooks centres around the degree of interpretive priority which ought to be given to the unpublished versus the published manuscripts. C. This material is surrounded by controversy, however, since some of it conflicts with views he expresses in his published works. D. The proponents of this school of thought believe that these texts should take precedence over the unpublished manuscripts when conflicts arise. E. One popular approach opines that the tradition of classical scholarly interpretation is to maintain that Nietzsche's published works express his more considered and polished views. 1) CBED 2) CEBD 3) CBDE 4) BCED 5) EDCB

Directions for questions 292 and 293: The sentences given in each question, when properly sequenced, form a coherent paragraph. Each sentence is labelled with a letter. Choose the most logical order of sentences from among the given choices to construct a coherent paragraph. 292.

A. The earliest traces of people on the Isle of Man can be found as far back as the Mesolithic Period, also known as the Middle Stone Age. B. They used small tools made of flint or bone. C. Representatives of these artefacts are kept at the Manx Museum. D. These have been found near the coast. E. The first residents lived in small natural shelters, hunting, fishing and gathering

57

A. They gradually ascended for half-a-mile, and then found themselves at the top of a considerable eminence, where the wood ceased, and the eye was instantly caught by Pemberley House. B. It was a large, stone, handsome building, standing well on rising ground, and backed by a ridge of high woody hills. C. They descended the hill, cross a bridge across a stream, and drove to the door. D. It was situated on the opposite side of

Verbal Ability a valley, into which the road with some abruptness winded. E. They entered the park at one of its lowest points, and drove for some time through a beautiful wood stretching over a wide extent. 1) ABDCE 2) ABCED 3) BDEAC 4) EADBC 5) ECBDA 293.

D.

E.

A. Our mental development is more like a metamorphosis than an incremental process of growth, so we butterflies can boast a precious little understanding of the caterpillars in our strollers. B. Everyone was a baby once, and most adults have spend plenty of time talking to small children. C. To see what is really happening in their heads, we need grown-up science, in the form of cunningly designed and rigorously executed experiments. D. Early childhood is both familiar and mysterious. E. But we simply can't remember what it was like to be younger than 5 or 6, and conversation between an adult and a preschool child is far from a dialogue between equals. 1) BEACD 2) BACDE 3) DBAEC 4) DBEAC 5) DAEBC

1) 3) 5)

DIRECTIONS for questions 294 to 296: In each question, there are five sentences/paragraphs. The sentence/paragraph labelled A is in its correct place. The four that follow are labelled B, C, D and E, and need to be arranged in the logical order to form a coherent paragraph/ passage. From the given options, choose the most appropriate option. 294.

A. Publishers of all types, from news to music, are unhappy that consumers won't pay for content anymore. At least, that's how they see it. B. They treat the words printed in the book the same way a textile manufacturer treats the patterns printed on its fabrics. C. If the content was what they were selling, why has the price of books or music or

58

movies always depended mostly on the format? Why didn't better content cost more? In fact consumers never really were paying for content, and publishers weren't really selling it either. Almost every form of publishing has been organized as if the medium was what they were selling, and the content was irrelevant. Book publishers, for example, set prices based on the cost of producing and distributing books. BDCE 2) BCED DBCE 4) DCEB CDBE

295.

A. Mary stepped out of the grass onto one of those rivers of stone she'd seen from the hill. B. When she came to the first trunk, she rested her hands on the deeply ridged red-gold bark. C. It might once have been some kind of lava-flow. It was as smooth as a stretch of well-laid road in Mary's own world, and certainly easier to walk on than the grass. D. The closer she got, the more astounded she was by the enormous size of the trunks, as wide, she estimated, as the house she lived in, and as tall, as tall as ... she couldn't even make a guess. E. She followed the one she was on, which flowed in a wide curve towards the trees. 1) CEBD 2) CEDB 3) DBEC 4) ECDB 5) ECBD

296.

A. Periodicals publish corrections in subsequent issues and some successful books are (expensively) reissued in new, improved editions. B. Old-fashioned, uncorrectable books may never disappear. C. Thanks to the advent of electronic reading gadgets, like Amazon's Kindle and Sony's Reader, such magic is getting closer. D. Yet it is now possible that the tyranny of print will meet some powerful resistance, and that readers will benefit.

Verbal Ability of raw vegetables and fresh fruits to cleanse his system.

E.

But in a better world, the book, magazine or newspaper in your hands would itself be updated when mistakes are discovered by its publisher. 1) BDEC 2) CBDE 3) CEDB 4) ECDB 5) ECBD

Martial governments prefer to proscribe(A)/ prescribe(B) debate rather than welcoming it. 1) ABBA 3) BBBA 299.

Directions for question 297: In each of the question there are four sentences. In each sentence a pair of words is italicized and highlighted0. From the italicized and highlighted words select the most appropriate word (A or B) to form correct sentences. The sentences are followed by options that indicate the words, which may be selected to correctly complete the set of sentences. From the options given choose the most appropriate one. 297.

It is credible(A)/creditable(B) that he has been able to make a mark in such a short span of time. Threat of military action has always served as a credible(A)/creditable(B) deterrent. The third quarter results were credible(A)/ creditable(B) enough for the company’s stock to rise. 2) ABAA 4) BBBB

DIRECTIONS for question 300: The sentences given in each question, when properly sequenced form a coherent paragraph. Each sentence if labeled with a letter. Choose the most logical order of sentences from among the given choices to construct a coherent paragraph.

Directions for question 298 : In each of the question there are four sentences. In each sentence a pair of words is italicized and highlighted. From the italicized and highlighted words select the most appropriate the word (A or B) to form correct sentences. The sentences are followed by options that indicate the words, which may be selected to correctly compete the set of sentences. From the options given choose the most appropriate one. 298.

Atlases were once placid, unhurried publications with additional information on the colours of national flags. Now atlases are freighted with maps showing cities that are likely to be submerged if the Arctic melts, or projected population growth, or the relative size of countries in terms of CO2 emissions, or areas where water scarcity will be most intense and resource wars most likely to break out. 1) Very soon climate change will be so rapid that an atlas will have a super-short shelf life before it is updated to incorporate the latest changes. 2) An atlas is no longer static; it is dynamic and allows users to interact with it through multimedia-rich tools. 3) An atlas is beginning to look like a longterm forecast-history before it happens. 4) It would be easy to imagine a parallel world in which the BBC News opened with the weather forecast.

There is credible(A)/creditable(B) evidence of the government’s involvement the scam.

1) ABBA 3) ABAB

2) AABA 4) BABB

300.

No society can legally proscribe(A)/ prescribe(B) the profanation of the sacred and call itself free. The doctor proscribed(A)/prescribed(B) consumption of any alcohol since it was detrimental to his liver. He was proscribed(A)/prescribed(B) a diet

59

A. And it seems likely that over the past few years the distinctions have eroded further, especially in the tropics. B. Most Europeans and North Americans ex perience the seasonal beat as a four-four march, spring, summer, autumn and win ter. C. The rhythm of the seasons is fundamen tal to mankind. D. But the seasons themselves have always been more variegated, a little less fixed in their categories than they seem-and

Verbal Ability that is especially true in the tropics, where distinctions between them are less sharp than in more temperate climes. 1) CBAD 2) CBDA 3) BCDA 4) BDAC

303.

Directions for Question 301: Each of the following sets of questions has a sentence with two blanks. Given below each question are four pairs of words. Choose the pair that best completes the sentence. 301.

1) 2) 3) 4)

Some will call it a hot trend, others a pipe dream, but the of letting diners choose what they pay for their meals over the last has been gaining decade. 1) 2) 3) 4)

cone; clique pyramid; niche system; place concept; territory

304. A. Welcome to the politics of the libertarian mob. B. This is the one threat that will bring Ameri cans into the streets. C. Anarchistic like the Sixties, selfish like the Eighties, contradicting neither, it is es tranged, aimless and as juvenile as our new century. D. A new strain of populism is metastasizing before our eyes. E. It appeals to petulant individuals con vinced that they can do everything them selves if they are only left alone, and that others are conspiring to keep them from doing just that. 1) ABCED 2) DABCE 3) DCEBA 4) DCEAB

notion; traction misconception; momentum strategy; incubation promotion; resonance

Directions for Question 302: The sentences given in each question, when properly sequenced form a coherent paragraph. Each sentence is labeled with a letter. Choose the most logical order of sentences from among the given choices to construct a coherent paragraph. 302.

The only way for craftsmen to survive in the age of mass-production is by imagining an alternate model; a , with expensive articles made by master craftsmen occupying a small at the top and everyday articles produced by younger apprentices at the bottom.

A. At a stroke, the sale made clear that the rich are back in their rightful place at the apex of the world. B. Not only does it signal that the plutocracy believes it successfully eluded financial Armageddon. C. Aesthetic choices are social markers with which the powerful signal their power and set themselves apart from other, inferior groups. D. That’s why the record set by Giacometti’s ‘Walking Man I’ is so significant. 1) ADBC 2) DABC 3) CDBA 4) CDAB

305. A. B. C. D.

E.

DIRECTIONS for Questions 303 to 305: Each of the following sets of questions has a sentence with two blanks. Given below each question are four pairs of words. Choose the pair that best completes the sentence.

F.

60

Writing and print solve problems of dissemi nation in time and space. One cost of storytelling is time. A visual form of narrative would solve this by telling stories in non-serial, sensorily-rich, and durable forms. Language allows only the serial transmission of information along a single channel; though it can compress events in time, it still takes time to tell them. But language, especially in written form, suffers from a lack of the sensory immediacy our minds have mostly evolved for. But though they take time, orally transmitted narratives, the only option for most of the history of story, do not last. 1) DECFBA 2) BDFAEC 3) ABCDFE 4) BDAECF

Verbal Ability sessing this art, but who never attained a phonetic alphabet and the use of writing will be classed as barbarians. 1) EDACB 2) BCEDA 3) BDCEA 4) BEDAC

Directions for question 306: Each of the following questions has a sentence with two blanks. Given below each question are four pairs of words. Choose the pair that best completes the sentence. 306.

No trait is good or bad in an sense; each with other personality traits, expressing itself differently depending on context. 1) 2) 3) 4)

Directions for Questions 308 and 309: Each of the following sets of questions has a sentence with two blanks. Given below each question are four pairs of words. Choose the pair that best completes the sentence.

ethical; vies unambiguous; develops absolute; jostles errant; blends

308.

Directions for Question 307: The sentences given in each question, when properly sequenced form a coherent paragraph. Each sentence is labelled with a letter. Choose the most logical order of the sentences from among the given choices to construct a coherent paragraph.

1) 2) 3) 4) 309.

307. A.

B.

C.

D. E.

From an arrangement meant to __________ the rearing of children, traditional marriage has come to be seen as a private relationship centered on the________ of adults for love and companionship.

In finding its termination, and the commence ment of the Middle Status, a difficulty is encountered in the unequal endowments of the two hemispheres, which began to be influential upon human affairs after the pe riod of savagery, had passed. The invention or practice of the art of pottery, all things considered, is probably the most effective and conclusive test that can be selected to fix a boundary line, necessarily arbitrary, between savagery and barbarism. It may be met, however, by the adoption of equivalents; in the Eastern hemisphere, the domestication of animals, and the Western, the cultivation of maize and plants by irri gation, together with the use of adobe-brick and stone in house building have been se lected as sufficient evidence of progress to work a transition out of the Lower and into the Middle Status of barbarism. The first sub-period of barbarism commenced with the manufacture of pottery, whether by original invention or adoption. The distinctness of the two conditions has long been recognized, but no criterion of progress out of the former into the latter has hitherto been brought forward; all such tribes, then, as never attained to the art of pottery will be classed as savages, and those pos

cherish; commitments nurture; demands foster; needs sustain; exigency

For months now, a bitter battle has been taking shape in Great Britain between defenders of homeopathy and __________ who point to the lack of scientific evidence that the remedies offer anything more than a ___________ effect. 1) 2) 3) 4)

opponents; recuperative detractors; placebo adherents; medicinal experts; therapeutic

Directions for question 310: Each of the following questions has a paragraph from which the last sentence has been deleted. From the given options, choose the option that completes the paragraph in the most appropriate way. 310.

Unhappiness and materialism are mutually reinforcing mechanisms. And we should reflect: isn't the effect of advertising to deliberately manufacture feelings of inadequacy, fragile self-worth, and so forth? Thus, it is not the case that all of this advertising and promotion of commerce does more than just capitalize on unhappiness. 1) It is geared towards creating unhappiness, because that's what generates a desire to accumulate material things. 2) It is geared towards utilizing the whole

61

Verbal Ability array of human emotions - ranging from love to fear to hatred and capitalizing on it. 3) It is based upon lowering of man's selfesteem, often artificially and then provid ing solutions to it. 4) It is based upon exploiting unhappiness inherent in all human beings and profiting from it.

Directions for Question 312: Each of the following sets of questions has a sentence with two blanks. Given below each question are four pairs of words. Choose the pair that best completes the sentence. 312.

Directions for question 311: The sentences given in each question, when properly sequenced form a coherent paragraph. Each sentence is labelled with a letter. Choose the most logical order of the sentences from among the given choices to construct a coherent paragraph. 311.

The very notion that one can sustain an enduring mental disorder based on anxious ___________ of a traumatic event that never materializes is a radical _________ from the clinical-and common sense-understanding that disabling stress disorders are caused by traumatic events that actually do happen to people. 1) 2) 3) 4)

A. On the other hand, he viewed the much older English iambic tetrameters as muddled and poorly documented. B. That commentary ended with an appen dix titled Notes on Prosody, which has developed a reputation of its own. C. It stemmed from his observation that while Pushkin's iambic tetrameters had been a part of Russian literature for a fairly short period of two centuries, they were clearly understood by the Russian prosodists. D. Nabokov's stature as a literary critic is founded largely on his four-volume translation of and commentary on Aleksandr Pushkin's epic of the Russian soul, Eugene Onegin, published in 1964. 1) DCBA 2) DBCA 3) DCAB 4) DBAC

62

anticipation; departure symptoms; revelation portrayal; escape premonition; aberration

Verbal Ability Verbal Ability ªScore Maximiserª DIRECTIONS for questions 316 and 317: In each of the questions, a word has been used in sentences in five different ways. Choose the option corresponding to the sentence in which the usage of the word is incorrect or inappropriate.

DIRECTIONS for questions 313 to 315: In each of the following questions there are sentences that form a paragraph. Identify the sentence(s) or part(s) of sentence(s) that is/are correct in terms of grammar and usage (including spelling, punctuation and logical consistency). Then, choose the most appropriate option.

316.

Prime 1) Johnson's prime attitude is to neglect his work and while away crucial time. 2) In his prime, Sachin Tendulkar was a batsman feared for his skills the world over. 3) The builder had been eyeing some prime commercial lots in the city. 4) 2 is the only even prime number. 5) It is important to prime the cabinet ministers as to the technical aspects of their respective jobs before commencing work.

313. A. Often-cited studies have shown that the difference in IQ B. among identical twins raised apart is only slightly less C. than twins raised together, whereas the correlation D. between the intelligence scores of a parent E. who adopts a child and those of that child are slim. 1) A & B 2) A & D 3) B & C 4) C & D 5) D & E 317.

314. A. Scientists often look for Fermi estimates of the answer to a problem B. before turning to more sophisticated methods to calculate a precise answer, C. This provides a useful check on the results: D. where the complexity of a precise calculation might obscured a large error, E. the simplicity of Fermi calculations makes them far less susceptible of such mistakes. 1) A & C 2) A, B & C 3) C & D 4) D & E 5) B, D & E

Flags 1) The garden path was paved with flags. 2) He can identify the flags of more than a hundred different countries. 3) After a scandal like this, people's confidence in their elected leaders flags. 4) The film begins with a scene in which a teacher flags a disobedient pupil with a cane. 5) This word-processing software flags incorrect spellings of words in a document.

315. A. The difference is that then everyone joined in B. the music making, as oppose to what we do today, C. sit quietly in passive listening mode. D. But to claim that we have more music now E. is not consistent to the evidence. 1) A & C 2) A & D 3) B & E 4) C & D 5) C & E

DIRECTIONS for questions 318 to 320: Each of the following questions has a sentence with two blanks. Given below each question are five pairs of words. Choose the pair that best completes the sentence. 318.

Attacking popular culture, which is the _______ of so much of our conventional wisdom, usually earns one the _______ of Puritan or crank. 1) 2) 3) 4) 5)

63

underlying … appellation underpinning … sobriquet understanding … moniker underplaying … label undermining … status

Verbal Ability 319.

The difference between Johnson and an ordinary _______ is that Johnson seems to have had the bad luck to start believing his own _______. 1) 2) 3) 4) 5)

320.

A. A Google search, once you have keyed the words in, take a broadband user less than a second, and the process will only get quicker. B. And soon pretty much everybody, from schoolchildren to drinkers in pubs, will be online pretty much all of the time. C. If you want to know who wrote 'Skellig', or whether Norway is a member of the European Union, or what Cary Grant's real name was, you just ask your laptop or your phone. D. As for those laborious keystrokes, voicerecognition technology will enable us for by passing them. E. In that context, perhaps there isn't any point no longer in keeping facts in our heads. 1) A & B 2) B & C 3) C & D 4) D & E 5) A & E

323.

A. They gradually ascended for half-a-mile, and then found themselves at the top of a considerable eminence, where the wood ceased, and the eye was instantly caught by Pemberley House. B. It was a large, stone, handsome building, standing well on rising ground, and backed by a ridge of high woody hills. C. They descended the hill, cross a bridge across a stream, and drove to the door. D. It was situated on the opposite side of a valley, into which the road with some abruptness winded. E. They entered the park at one of its lowest points, and drove for some time through a beautiful wood stretching over a wide extent. 1) A & E 2) A, B & E 3) A C & D 4) B & D 5) C, D & E

guru … assessments pundit … panacea charlatan … nostrums quack … proficiency mountebank … perspicacity

It is rarely _____ which controls the sorry fate of humankind; rather its own intuitive instincts are the _____ of humankind's evil ways. 1) 2) 3) 4) 5)

law ... revelation divinity ... aggrandizement providence ... manifestation sin ... externalization God ... substantiation

DIRECTIONS for question 321: In each of the following sentences, a part of the sentence is highlighted. Beneath each sentence, five different ways of phrasing the underlined part are indicated. Choose the best alternative from among the five. 321.

322.

To the thinkers of the Enlightenment, it seemed little more than a superstition to believe that artefacts or ceremonies could possess a sacred character, when these things were the products of human design. 1) it seemed little more than a superstition to believe that artefacts or ceremonies could possess a sacred character, 2) it seemed little more than superstition to believe that artefacts or ceremonies could possessed a sacred character, 3) it seemed a little more than superstition to believe that artefacts or ceremonies could possess a sacred character, 4) it seemed a little more than superstition to believe that artefacts or ceremonies could possessed a sacred character, 5) it seemed a little more than a superstition to believe that artefacts or ceremonies could possessed a sacred character,

Directions for questions 322 to 323: Each question consists of five sentences on a topic. Some sentences are grammatically incorrect or inappropriate. Select the option that indicates the grammatically correct and appropriate sentence(s).

DIRECTIONS for question 324: For each statement, there are four different sentences given below it. Pick out the one that appropriately conveys the meaning of the statement in a grammatically correct manner. If none of them convey the meaning of the statement, the answer is [5), i.e., 'None of these'.

64

Verbal Ability 324.

326.

Mary was obliged to mix more with the world; but as she no longer had to endure comparisons between her sisters' beauty and her own, it was suspected by her father that she submitted to the change without much reluctance.

1) 'It was only by God's grace that I escaped that terrible sandstorm,' said Fred. 2) It would have been impossible for me to stay in their good grace for so long if I had not proved my loyalty to the revolution. 3) She walked with a grace that transcended all imagination and earthly notions of beauty. 4) I have fired him because he does not have the grace to perform his assigned tasks efficiently. 5) The chief guest refused to grace with her presence the occasion of the inauguration of the new library as she had developed some serious differences with the management.

1) Mary was forced to become more sociable; but as her beauty was no longer compared to her sisters', her father felt that she was not against the change. 2) Mary was forced to become more sociable; but as her looks were no longer compared unfavourably to her sisters', her father suspected that she did not mind the change. 3) Mary was forced to participate in social occasions; but as she was no longer considered less beautiful than her sisters, he father thought that she did not mind the change. 4) Mary was forced to participate in social occasions; but as there were no more unfavourable comparisons between her looks and her sisters', she did not mind the change.

DIRECTIONS for question 327: Each of the following questions has a paragraph with one italicized word that does not make sense. Choose the most appropriate replacement for that word from the options given below the paragraph.

DIRECTIONS for question 325 and 326: In each of the questions, a word has been used in sentences in five different ways. Choose the option corresponding to the sentence in which the usage of the word is incorrect or inappropriate. 325.

Grace

327.

Farce

The classical definition of a habitable environment is one that has the presence of a solvent (for example water), availability of the raw materials for life, virfny weather conditions and some kind of energy source. 1) roily 3) clement 5) benevolent

1) Being a comedian with some oratorical skills to her credit, she often used to farce her speeches with subtle wit and humour. 2) The play was a farce, with exaggerated humour, and was good for simple, light entertainment. 3) The meeting between the management and the employees' union was a farce as it ended in a shouting match and no concrete decisions were taken. 4) The recent rigged elections in Iran were a farce, resulting in a comprehensive victory for the ruling party. 5) It is indeed a sad farce that in spite of having worked so hard, the product team could not achieve its targets.

2) facile 4) turbulent

DIRECTIONS for question 328: Read the short passages given below and answer the question that follows each. 328.

65

In the last few years, scores of applications that your operating system used to manage have migrated to the browser: word processors, IM clients, e-mail, games, music players, personal finance tools, and on and on. Which leads inevitably to the question: If the primary function of computers these days is to run a browser and connect to the Internet, do we really need Windows? The browseras-operating-system idea is certainly attractive, especially as an alternative to Windows'

Verbal Ability 3) For example, when you ask yourself, 'Do I hate my job because I'm awful at it, or am I awful at my job because I hate it?' you're being Socratic in your search. 4) Such methods of learning were rare in the ancient world, and consequently Socrates was hailed as a genius by his students. 5) If this did not work out, he reverted back to the lecturing method.

bloat and sluggish performance. Firefox, with its open platform and sprawling library of free add-ons, seems in theory like a much better model for how your PC's core software should work. It looks like the day when we relocate personal computing to the browser may not be very far off. Which of the following, if true, would not weaken the author's argument substantially? 1) Browsers themselves need an operating system to function. 2) Not all operating system-based applications have migrated to the browser as yet. 3) A browser that is made to perform all the functions of an operating system would inevitably become bloated as well. 4) Most people would not like to store their private information in a browser, as there would be numerous security concerns. 5) The idea can work only with a cheap and uninterrupted broadband Internet connection, which may be available to only a tiny percentage of computer users.

330.

1) Today, 70 percent of India's irrigation needs and 80 percent of its domestic water supplies come from groundwater, a commendable fact nevertheless. 2) Faced with poor water supply services, farmers and urban dwellers alike have resorted to helping themselves by pumping out groundwater through tube wells. 3) A number of areas are already in crisis regarding housing infrastructure: among these are the most populated and economically productive parts of the country. 4) Severe water shortages have already led to growing political mudslinging in Parliament. 5) Estimates reveal that by 2020, India's demand for water will exceed all sources of supply.

DIRECTIONS for questions 329 to 332: Each of the following questions has a paragraph from which the last sentence has been deleted. From the given options, choose the sentence that completes the paragraph in the most appropriate way. 329.

India's past investments in large water infrastructure have yielded spectacular results with enormous gains in food security and in the reduction of poverty. However, much of this infrastructure is now crumbling. Shortfalls in financing have led to an enormous backlog of maintenance. The implicit philosophy has been aptly described as Build-Neglect-Rebuild. Much of what currently masquerades as 'investment' in irrigation or municipal water supply is in fact a belated attempt to rehabilitate crumbling infrastructure.

Like many a good philosopher, Socrates (470399 BCE) was obsessed with truth and the correct way to stumble into it. In fact, in his effort to find truth, Socrates placed value not just on knowledge, but also on how we know knowledge, and his inquisitive teaching style reflected it. For one thing, Socrates never lectured. Instead, he asked questions on top of questions (a teaching method still used to this day). The more his students answered, the more they knew or, more accurately, learned what they didn't know.

331.

1) As a master philosopher, Socrates' great est rhetorical tool was irony. 2) Unfortunately for Socrates, endless questioning was also extremely annoying, and the barefoot philosopher's inquisitiveness made him powerful enemies.

66

Adult enthusiasm for video games is part of a broader transformation - a blurring of childhood and adulthood. Children and adults now dress more alike, and they have begun to behave more alike. It isn't unusual to see adults engaged in 'children's play'. The latest generation of playthings - video and computer games - are avidly played by both adults and children. Whether this is a good thing or not,

Verbal Ability DIRECTIONS for questions 333 and 334: In each question, there are five sentences/paragraphs. The sentence/paragraph labelled A is in its correct place. The four that follow are labelled B, C, D and E, and need to be arranged in the logical order to form a coherent paragraph/ passage. From the given options, choose the most appropriate option.

is difficult to say. 1) Going by the general reaction to this trend, though, most social critics have planted their flags firmly in the 'not' category. 2) However, this is true only of Western cultures - in other parts of the world, childhood and adulthood are still clearly distinct. 3) Not that it matters - the transformation is so well underway, it is unlikely to stop now. 4) For better or worse, though, childhood and adulthood, as they were once defined, no longer exist. 5) This transformation isn't limited to only video games, but spills over into almost all aspects of popular culture. 332.

333.

There is no known culture, now or in the past, that lacks music. Indeed, some of the oldest human-made artefacts found at archaeological sites are musical instruments. Could our fascination with music merely be (as popular scientist and language theorist Dr Steven Pinker argues) an accidental byproduct of our brain's fascination with sound and pattern? Or has music played a crucial role in the evolution of the human mind? Music theories have been propounded since ancient Greek times. 1) Those ancient theories, however, have little or no value in today's high tech world. 2) But these are fairly crude compared to the modern scientific theories put forth by Dr Pinker and his colleagues. 3) But what those ancient cultures called music would not be recognizable as such today. 4) But in recent years, with advances in neurology, psychology and anthropology, we seem to be moving closer to an understanding of music's evolution. 5) If a culture that lacked music were to be found, it would turn our views on the evolution of music upside down.

67

A. Today complex numbers have such wide spread practical use - from electrical engineering to aeronautics - that few people would expect the story behind their derivation to be filled with adventure and enigma. But such is the 2000-year-old history of one of mathematics' most elusive numbers, the square root of minus one, also known as i. B. By the time of Descartes, a theoretical use for these elusive square roots - now called 'imaginary numbers' - was suspected, but efforts to solve them led to intense, bitter debates. C. In the first century, the mathematicianengineer Heron of Alexandria encountered i in a separate project, but fudged the arithmetic; medieval mathematicians stumbled upon the concept while grappling with the meaning of negative numbers, but dismissed their square roots as nonsense. D. In 1878, when two brothers stole a mathematical papyrus from the ancient Egyptian burial site in the Valley of Kings, they led scholars to the earliest known occurrence of the square root of a negative number. The papyrus offered a specific numerical example of how to calculate the volume of a truncated square pyramid, which implied the need for i. E. The notorious i finally won acceptance and was put to use in complex analysis and theoretical physics in Napoleonic times. 1) CBDE 2) CBED 3) CDBE 4) DBEC 5) DCBE

Verbal Ability 334.

Cyclones will weaken rapidly when they travel over land or colder ocean waters - locations where their heat and/or moisture sources do not exist. Related to having warm ocean water, high relative humidities in the lower and middle troposphere are also required for cyclone development. These high humidities reduce the amount of evaporation in clouds and maximize the latent heat released because there is more precipitation. The vertical wind shear in a tropical cyclone's environment is also important.

A. The 'uncanny valley' hypothesis, intro duced by Japanese roboticist Masahiro Mori in 1970, holds that when robots and other facsimiles of humans look and act almost like actual humans, it causes a response of revulsion among human observers. B. The name captures the idea that a robot which is 'almost human' will seem overly 'strange' to a human being and thus will fail to evoke the empathetic response required for productive human-robot interaction. C. Mori's hypothesis states that as a robot is made more humanlike in its appearance and motion, the emotional response from a human being to the robot will become increasingly positive and empathic, until a point is reached beyond which the response quickly becomes that of strong repulsion. D. This area of repulsive response aroused by a robot with appearance and motion between a 'barely human' and 'fully human' entity is called the uncanny valley. E. However, as the appearance and motion continue to become less distinguishable from a human being, the emotional response becomes positive once more and approaches human-to-human empathy levels. 1) BDEC 2) CBDE 3) CEDB 4) DBCE 5) DEBC

1) When the wind shear is weak, the storms that are part of the cyclone grow vertically, and the latent heat from condensation is released into the air directly above the storm. 2) Wind shear is the amount of change in the wind's direction or speed with increasing altitude. 3) When there is stronger wind shear, this means that the storms become more slanted and the latent heat release is dispersed over a much larger area. 4) Cyclones evolve through a life cycle of stages from birth to death. 5) Cyclones can often live for a long period of time - as much as two to three weeks. Directions for question 336: The following six questions are divided into 3 pairs. For each pair, refer to the set of sentences given only in the first question of the pair. 336.

Directions for question 335: Each of the following questions has a paragraph from which the last sentence has been deleted. From the given options, choose the sentence that completes the paragraph in the most appropriate way. 335.

Cyclones are formed from simple thunderstorms. However, these thunderstorms can only grow to cyclone strength with cooperation from both the ocean and the atmosphere. First of all, the ocean water itself must be warmer than 26.5 degrees Celsius (81°F). The heat and moisture from this warm water is ultimately the source of energy for cyclones.

68

A. A Google search, once you have keyed the words in, take a broadband user less than a second, and the process will only get quicker. B. And soon pretty much everybody, from schoolchildren to drinkers in pubs, will be online pretty much all of the time. C. If you want to know who wrote 'Skellig', or whether Norway is a member of the European Union, or what Cary Grant's real name was, you just ask your laptop or your phone. D. As for those laborious keystrokes, voicerecognition technology will enable us for bypassing them. E. In that context, perhaps there isn't any

Verbal Ability DIRECTIONS for questions 340 and 341: In each of the questions a word has been used in sentences in four different ways. Choose the option corresponding to the sentence in which the usage of the word is incorrect or inappropriate.

point no longer in keeping facts in our heads. 1) ABCDE 2) ADBEC 3) ACEDB 4) CABDE 5) CEADB

340.

Directions for questions 337 and 338: In each of the questions, a word has been used in sentences in four different ways. Choose the option corresponding to the sentence in which the usage of the word is incorrect or inappropriate. 337.

1) The police were forced to rule out murder, and eventually decided it was a suicide. 2) The high court decided to rule over the earlier decision. 3) During his rule, King Daksha built many palaces and pleasure halls. 4) I don't eat very much, as a rule.

Rope 1) I have given him enough rope already; it’s time to let him go. 2) Try to rope her into this project. 3) I am still trying to learn the rope. 4) I am at the end of my rope with the kids.

338.

341.

Run

DIRECTIONS for questions 342 and 343: In each of the following questions there are sentences that form a paragraph. Identify sentences(s) or part(s) of sentences(s) that is/are correct in terms of grammar and usage (including spelling, punctuation and logical consistency). Then, choose the most appropriate option.

A complacent (A)/complaisant (B) attitude has no place in modern corporations with flat hierarchies. His complacent (A)/complaisant (B) behavior led to him underestimating the extent to which the elections would impact the stock market.

342.

A dominant state fosters a culture that is complacent (A)/complaisant (B) as opposed to one that is open to debate. Favoured by those in power for his complacent (A)/complaisant (B) attitude, he continued to rise over others who were more qualified. 1) BABB 3) AABA

Cut 1) He wasn't a very serious student; it is no surprise that he failed to make the cut. 2) If we make him a partner, we'll have to give him a cut of the profits. 3) He was cut off in the prime of his life by the smallpox. 4) I believe Mr. Khan is not really cut up to be president.

1) One has to run a risk to strike it big on the stock market. 2) I had warned him not to run afoul of the government. 3) He has hinted that he would run out on the family. 4) The movie was a run away success thanks to its outstanding opening. 339.

Rule

2) AABB 4) ABAA

69

A. For the last two years, the Shanghai Commission for the Management of Lan guage Use B. has tried to clean up English-language signs and menus to rid them of their malapropisms. C. The commision has fixed more than 10,000 public signs, D. rewritten English-language placards E. and helped hundreds of restaurants recast offerings. 1) A, B, C and E 2) A, C, D and E 3) A, B, and E 4) A, D and E

Verbal Ability 343.

4) It was tough to register even the slightest of expressions on his face.

While there is no doubt that people can be allergic to certain foods, B. with reproducible responses ranging from a rash C. to a severe life-threatening reaction D. the true incidents of food allergies is only E. about 8 percent for children and lesser than 5 percent for adults. 1) B and C 2) C and D 3) B, C and D 4) D and E

346.

1) The stadium has state-of-the-art facility. 2) He negotiated even the toughest bends with great facility. 3) All web pages have a Google-powered search facility. 4) The institute provides the facility to apply to multiple courses.

Directions for Question 344: In each of the question there are four sentences. In each sentence a pair of words is italicized and highlighted. From the italicized and highlighted words select the most appropriate the word (A or B) to form correct sentences. The sentences are followed by options that indicate the words, which may be selected to correctly compete the set of sentences. From the options given choose the most appropriate one. 344.

Directions for Questions 347 and 348: In each of the following questions there are sentences that form a paragraph. Identify the sentences(s) or part(s) of sentences(s) that is/are correct in terms of grammar and usage (including spelling, punctuation and logical consistency). Then, choose the most appropriate option.

The business completely floundered(A)/ foundered(B) after the death of the patriarch.

347.

A. A comprehensive study by a leading corporation showed that B. the longer the vacation their employees took, C. the better their performance. D. The longer and more continuosly people work, E. the lesser the marginal return they get from each additional hour. 1) A B and C 2) A, B and E 3) A, C and D 4) A, B, C and E

348.

A. If a machine can tell me what I am about to decide B. before I decided it, this means that, in some sense, C. the decision has already been made before I became consciously involved. D. But if that is the case, how am I, E. as a moral agent, to be held accountable for my actions? 1) A B and E 2) A, B and D 3) A, C and D 4) A, D and E

Lacking directions they floundered(A)/ foundered(B) about town trying to find their destination. He floundered(A)/foundered(B) helplessly on the first day of his new job. Lacking public support the floundered(A)/foundered(B). 1) ABAB 3) ABBB

project

2) BAAB 4) ABBA

Directions for questions 345 and 346: In each of the questions, a word has been used in sentences in four different ways. Choose the option corresponding to the sentence in which the usage of the word is incorrect or inappropriate. 345.

Facility

Register 1) The gatekeeper maintains a register of outsiders who enter the building. 2) He was not afraid of speaking in an in formal register with his superiors. 3) Earthquakes that register less than a 3 on the Richter scale are imperceptible.

70

Verbal Ability Directions for questions 349 to 351: In each of the following questions there are sentences that form a paragraph. Identify the sentences(s) or part(s) of sentences(s) that is/are correct in terms of grammar and usage (including spelling, punctuation and logical consistency). Then, choose the most appropriate option. 349.

350.

351.

Directions for questions 352 and 353: In each of the question there are four sentences. In each sentence a pair of words is italicized and highlighted. From the italicized and highlighted words select the most appropriate word (A or B) to form correct sentences. The sentences are followed by options that indicate the words, which may be selected to correctly complete the set of sentences. From the options given choose the most appropriate one.

A. For millennia, there has been two avail able answers. B. To the devoutly religious morality is the word of God, C. handed down to holy men in groves or mountaintops. D. To moral philosophers like Kant, it is a set of rules to be worked out E. by reason, chin on fist like Rodin’s thinker. 1) A, B and D 2) B, C and E 3) B and D 4) D and E

352. He is loath(A)/loathe(B) to admit his mistakes. The job had many perquisites(A)/ prerequisites(B) which made him accept the offer. He had an unexceptional(A)/ unexceptionable(B) track record making him the ideal candidate. He did not want to judge the case, as he was totally disinterested/uninterested in the issues involved.

A. Let’s say that a species migrates out of Africa into Europe B. around 400,000 years ago and become reproductively isolated C. from its ancestral population for next 320,000 years. D. It evolves distinctive anatomical features and E. adaptations for the colder climes. 1) A, D and E 2) B and D 3) A and C 4) B, C and E

1) BABA 3) BABB

2) AABA 4) AAAA

353. The plan contemplates placing the troops under canvas (A)/ canvass (B) outside the city. You will have to use more discreet(A)/ discrete(B) methods if you wish to not attract attention to yourself.

A. William Jacques, the Cambridge-edu cated serial book thief B. who has just been sent to the prison for stealing books from the Royal Horticultural Society C. probably thought that he was showing his conoisseurship D. and even his erudition by his activities. E. After all it takes knowledge and discrimination to know which books to steal from library shelves. 1) C and E 2) A and D 3) B and E 4) All are correct

The crops were affected(A)/effected(B) by the incessant rains. He was ambiguous(A)/ambivalent(B) about science; always careful to highlight its limitations while praising its achievements. 1) BAAB 3) BABA

71

2) ABBA 4) AAAB

Verbal Ability Directions for Questions 354 and 355: In each of the following questions, there are sentences that form a paragraph. Identify the sentences(s) or part(s) of sentences(s) that is/are correct in terms of grammar and usage (including spelling, punctuation and logical consistency). Then, choose the most appropriate option.

357.

1) It had been a long day and James decided to turn in for the night. 2) Seeing that the battle was lost, the tyrant’s own guards decided to turn on him. 3) The new dictator of the land whipped the prisoners and made them turn the statue of their earlier leader and carry it out of town. 4) One should never turn on a blast furnace without proper precautions; it could explode.

354. A. B. C. D. E.

About 2,000 years earlier, Aristotle had argued that heavy objects should fall more rapidly than lighter objects. Aristotle is universally recognized as one of the greatest genius in recorded history, and he backed up his argument with seeming airtight reasoning. 1) A and D 2) B and E 3) B, C and D 4) A, B and E

Directions for questions 358 and 359: In each of the following questions there are sentences that form a paragraph. Identify the sentences(s) or part(s) of sentences(s) that is/are correct in terms of grammar and usage (including spelling, punctuation and logical consistency). Then, choose the most appropriate option.

355. A. B. C. D. E.

Turn

358. A.

A woman in the early 1800s had fewer legal rights than a freed male slave. The law considered married couple a single legal entity, and that entity was de facto male. So wholly did the law considered man and wife united that spouses were not allowed to testify against each other. 1) A, B and E 2) B, C and D 3) B, D and E 4) A, C and E

B. C. D.

E.

Directions for Questions 356 and 357: In each of the following questions, a word has been used in sentences in four different ways. Choose the option corresponding to the sentence in which the usage of the word is incorrect or inappropriate.

359. A.

356.

B.

Hung 1) The convict was hung a week after being sentenced to death. 2) She hung up the phone in irritation. 3) The walls were hung with decorations for the festival. 4) Last weekend we hung out at the mall, let’s do something different this time!

C. D. E.

72

Back in university, in love with my new found radicalism, as students tend to be, reading things was started. Not the stuff I was supposed to be reading about Lollards and John Wycliffe and pre-reformation Europe, but green po litical thought: wild ideas I had never come across before. I could literally feel my mind levering itself open. 1) A, B and D 2) A and E 3) C, D and E 4) B and C

China’s commitment to foreign trade and in vestment have generated an astounding three decades of double digit growth, a trend too impressive enough to inspire some to believe that it just might last forever. Western firms, large and small, now banking on the promise of steady long term profits as China develops what is expected to be come the largest middle class in history. 1) A, D and E 2) B and D 3) A and C 4) B and E

Verbal Ability Directions for questions 360 and 361: In each of the questions a word has been used in sentences in four different ways. Choose the option corresponding to the sentence in which the usage of the word is incorrect or inappropriate. 360.

3) But it is also true that through the abuse of the language arise, sometimes, new meanings, associations and ways of thinking which would otherwise have been impossible. 4) Language is always linked to culture and hence the decline of a language reflects the decline of a culture.

Stand 1) It's high time you stand up to your parents. 2) Could you please stand in for me at the rehearsal? 3) I can't stand for their duplicity. 4) What is your stand about the new tax regime?

361.

363.

Make 1) His new book will not make for com fortable reading. 2) I need to make up the place. 3) You have to make do with this. 4) You will have to make up to her for your forgetfulness.

1) This was bound to happen since global ization tends to extend competition between countries into all spheres. 2) This competition can be unlikely trigger for a major confrontation between Europe and America for cultural and intellectual supremacy. 3) Other European countries are soon ex pected to send their best resources to contribute to the creation of European Ivy leagues. 4) Behind all this is the idea that worldclass universities can make a dispropor tionate contribution to economic growth.

Directions for Questions 362 and 363: Each of the following questions has a paragraph from which the last sentence has been deleted. From the given options, choose the option that completes the paragraph in the most appropriate way. 362.

Globalization is going deeper than just the competition for talent: a growing number of countries are trying to create an elite group of "global universities" that are capable of competing with the best American institutions. China and India are focusing resources on a small group. The French and German governments are doing battle with academic egalitarians in an attempt to create European Ivy Leagues.

Most people who bother with the matter at all would admit that the English language is in a bad way, but it is generally assumed that we cannot by conscious action do anything about it. Our civilization is decadent and our language -- so the argument runs - must inevitably share in the general collapse. It follows that any struggle against the abuse of language is a sentimental archaism, like preferring candles to electric light or hansom cabs to airplanes.

Directions for Questions 364 to 366: Each of the following questions has a paragraph from which the last sentence has been deleted. From the given options, choose the option that completes the paragraph in the most appropriate way. 364.

1) It is no surprise since Galileo was pros ecuted for saying that the earth revolved around the sun; old is always gold new is always askew. 2) Underneath this lies the half-conscious belief that language is a natural growth and not an instrument, which we shape for our own purposes.

“You should never read just for “enjoyment.” Read to make yourself smarter! Less judgmental. More apt to understand your friends’ insane behavior, or better yet, your own. Pick ‘hard books.’ Ones you have to concentrate on while reading. And for God’s sake, don’t let me ever hear you say, ‘I can’t read fiction. I only have time for the truth.’ 1) Fiction is the truth, fool!” 2) It means that you can’t read it!”

73

Verbal Ability 3) But non-fiction can hardly qualify as truth; at best it is manipulated informa tion!” 4) Truth has nothing to do with anything!” 365.

DIRECTIONS for questions 367 and 368: Each of the following questions has a paragraph from which the last sentence has been deleted. From the given options, choose the option that completes the paragraph in the most appropriate way.

Experimental psychology seems to produce a stick-figure version of our doings. To give an example of what worries me: It’s easy to demonstrate that tasters prefer cheap wine when it’s presented in fancy bottles. That experiment is cited in almost every overview on the topic of choice. But what we get wrong is easier to discuss than what we get right. How is it that, when they’re not fooled, humans can be oenophiles, discerning and enjoying subtle differences between vintages of burgundy? Why don’t we all drink milk or simple syrup?

367.

1) It’s high time psychologists explained the truth behind our appreciation of wine. 2) Psychology is less attentive to drinks than to the containers that hold them. 3) Our sense of taste might be influenced by packaging but is not rooted in it. 4) Humans are superior beings and psy chologists need to acknowledge this fact.

You won't find a society where people don't have some notion of fairness, don't put some value on loyalty and kindness, don't distinguish between acts of cruelty and innocent mistakes, don't categorize people as nasty or nice. These universals make evolutionary sense. Since natural selection works, at least in part, at a genetic level, there is a logic to being instinctively kind to our kin, whose survival and well-being promote the spread of our genes. More than that, it is often beneficial for humans to work together with other humans, which means that it would have been adaptive to evaluate the niceness and nastiness of other individuals. 1) All this is reason to consider the socio logical basis of at least basic moral concepts. 2) All this is reason to consider the adaptive nature of at least basic moral concepts. 3) All this is reason to consider the innatene ss of at least basic moral concepts. 4) All this is reason to consider the benefits of at least basic moral concepts.

366. A. Professionalism has certainly travelled a long way in a short time. B. Now, amateurism is a byword for sloppi ness, disorganisation and ineptitude, while professionalism-is the default description of excellence. C. In the space of a hundred years, the words "professional" and "amateur" have virtually swapped places. D. At the end of the 19th century, an amateur meant someone who was motivated by the sheer love of doing something; profes sional was a rare, pejorative term for grubby money-making. 1) DBCA 2) DBAC 3) CADB 4) ACDB

368.

For a long time, only one area of human activity appeared to be immune to quantification; in the cozy confines of personal life, we rarely used the power of numbers. The techniques of analysis that had proved so effective were left behind at the office at the end of the day and picked up again the next morning. The imposition, on oneself or one's family, of a regime of objective record keeping seemed ridiculous. 1) It was a convenient way of de-stressing oneself from the pressures of work. 2) A journal was respectable; a spreadsheet was creepy. 3) In science and in business, numbers have won fair and square. 4) And yet, almost imperceptibly, numbers are infiltrating the last redoubts of the personal.

74

Verbal Ability DIRECTIONS for questions 369 and 370: Each of the following sets of questions has a sentence with two blanks. Given below each question are four pairs of words. Choose the pair that best completes the sentence.

E. The finest, and the only first-rate, criti cism is produced when, by an exceptional accident, a creative artist of balanced and powerful temperament is moved to deal exhaustively with a subject.

369.

1) ECDAB 3) CDAEB

It's not _____________ that conflicts over patrimony have _____________ in recent decades thanks to globalizing trends: the increasing circulation of information along with objects and money and the evolution of institutions like museums from sleepy, scholarly repositories of artifacts into entertainment palaces and virtual town squares. 1) 2) 3) 4)

370.

372.

improbable; magnified surprising; metamorphosed coincidental; accelerated incidental; disappeared

Small groups that speak_____________ languages often know things about plant and animal species that are lost when their lands are "developed" and they are _____________ into the larger community. 1) 2) 3) 4)

2) EABCD 4) DBEAC

Directions for Questions 373: Each of the following sets of questions has a sentence with two blanks. Given below each question are four pairs of words. Choose the pair that best completes the sentence.

Directions for Questions 371 and 372: The sentences given in each question, when properly sequenced form a coherent paragraph. Each sentence is labeled with a letter. Choose the most logical order of sentences from among the given choices to construct a coherent paragraph. 371.

A. Instead of interrogating their inner worlds through talking and writing, they are using numbers. B. From the languor of the analyst's couch to the chatty inquisitiveness of a selfhelp questionnaire, the dominant forms of self-exploration assume that the road to knowl edge lies through words. C. They are constructing a quantified self. D. Even as therapeutic concepts of the self spread widely in simplified, easily ac cessible form, they retained something of the prolix, literary humanism of their inventors. E. Trackers are exploring an alternate route. 1) BDAEC 3) BDEAC

threatened; absorbed endangered; sundered extant; assimilated complex; submerged

2) CDABE 4) EBCDA

373.

A. On the other hand, a creative artist cannot do the work of a critic because he has neither the time nor the inclination to master the necessary critical apparatus. B. Hence critical work seldom or never sat isfied the artist, and the artist's ideal of what critical work ought to be is an impossible dream. C. There is a one-sided feud between artists and critics. D. Artists expect from critics an imaginative comprehension, which in the nature of the case only a creative artist can pos sess.

An awesome, and roughly , example of in a given field can be an especially important impetus to similar accomplishment. 1) 2) 3) 4)

374.

75

simultaneous; skill paradoxical; genius unparalleled; poise contemporaneous; prowess

A. Somewhere on the planet, wind drops, energy stalls, lights go out. B. And whether we know it or not, we’re all in trouble. C. As we know, everything is connected. D. So what happens when the butterfly in Brazil, whose fluttering wings are creat ing wind-power in Texas, dies off because

Verbal Ability 379.

of pollution or deforestation? 1) DABC 2) DACB 3) CDAB 4) DCAB Directions for Questions 9 and 10: Each of the following sets of questions has a sentence with two blanks. Given below each question are four pairs of words. Choose the pair that best completes the sentence. 375.

Rigorous study of what psychologists call emotion is fairly new, and for obvious reasons has focused far more on untamed passions than on the _____________________________________ variety. 1) 2) 3) 4)

376.

Sheer facility, whatever its and appeal, need not denote depth of content. 1) 2) 3) 4)

377.

Directions for questions 380 and 381: Each of the following questions has a paragraph from which the last sentence has been deleted. From the given options, choose the option that completes the paragraph in the most appropriate way.

illustrative; aptitude descriptive; intelligence rhetorical; originality pedagogical; diversity

380.

domestically by the weight Still of under-class voting blocs, the aspirations of the Indian middle-class have found expression in the abandonment of India’s neutralist ideological, foreign policy for a military and diplomatic rapprochement with the United States. 1) 2) 3) 4)

378.

intelligence; pleasant regulation; domesticated stability; reliable sensitization; placid

bound; tentative fettered; burgeoning shackled; nuanced relegated; thriving

None of the greatest Western philosophers has produced a paean to pleasure that can serve as much of a guide for today’s enlightened hedonist. Philosophers tend to be ruminative, cerebral and cautious. To expect to find a hedonist at work in the groves of academe is rather like expecting to find a vegetarian at work in a slaughter-house. Thus Kant preached a stern gospel of dutifulness, and Plato’s pleasures were unstintingly abstract and intellectual. 1) But both of them could have done with a little dose of hedonism! 2) But there is nothing that justifies plea sure being singled out as being unworthy of philosophical enquiry. 3) A good Platonist would rather contem plate the perfect meal than eat it. 4) Abstraction and correctness are the cor nerstones of philosophy and hedonism its outpost.

Culture, in compelling but unspoken ways, the proven, acceptable methods by which members of the group address problems. 1) 2) 3) 4)

A. The poem asks the reader to work at it, refusing easy answers, and even Eliot’s own explanatory notes raised further questions. B. In its 434 lines, T.S. Eliot assembled Arthurian legend, Jacobean tragedy, Bud dhist scrip ture, Roman lyric and street slang into a puzzle to be worked out by pale and interesting types the world over. C. Published in 1922, “The Waste Land” soon became a landmark of Western lit erature. D. And that is precisely why it wouldn’t be published now; in becoming “consumers” of culture, we’re losing the ability to engage with it. E. Some interpreted it as the private jottings of a depressive polymath; others as the vision of a civilisation which God had forsaken. 1) CABED 2) CBAED 3) CBEAD 4) CAEBD

emphasizes; contingent negates; typical threatens; catastrophic dictates; recurrent

76

Verbal Ability 381.

Whether in the work of the Pre-Raphaelites, the Futurists or the Abstract Expressionists, innovation in the visual arts has always happened when a group of progressive young artists meet, work and exhibit together. Though the act of creation is highly personal, it rarely happens in isolation—even if, as in the case of Braque and Picasso, it was only two young men “yoked together” in their exploration of cubist space.

achieved by other sciences. 3) But if you put the same question to a philosopher, he will say that unlike other sciences, philosophy is subjective and you cannot determine anything with cer tainty. 4) But if you put the same question to a philosopher, he will probably be able to elaborate much more and probably much deeper than people from other disciplines.

1) The unrecognized genius who dies alone in a garret is largely a myth. 2) But it does not mean that art is created in groups. 3) It is only the rare genius who can create in isolation. 4) Genius of course needs no companion; only adulation, maybe.

383.

Directions for questions 382 and 384: Each of the following questions has a paragraph from which the last sentence has been deleted. From the given options, choose the option that completes the paragraph in the most appropriate way. 382.

I think we underemphasize the prevalence of certain normal errors inherent in medical practice. Surgeons are fooled when they open an acutely painful abdomen only to find a normal appendix: in the days before CT scans, it was said that if that didn’t happen once in a while, you weren’t operating often enough. When in doubt, it was safer (and wiser) to operate than to risk a rupture and peritonitis, even if the diagnosis was “wrong.” 1) Here was an error that wasn’t an error, but rather a predictable side effect of bal ancing known risks with imperfect infor mation. 2) I suggest that we apply a similar principle to the prescribing of narcotic painkillers and anti-anxiety drugs. 3) To put it another way, I’d rather be con sidered incompetent once in a while than know that my suspicion had denied someone legitimate help. 4) Such false positives and false negatives are part and parcel of medical profession and there is no way you can eliminate them.

Philosophy, like all other studies, aims primarily at knowledge. The knowledge it aims at is the kind of knowledge that gives unity and system to the body of the sciences, and the kind that results from a critical examination of the grounds of our convictions, prejudices, and beliefs. But it cannot be maintained that philosophy has had any very great measure of success in its attempts to provide definite answers to its questions. If you ask a mathematician, a mineralogist, a historian, or any other man of learning, what definite body of truths has been ascertained by his science, his answer will last as long as you are willing to listen.

384.

1) But if you put the same question to a philosopher, he will say that philosophy could establish the truth of certain an swers to such fundamental questions per taining to life. 2) But if you put the same question to a philosopher, he will, if he is candid, have to confess that his study has not achieved positive results such as have been

77

A. Knowledge is acquired when we succeed in fitting a new experience into the system of concepts based upon our old experiences. B. Understanding comes when we liberate ourselves from the old and so make possible a direct, unmediated contact with the new, the mystery, moment by moment, of our existence. C. The new is the given on every level of experience—given perceptions, given emotions and thoughts, given states of

Verbal Ability evident that the various sensations, or ideas imprinted on the sense, however blended or combined together (that is, whatever objects they compose), cannot exist otherwise than in a mind perceiving them. E. There was an odour, that is, it was smelt; there was a sound, that is, it was heard; a colour or figure, and it was perceived by sight or touch; this is all that I can understand by these and the like expres sions. 1) DBCEA 2) DBECA 3) DABCE 4) ADBEC

unstructured awareness, given relation ships with things and persons. D. The old is our home-made system of ideas and word patterns. E. Knowledge is primarily a knowledge of these finished articles. F. It is the stock of finished articles fab ricated out of the given mystery by memory and analytical reasoning, by habit and the automatic associations of accepted notions. 1) DCBAFE 2) AFEBCD 3) DCAFEB 4) ABCDFE Directions for question 385: Each of the following questions has a sentence with two blanks. Given below each question are four pairs of words. Choose the pair that best completes the sentence. 385.

The entire point of religion, at least of the theistic kind, is to affirm a to an authority and to a set of that exceed, and sometimes clash with, what is required by the state. 1) 2) 3) 4)

386.

Directions for Questions 387 and 388: Each of the following questions has a paragraph from which the last sentence has been deleted. From the given options, choose the option that completes the paragraph in the most appropriate way. 387.

tendency; laws fidelity; imperatives loyalty; morals servitude; codes

For many centuries in the Western tradition, how well you expressed a position corresponded closely to the credibility of your argument. Rhetorical styles might vary from the spartan to the baroque, but style itself was never a matter of indifference. And “style” was not just a well-turned sentence: poor expression belied poor thought. 1) Unfortunately today, style has become everything; a fascinating turn-of-phrase has trumped originality of thought. 2) Confused words suggested confused ideas at best, dissimulation at worst. 3) What passes for style today is not origina lity but plagiarism; a mish-mash camou flaged as fusion. 4) Style was content; content had to find style.

A. For as to what is said of the absolute existence of unthinking things without any relation to their being perceived, that is to me perfectly unintelligible; it is not possible that they should have any ex istence out of the minds or thinking things that perceive them. B. I think an intuitive knowledge may be obtained of this by any one that shall attend to what is meant by the term exist when applied to sensible things. C. The table I write on I say exists, that is, I see and feel it; and if I were out of my study I should say it existed - meaning thereby that if I was in my study I might perceive it, or that some other spirit actually does perceive it. D. That neither our thoughts, nor passions, nor ideas formed by the imagination, exist without the mind, is what every body will allow. And to me it is no less

388.

The popularity of science fiction in the visual media had a profound effect on the genre. Science fiction authors who had been content with an elite circle of readers now wanted the recognition and money that went with mass appeal. Escapism reigned supreme. The literary excellence of the 1960s was replaced by a return to the simpler storytelling of the 1940s. 1) This has helped these writers to reach

78

Verbal Ability a wider audience leading to many more people reading this stuff. 2) Even this failed to bring in more readers, and this genre soon started declining giving way to other genres in literature. 3) Rather than developing in new areas, writers turned to the easy money found in producing sequels to great works of the past. 4) This period coincided with the American counterculture movement, which es poused a return to simpler past rather than the scientific future. 389.

391.

The millions of urban ___________ pouring into mega cities each year are not simply a new global migrant underclass, __________to live in chaos and work in the shadow economy; instead, they often form functional, selforganizing ecosystems that are “off the grid”. 1) 2) 3) 4)

390.

4) Nouvel's buildings, however, not only grab you; they get you thinking about ar chitecture in a more serious way. A. But it won't be a smooth ride. B. Investors trying to manage political risk have historically moved between ex tremes of "optimism and skittishness", which sounds a lot like the current situation. C. Political risk adds new complexity to markets, and as the system gets more complex, it becomes harder to forecast. D. That doesn't mean you should put your money in gold; over time, even volatile markets can rise. 1) CBDA 2) BCDA 3) CBAD 4) BCAD

Directions for questions 392 to 393: Each of the following questions has a sentence with one/two blank(s). Given below each question are four options. Choose the option that best completes the sentence.

immigrants; authorized inmates; apportioned residents; permitted squatters; consigned

392.

When you catch your first glimpse of 100 Eleventh Avenue, a new apartment tower in Chelsea designed by the French architect Jean Nouvel, its curving façade, an abstract arrangement of windows slanting in multiple directions, looks like a gimmick. The building clatters; it jangles like a bracelet. Beside the smooth, milky-white exterior of Frank Gehry's I.A.C. Building, across Nineteenth Street, it seems nervous, and even the architect's description of it, "a vision machine", smacks of trying too hard. But Nouvel, one of the world's most famous and prolific architects, is anything but insecure. Often when an architect hits you with razzle-dazzle, it's because that's all he's got.

Rising __________ might not lead to a widespread feeling of __________ as long as people can aspire to climb up the ladder. 1) 2) 3) 4)

393.

disparity; anomie despondency; apathy inequality; inequity imbalances; hostility

By turning chefs into entertainers, we have widened the __________ between ourselves and the once ordinary task of cooking. 1) niche 3) abyss

1) Nouvel's buildings are no different flashy and stylish, they epitomize what a building should look like. 2) Nouvel's buildings epitomize what you call an "architect's vision". 3) These buildings represent a self-suffi cient and a self-contained world in itself, something that every architect aspires for.

79

2) breach 4) crevice

Reading Comprehension Reading Comprehension

DIRECTIONS for questions 394 to 442: The passages given below is followed by a set of questions. Choose the best answer to each question. 'You should write a book about how to write,' my wife said in June of 1974 when I was complaining to her, as I often did, that I had run out of things to write about. Caroline's suggestion came from out of nowhere - I had never thought of writing a textbook - but it felt right. I had then been teaching my course at Yale for four years, and I liked the idea of trying to capture it in a book. Many questions, however, occurred to me. Who would I be writing for? What tone should I adopt? How would my book differ from all the other books on writing? The dominant manual at that time was The Elements of Style, by E. B. White and William Strunk Jr. The Elements of Style was essentially a book of pointers and admonitions: Do this, don't do that. As principles they were invaluable, but they were only principles, existing without context or reality. What his book didn't teach was how to apply those principles to the various forms that nonfiction writing can take, each with its special requirements: travel writing, science writing, business writing, the interview, memoir, sports, criticism, humour. That's what I taught in my course, and it's what I would teach in my book. I wouldn't compete with The Elements of Style; I would complement it. The personal voice of the teacher, not the literary voice of the essayist, was the one I wanted narrating my book. It would have to come from an entirely different field, and it did. My model for On Writing Well was American Popular Song: The Great Innovators, 1900-1950, by the composer Alec Wilder. To write his book, Wilder examined the sheet music of 17,000 songs, selecting 300 in which he felt that the composer had pushed the form into new territory. Along with his text, he provided the pertinent bars of music to illustrate a passage that he found original or somehow touching. But what I loved most about Wilder's book went beyond his erudition. It was his total commitment to his enthusiasms, as if he were saying: 'These are just one man's opinions - take 'em or leave 'em.' His pleasure was to praise. That connected with my own principle of not teaching by bad example. I may cite some horrible example of jargon or pomposity to warn against the prevailing bloatage, but I don't deal in junk. Writing is learned by imitation, and I want my students to imitate the best. I would write from my own convictions - take 'em or leave 'em - and I would illustrate my points with passages by writers I admired. I would treat the English language spaciously, as a gift waiting for anyone to unwrap, not as a narrow universe of grammar and syntax. Above all, I would try to enjoy the trip and to convey that enjoyment to my readers. I began by writing brief chapters, in a teacher's style, often in first person, on fundamental principles such as clarity, simplicity, brevity, usage, voice and the elimination of clutter. Then I settled into the heart of the book - longer chapters explaining how to write a lead, how to write an ending, how to conduct and construct an interview, how to write about travel and technology and sports, and how to write other forms of nonfiction. Throughout, I supplied examples of writing I admired. My authors were widely different in personality and style, but they all wrote well. That was the premise I wanted to establish: that nonfiction is hospitable to an infinite number of voices if the writing is good. The book got a few pleasant reviews and sold in modest numbers, matching my own modest expectations. But then I began to get letters and calls from colleges inviting me to come and talk about writing to their students and faculty - a visit that often began with a lecture that was open to the whole town. Deciding which invitations to accept, I chose colleges in parts of America where I had never been. Almost all of them were colleges I had never heard of.

80

Reading Comprehension The huge bonus of those travels was to put me in touch with my readers. They told me which parts of the book they found most helpful and what subjects they hoped I would cover in future editions. What they liked most was that I made myself available. They weren't hearing from a professor; they were hearing from a writer who had wrestled with the same problems they were facing. They also liked the book's humour. Students, especially, couldn't believe they had been assigned a textbook that actually made an effort to keep them amused. So I was persuaded that my initial fear of immodesty was misguided. The best teachers of a craft, I saw, are their own best textbook. Students who take their classes really want to know how they do what they do - how they grew into their knowledge and learned from their wrong turns. It now occurs to me that I didn't really find my style until I wrote On Writing Well, at the late age of 52. Until then my style more probably reflected who I wanted to be perceived as - the urbane columnist and humorist and critic. Only when I started writing as a teacher and had no agenda except to be helpful did my style become integrated with my personality and my character.

ªScore Enhancerª 394.

Why does the author decide to write a book about writing? 1) Because he likes his wife's suggestion. 2) Because he cannot think of anything else to write about. 3) Because he wants to teach his readers how to apply the principles of language to various forms of writing. 4) Because he wishes to write from the heart and in his own unique style of a teacher rather than a writer. 5) All of the above.

395.

What is the author's opinion regarding imitation? 1) He regards imitation akin to plagiarism, and strongly advocates developing one's own unique style. 2) He believes that the styles of only well-known authors are worthy of imitation. 3) He believes that one must write with conviction and enthusiasm, enjoying the art, and imitate only the best. 4) He feels that imitation is essential for creating good literary pieces. 5) He is against imitation since he feels it destroys the individuality and creativity of the imitator.

ªScore Maximiserª 396.

Which of these was a fundamental reaction of college students to On Writing Well? 1) They felt it offered deep insight into the complex world of language. 2) They felt it most importantly had a unique sense of humour unlike other books on language, which indulged in cheap attempts at amusing readers. 3) They felt a personal connection with the author and empathized with his problems. 4) They did not appreciate the book considering it intellectual jargon. 5) They felt the book was helpful but needed additional inputs to make it universally interesting.

ªScore Maximiserª 397.

Suggest a suitable title for the passage. 1) The Genesis of Writing 3) Develop your Own Style 5) The Creative Genius Inside Us

2) Writing on Writing 4) The Rediscovery of Language

81

Reading Comprehension Over the past decade, authoritarian capitalist countries have built impressive economic resumes. China has grown by over 10 percent in most years, and some of its biggest cities, like Shanghai, now boast per-capita incomes of more than $7,000 per year, the same level as a middle-income nation. Russia, all but bankrupt in the late 1990s, has delivered strong enough growth that it now boasts the thirdlargest capital reserves in the world and has built its gas companies into such powerhouses that they now dominate the markets of Europe. The authoritarian capitalists proved so successful, in fact, that some in the West began wondering whether their model of development had surpassed liberal democratic capitalism. In Russia, Vladimir Putin kept populations quiet by assuring economic safety and prosperity. Many still give in to this promise - for without Putin, people visualized an even more authoritarian government, chaos, poverty, hunger, strikes and so on. In China, the regime made a similar bargain, if not with the masses, at least with its urban middle classes. The regime's investment and largesse was slanted towards the big cities. As Deng Xiaoping vowed when he opened China's economy, 'Some will get rich first' – and those nouveau riche would appreciate who paid for their cars, homes, and glittery new mobile phones. In recent years, according to China expert Jonathan Unger, the government has made a deliberate policy of favouring this population. In a poll by the Pew Research Centre, over 80 percent of Chinese said they were satisfied with conditions in their country, among the highest of responses in the world. Even after two terms in office, Vladimir Putin enjoyed popularity ratings that would be the envy of any Western leader. But it matters not what the people of these regimes eat in good times, but in bad. The autocracies are particularly poorly prepared for a global economic crisis because they have weak domestic consumer markets and rely upon exports to survive. Powerful authoritarian regimes like Russia and the Persian Gulf states are dependent on exports of petroleum or one sole commodity. In Venezuela, energy accounts for some 95 percent of all export revenue. In Iran, it provides some 80 percent of revenues. But the price of oil has dropped by more than half in the past six months. And China, 35% of whose GDP comprises of exports, which depends largely on exporting manufactured goods to wealthy nations, will also suffer from the financial crisis as consumer spending drops in the U.S., Europe and Japan. Despite valiant efforts to assure their people that nothing is wrong, the autocrats cannot cover their economic holes. In Russia, where the stock market has fallen by some 70 percent since last spring and the rouble has weathered fierce attacks, Vladimir Putin recently declared he would launch new tax cuts because of the steep drop in Russia's economy. While unemployment, poverty and unrest indicate cracks in the system of autocracy, there are signs that a nascent movement toward liberal democracy could take its place. Indeed, increasing numbers of Chinese are challenging the government, and in December, 303 Chinese intellectuals signed and published a daring manifesto called Charter 08, which demands an end to one-party rule. Charter 08 is only one sign that the autocracies are feeling the pressure. In December, in Russia, the police arrested some 100 people at a protest in the poor eastern city of Vladivostok; at roughly the same time, 1,000 people attended a protest in Moscow against the government. The autocracies have money to burn. Unlike 20th-century autocrats, such as Fidel Castro, who led their countries in wars of independence, most of today's leaders came up through the political system and have no revolutionary bonafides to play. The modern authoritarian governments abandoned real ideology long ago. China remains a nominally communist country, but if Karl Marx were to visit today, he would be horrified. With policies that favour the urban elite and virtually no social welfare programs left, this 'communist' nation has become one of the most unequal societies in Asia. Lacking any ideology other than sour nationalism, the new autocrats cannot rally their population in down times by appealing to their political idealism, as they did in the 1950s and early 1960s. This combined with considerable economic freedom, make for a lethal combination.

82

Reading Comprehension In order to improve their standing on the world stage, today's autocrats at least try to create the facade of democracy. Their people know about democratic movements in other countries, can access free media, and are not easily subdued. Because the authoritarian governments have created some semblance of a legal system, workers have begun to think they have rights. But neither the short term nor the long term looks good for Moscow, Beijing, and the other autocrats. In the near future, their economies will slow down severely and, in the case of Russia, likely fall into a serious recession. In China, many analysts believe unemployment will rise to its highest level in a decade. Growth is likely to dip below 8 percent, the magic number needed to keep creating enough jobs for all the people entering the work force in China. Millions of Chinese migrant workers who can no longer find factory jobs will return to the interior of the country. Back in rural areas, anger is already rising. These unemployed workers, who have seen the wealth of urban elites in cities like Shanghai, could begin organizing larger demonstrations. Thus far, the autocracies have kept groups of people with grievances against the government from forming united fronts. Moscow has achieved this through the skilful use of nationalism, which drives a wedge between liberal Russians and ethnic minorities with grievances against the government. Beijing has used a combination of crackdowns and payoffs to stop demonstrators to keep labour protests separate from one another, preventing them from developing a common theme or common leaders. Divide and conquer, though, won't work forever. In China, rural and urban protests might soon begin to link up – through activist networks, religious groups or blogs – and form a national protest. Charter 08 and a nationwide taxi-driver strike, both organized on the Internet, are a first hint of this nationwide movement. The Great Depression fed dangerous new autocratic ideologies like fascism and communism; it can be said that a second Great Depression could destroy them.

ªScore Maximiserª 398.

Which of the following can be inferred from the passage? 1) Authoritarian governments in Russia and China were formed with the popular support of the people. 2) Continuing support of the masses is essential for any government to survive in the long term. 3) We may see a gradual shift in the ideological position of the Western democracies from being liberal democratic governments to a more interventionist approach. 4) Russia and China have been suppressing dissent for decades, while pushing for economic growth and social reform. 5) Labour unions play a very important role in the functioning of day-to-day activities of the state.

ªScore Enhancerª 399.

The major cause(s) of economic and social crises in the autocratic nations have been: 1) 2) 3) 4)

abuse of authority through cracking down on dissent by force. excessive reliance on one or two export products. coming together of organizations to garner unified support against government policies. absence of a real ideological base amongst current leaders as opposed to the leaders of previous generations who participated in successful revolutions. 5) all of these.

83

Reading Comprehension 400.

What is the tone of the passage? 1) Critical

2) Objective

3) Sarcastic

4) Plaintive

5) Indifferent

ªScore Maximiserª 401.

What can be a suitable title for the passage? 1) 2) 3) 4) 5)

The Fall of Communism Twilight of the Non-democracies A Peek into the Harsh Realities of Authoritarianism The Malady of Recession Autocrats Beware!

When I talk about UFOs in my introductory astronomy classes, I always tell my students that I absolutely believe in UFOs. After a brief pause for incredulous stares, I ask them to think about what the term UFO actually means. I explain that I fully believe there are objects in the sky that the average person may not be able to identify. This does not mean, however, that no one can identify these objects. It only means that they could appear 'unidentified' to someone who is not familiar with the sky or with the full range of sky phenomena that can surprise a novice. Indeed, upon more careful investigation, many so-called UFOs turn out to be perfectly natural objects or processes in the Earth's atmosphere or beyond. As the late Carl Sagan emphasized, 'Extraordinary hypotheses require extraordinary proof.' Surely, the notion that some mysterious phenomenon you briefly observed in the sky must be an interstellar spacecraft (and not a human craft, meteor, or a bright planet) qualifies as such an 'extra-ordinary' hypothesis! Yet, amazingly, given the number of UFO incidents believers report, not one UFO has left behind any proof - a piece of spacecraft material or machinery (or even a sandwich wrapper) that laboratory analysis has shown to be of clearly extraterrestrial origin. It's also remarkable how unlucky the UFO occupants are in their choice of people to kidnap. Never do 'aliens' seem to snatch a person with a good knowledge of astronomy or physics or someone with high-level government clearance. Time after time, their 'victims' turn out to be homemakers, agricultural workers, or others whose relevant knowledge base seems to be limited to reading UFO enthusiast literature. Even UFO sightings turn out to be reserved (for the most part) for those who have not studied the sky in any serious way. Although the world's supply of professional astronomers is not much larger than the population of Wasilla, Alaska, the world has many tens of thousands of active amateur astronomers who spend a great deal of time observing the sky. You would think that if UFOs really are alien spacecraft, a large majority of reported sightings would come from this group. Yet, unsurprising to astronomers, you almost never get UFO reports from experienced amateurs whose understanding of what they see in the sky is much more sophisticated than that of the average person. All of which does not mean that astronomers in general are pessimistic about the presence of intelligent life on planets around other stars. Indeed, many observations over the last few decades have increased the level of optimism in the astronomical community about the potential for life to exist out there. Primary among these is the discovery of more than 300 planets around relatively nearby stars, which certainly shows that planetary systems like our own may be far more common than we dared to hope. We just don't think that intelligent aliens are necessarily visiting Earth.

84

Reading Comprehension The problem is that the stars are fantastically far away. If our Sun was the size of a basketball (instead of 864,000 miles across), Earth would be a small apple seed about thirty yards away from the ball. On that scale, the nearest star would be some 4,200 miles (7,000 km) away, and all the other stars would be even farther! This is why astronomers are sceptical that aliens are coming here, briefly picking up a random individual or two, and then going back home. It seems like an awfully small reward for such an enormous travel investment.

ªScore Enhancerª 402.

The primary purpose of the author in the passage is to: 1) make distinctions between those who are mere lay people interested in the extraterrestrial and qualified scientists and astronomers. 2) present us with a wealth of information to enable us to make our own judgements regarding UFOs. 3) justify the presence of UFOs through examples of sightings and people's accounts. 4) debunk claims about UFO sightings by lay people, using reasoning and analysis to back his assertion. 5) state that though he does not believe in the stories of UFO sightings, he believes in the existence of aliens.

403.

The author's statement in the first paragraph that he 'absolutely believes' in UFOs is: 1) 2) 3) 4) 5)

404.

purely sarcastic. intended to ridicule those who believe in UFO sightings. sarcastic, but also with a different meaning to it. an honest and genuine assertion. none of these.

Which of these is a main argument of the author against the reported sightings of UFOs? 1) UFO sightings are uncommon and seem to be experienced only by lay people, and not by astronomers; hence they are unrealistic. 2) UFO sightings seem to happen only to amateurs with basic scientific knowledge. 3) UFO sightings are common only in certain areas and therefore cannot be described as a widespread or universal phenomenon. 4) UFO sightings are little more than fantasy since there is little chance of intelligent beings existing on any planet apart from Earth. 5) UFO sightings can be verified and legitimized only by scientists and astronomers, and no such 'sightings' till date have been thus verified or legitimized.

405.

Which of these is the author's precise outlook on the subject of UFOs and extraterrestrial life? 1) He believes that the presence of intelligent life apart from human beings is possible, but that aliens are visiting us in UFOs is merely an illusion. 2) He believes that extraterrestrial life exists, but it is not advanced enough to send 'UFO's' into space. 3) He believes that extraterrestrial life exists, but UFOs are mere fantasy. 4) He advances the idea of extraterrestrial life while being non-committal as regards the existence of UFOs. 5) He merely states that astronomers believe that extraterrestrial life can exist, without revealing his own opinion, but debunks UFO theories.

85

Reading Comprehension ªScore Maximiserª 406.

The argument made in the last paragraph, that it would be unrealistic for aliens to merely visit earth due to the distance involved, can be refuted by which of the following counter-arguments? 1) Extraterrestrial beings may be much more advanced than human beings and could travel faster. 2) The assertion that a visit by aliens would be costly for them is made by astronomers based on empirical data only. 3) We do not possess technology advanced enough to make judgements about the presence and development of an alien society. 4) There is general divergence of views upon this subject among astronomers and other scientists. 5) None of these.

Wouldn't you feel more comfortable at work if you had an employer who 'spoke your language?' In other words, wouldn't it be easier for an employee and an employer to communicate effectively if they were aware of each other's cultures, beliefs and morals? Some may argue that leadership skills are universal regardless of what country the leader is in. On the other side of the debate are those that say that leaders must adapt their leadership style to different national cultures. But it is essential that leaders understand the culture of their employees. Leaders cannot choose their styles at will. They are constrained by the cultural conditions that their followers have come to expect. The reason behind this is clear. Different cultures perceive what a leader may be trying to communicate differently. For example, a Japanese employee may see what may be seen as harsh to an American employee, as normal disciplinary action. While it is true that business is global, business organizations have a culture shaped by the business it is in and the people who run the business. Managers are products of the distinct cultures in which they have learned and conducted business. Imagine a situation in which a manager who was trained at an American school is asked to run the Japanese manufacturing facility of his British firm. This individual needs to understand the culture within which he works and how his employees perceive leadership. Research has found that 'one size does not fit all'. A manager needs to modify leadership qualities, tailored to the unique culture within which he or she works. Communication skills are also important to the leader. But again, how these skills are perceived differs among and within cultures. What one culture views as effective communication may be seen as unclear in another. For example, American managers are more likely to provide directions to lower level workers on a face to face basis while Japanese managers are likely to use written memos. Likewise, in the United States, a manager will most likely give direct negative feed back to an employee, whereas in Japan, a manager will have the information relayed by a peer of the employee. Charismatic leadership is thought to broaden and elevate the interest of followers, generate awareness and acceptance among the followers of the purpose and mission of the group and motivate followers to go beyond their self-interest for the good of the organization. But different cultural groups may vary in their conceptions of the most important characteristics of charismatic leadership. For example, in some cultures, one might need to take strong, decisive action in order to be seen as a leader, while in other cultures an independent approach may be the preferred approach to executing effective leadership. Leaders are expected to have vision, but how this is displayed differs from culture to culture. Leaders are often thought to be risk takers but research found that risk taking is not universally valued as contributing to outstanding leadership. In conclusion, national cultures affect leadership style by the way of the follower. Knowing the manner in which the follower perceives communication, vision, risk taking and charisma is key when a leader is developing his leadership techniques for a culture that is not his own.

86

Reading Comprehension ªScore Enhancerª 407.

From the comparison between American and Japanese work cultures we can infer that: 1) American employees prefer a more honest assessment of their work by superiors as compared to Japanese employees. 2) American employers tend to value organizational discipline less seriously than Japanese employers. 3) American employers believe in cooperation and goodwill while Japanese employers believe in authority and discipline. 4) American employees prefer a more open and informal work environment as opposed to Japanese employees who prefer discretion and formality. 5) American employers tend to be aggressive and performance oriented while Japanese employers lay emphasis on the personal touch.

408.

Which of the following have been described in the passage as qualities most essential for effective leadership? 1) An ability to identify oneself with the culture of the place where one is a leader, and possessing a democratic attitude. 2) An ability to understand and appreciate the culture in which one is a leader, and possessing charisma and communication skills. 3) Being strong-willed and uncompromising, and possessing risk-taking ability. 4) An ability to adapt oneself to the changing scenarios in the corporate world, and lead according to the circumstances. 5) Understanding the culture in which one is working, and being charismatic, visionary and communicative in one's approach.

ªScore Maximiserª 409.

What stance does the author take as regards the debate about the universality of leadership skills? 1) He disagrees with the popular notion that leadership skills are universal, and uniquely roots for adaptive leadership. 2) He merely seeks to present both sides of the debate and avoids taking a specific stance. 3) He is somewhat impartial in his analysis but seems to slightly favour the idea that leadership skills can be universally applied. 4) He thoroughly explores the arguments and counter-arguments put forth as regards the usage of leadership skills, but finally tilts towards adaptive leadership. 5) None of these.

ªScore Enhancerª 410.

By inference from the passage, which of the following actions of a manager working in an American firm will most satisfy her employees? 1) Taking a tough stand on the issue of employee discipline and imposing penalties for late-coming and absenteeism. 2) Allowing employees to air their grievances freely by following an open door policy throughout the day. 3) Creating work teams to facilitate better coordination at work, and at the same time improving employee relations. 4) Providing quick promotions to those who prove their mettle and backing them to the hilt. 5) Encouraging employees to prepare their own performance evaluation reports and make suggestions, based on loose guidelines.

87

Reading Comprehension 411.

Suggest an appropriate title for the passage. 1) 2) 3) 4) 5)

The Dynamics of Leadership Situational Leadership Styles The Necessity for Adaptive Leadership Universal Leadership Style versus Adaptive Leadership Style American and Japanese Leadership Models: A Comparison

Dine on a steady diet of books like The New Asian Hemisphere: The Irresistible Shift of Global Power to the East or When China Rules the World, and it's easy to think that the future belongs to Asia. As one prominent herald of the region's rise put it, 'We are entering a new era of world history: the end of Western domination and the arrival of the Asian century.' Sustained, rapid economic growth since World War 2 has undeniably boosted the region's economic output and military capabilities. But it's a gross exaggeration to say that Asia will emerge as the world's predominant power player. At most, Asia's rise will lead to the arrival of a multi-polar world, not another unipolar one. Asia is nowhere near closing its economic and military gap with the West. The region produces roughly 30 percent of global economic output, but because of its huge population, its Per Capita GDP is only $5,800, compared with $48,000 in the United States. Asian countries are furiously upgrading their militaries, but their combined military spending in 2008 was still only a third that of the United States. Even at current torrid rates of growth, it will take the average Asian 77 years to reach the income of the average American. The Chinese need 47 years. For Indians, the figure is 123 years. And Asia's combined military budget won't equal that of the United States for 72 years. In any case, it is meaningless to talk about Asia as a single entity of power, now or in the future. Far more likely is that the fast ascent of one regional player will be greeted with alarm by its closest neighbours. Asian history is replete with examples of competition for power and even military conflict among its big players. China and Japan have fought repeatedly over Korea; the Soviet Union teamed up with India and Vietnam to check China, while China supported Pakistan to counterbalance India. Already, China's recent rise has pushed Japan and India closer together. If Asia is becoming the world's centre of geopolitical gravity, it's a murky middle indeed. Those who think Asia's gains in hard power will inevitably lead to its geopolitical dominance might also want to look at another crucial ingredient of clout: ideas. Pax Americana was made possible not only by the overwhelming economic and military might of the United States but also by a set of visionary ideas: free trade, Wilsonian liberalism, and multilateral institutions. Although Asia today may have the world's most dynamic economies, it does not seem to play an equally inspiring role as a thought leader. The big idea animating Asians now is empowerment; Asians rightly feel proud that they are making a new industrial revolution. But self-confidence is not an ideology, and the much-touted Asian model of development does not seem to be an exportable product.

ªScore Enhancerª 412.

According to the passage, what was the prime reason for the rise of the United States as a global superpower? 1) 2) 3) 4) 5)

The military and economic might that it has possessed in the post World War 2 era. A truly dynamic economy capable of meeting all sorts of challenges and set-backs. A vision that manifests itself in free trade, Wilsonian liberalism and multilateral institutions. Both [1) and [2) Both [1) and [3)

88

Reading Comprehension 413.

Which of the following best expresses the author's estimate of Asia's rising power? 1) He acknowledges Asia's rise post World War 2 but provides information to argue why Asia cannot be a superpower. 2) He believes that Asia is the most backward continent in the world and its tallclaims of being the future superpower are ridiculous and fantastic. 3) He points out the obstacles Asia faces in its path to becoming a superpower and also suggests measures to overcome them. 4) He is optimistic about the rise of Asia but warns that this would take a very long time. 5) He is amused by the proclamations of Asia's imminent rise and regards these proclamations as delusional fantasies.

ªScore Maximiserª 414.

Which of the following cannot be inferred from the penultimate paragraph? 1) 2) 3) 4)

The Asian countries touted to become superpowers seem to be playing a game of one-upmanship. Two Asian countries often team up with each other in order to check a third’s growth. There is animosity even between Asian countries that are each others' neighbours. Considering that Asian nations refuse to cooperate with each other, it is difficult to see them functioning as a single entity. 5) None of these. 415.

Which of the following methodologies are used by the author to arrive at his conclusion? 1) Using statistics to prove his case and then backing this up with analytical arguments based on economic, social and military factors. 2) Arguing his case using statistics and economic indicators of development together to form a composite analysis. 3) Using a combination of statistics and social indicators to arrive at the conclusion. 4) First presenting some statistics to the readers and based upon these, drawing general conclusions. 5) Using statistics as one basis and economic, social and military indicators as another, separate basis to arrive at a single conclusion.

ªScore Enhancerª 416.

In the passage, the term 'thought leadership' is meant to be which of the following? 1) 2) 3) 4)

The presence of a cohesive identity that would enable the creation of a geopolitical superpower. A Renaissance in scientific thought, culture and the arts in the Asian continent. A visionary, humanistic and broad ideology that serves as a strong factor in geopolitical dominance. A liberal, universal ideology that would bring the Asian continent into the 'free world' and help it gain respect internationally. 5) An ideology that would be acceptable to the whole or most of the civilized world. In the second decade of the twentieth century, it was almost impossible to build an airplane in the United States. That was the result of a chaotic legal battle among the dozens of companies - including one owned by Orville Wright - that held patents on the various components that made a plane go. No one could manufacture aircraft without fear of being hauled into court. The First World War got the industry started again, because Congress realized that something needed to be done to get planes in the air. It created a 'patent pool', putting all the aircraft patents under the control of a new association and letting manufacturers license them for a fee. Had Congress not stepped in, we might still be flying around in blimps.

89

Reading Comprehension The situation that grounded the U.S. aircraft industry is an example of what law professor Michael Heller calls the 'anticommons'. We hear a lot about the 'tragedy of the commons': if a valuable asset (a grazing field, say) is held in common, each individual will try to exploit as much of it as possible. Villagers will send all their cows out to graze at the same time, and soon the field will be useless. When there's no ownership, the pursuit of individual self-interest can make everyone worse off. But Heller says that having too much ownership creates its own problems. If too many people own individual parts of a valuable asset, it's easy to end up with gridlock, since any one person can simply veto the use of the asset. The commons leads to overuse and destruction; the anticommons leads to underuse and waste. In the cultural sphere, ever tighter restrictions on copyright and fair use limit artists' abilities to sample and build on older works of art. In biotechnology, the explosion of patenting over the past twenty-five years - particularly efforts to patent things like gene fragments - may be retarding drug development, by making it hard to create a new drug without licensing myriad previous patents. Even divided land ownership can have unforeseen consequences. Wind power, for instance, could reliably supply up to twenty per cent of America's energy needs - but only if new transmission lines were built, allowing the efficient movement of power from the places where it's generated to the places where it's consumed. Don't count on that happening anytime soon. Most of the land that the grid would pass through is owned by individuals, and nobody wants power lines running through their back yard. The point isn't that private property is a bad thing, or that the state should be able to run roughshod over the rights of individual owners. Property rights (including patents) are essential to economic growth, providing incentives to innovate and invest. But property rights need to be limited to be effective. The more we divide common resources like science and culture into small, fenced-off lots, the more difficult we make it for people to do business and to build something new. Innovation, investment, and growth end up being stifled. Opportunities forgone aren't always easy to see. The effects of overuse are generally unmistakable you can't miss the empty nets of fishing boats working overfished oceans, or the scrub that covers an overgrazed field. But the effects of underuse created by too much ownership are often invisible. They're mainly things that don't happen: inventions that don't get made, useful drugs that never get to market. In theory, one should be able to break a gridlock by striking a deal that would leave all sides better off. Sometimes that happens. Just the other week, for instance, Nokia and Qualcomm settled a three-year-long patent battle, which could accelerate the spread of third-generation cell-phone technology. In a less contentious fashion, products like the DVD player quickly became mainstream and affordable because many companies worked together to form patent pools. One reason deals founder is that there are simply too many interested parties. If, in order to create a new drug, you have to strike bargains with thirty or forty other companies, it's easy to decide that the price is too high. But often things go awry because owners won't make a deal at a reasonable price, as with America's nascent aircraft industry. Or take a problem that bedevils the oil-and-gas industry. When different companies own adjacent patches of an oil field, each will be tempted not only to drill its own patch but also to try to suck out the resources of its neighbour's patch. For geological reasons, overdrilling actually reduces the total amount of oil you can get out of the field - all sides end up worse off. An obvious solution is to have one company do the drilling and share the revenues with the other players. But such agreements are often reached only belatedly, if ever. Recent experimental work by psychologists helps explain why. When something you own is necessary to the success of a venture, even if its contribution is small, you'll tend to ask for an amount close to the full value of the venture. And since everyone in your position also thinks he deserves a huge

90

Reading Comprehension sum, the venture quickly becomes unviable. So the next time we start handing out new ownership rights - whether via patents or copyright or privatization schemes - we'd better try to weigh all the good things that won't happen as a result. Otherwise, we won't know what we've been missing.

ªScore Enhancerª 417.

Going by the explanation in the passage, we can say that the concept of the 'anticommons' is a situation in which: 1) there is no ownership of any asset, so all those who use it try to exploit it for their selfish ends. 2) there are too many property rights and patents, leading to growth, innovation and investment being stifled. 3) there are no private property rights or patents, leading to lack of incentives for innovation and investment. 4) too many parties are allowed use of a valuable asset, which leads to overuse of it, as all the users try to take advantage of it to the fullest. 5) too many parties own various parts of one asset, leading to disuse of the asset because one of the owners refuses to allow its use.

418.

Which of the following can be called a 'tragedy of the anticommons'? 1) A large tract of arable land lies unused because its eccentric owner refuses to let anyone farm on it. 2) A pharmaceutical company invents a potentially life-saving medicine, but is unable to manufacture it due to lack of funding. 3) A new communication technology never gets developed because its main components were manufactured by a company that has gone out of business. 4) A new mode of transport becomes impossible to build because it would require the permission of a dozen different parties, not all of whom are cooperative. 5) An inventor comes up with a revolutionary new type of household gadget, but discovers he can't manufacture it, because the same design is already patented by someone else.

419.

The author is in favour of: 1) changing copyright restrictions. 3) limiting property rights. 5) none of the above.

2) ignoring copyright restrictions. 4) eradicating property rights.

ªScore Maximiserª 420.

According to the author, we do not notice the results of the tragedy of the anticommons, because they are often: 1) intolerable. 4) inconceivable.

421.

2) intangible. 5) incomprehensible.

3) indefensible.

What does the author mean when he says 'Had Congress not stepped in, we might still be flying around in blimps'? 1) If Congress had not interfered, the technology of blimps would have had a chance to be developed further. 2) If Congress had not intervened, airplane technology would not have been developed further, and we would have to make do with earlier technology, i.e., blimps. 3) Congress forced all the patent owners for airplane components to work together in order to break the impasse that had formed in furthering airplane technology.

91

Reading Comprehension 4) If Congress had not become involved, the technology for building airplanes would never have been developed, thus forcing us to resort to blimps for air travel. 5) Congress agreed to step in and break the impasse of airplane component patent holders when it realized that the alternative would be flying in blimps. Video games, in artistic terms, are about where movies were in the late 1920s. That is to say, they comprise a sophisticated and potent but still nascent medium that is only going to become more significant in decades to come. Like all great art, they deserve a criticism to match, and will cease to be great without it. (Witness modern movies - and modern movie criticism.) Commerce and technology will ensure the continued production of video games for years to come, but it will be up to critics whether they are regarded, and regard themselves, as art. Recognizing this inexorable significance, a few years ago I proposed video game reviews to a major newspaper I wrote for. To my surprise, I was told the paper already ran video game reviews. I had to dig deeply into a weekend edition to find this elusive creature: a syndicated featurette that consisted essentially of tips on which button combinations result in specialty moves in Nintendo console games. The Generation X version of a chess column, such tripe embodies the problem with the current state of video game criticism: mechanistic, myopic and often condescending. Many publications, including the paper previously referred to, now run syndicated video game features that are a bit wordier, a bit more narrative, but essentially the same thing: insider-y, superficial, favouring console games over the far superior PC platform. This type of 'criticism' is mostly a sop to alreadyinterested readers, a way of acknowledging they exist without actually offering them anything. Such condescension towards video games can also be seen in journalistic reporting about the medium. This consists predominantly of expressions of horror over video game content and how it will turn our children into lust killers, and business reporting about the 'surprising' success of the medium, product placements within it, etc. In all cases, the emphasis is on extreme cases and curiosities - people with gaming addictions, easter eggs hidden in games - that actually say little about the state of the medium, let alone respect it. Some academic articles and books take video games seriously, but their approach tends to be empiricist, not artistic, focused on the economic or behavioural/social impacts of gaming. What is left is a gaping hole in mainstream popular criticism, cultivating ignorance of what is already an exciting and rich body of outstanding modern (post-1990s computing renaissance) video games: the powerful and often moving drama of 'Planescape: Torment' and 'Baldur's Gate'; the savage wit of 'Grand Theft Auto'; the unearthly beauty of 'Morrowind'; the Hitchcockian tensions of 'Splinter Cell' and 'HalfLife'; the amazingly self-created, neverending worlds of 'EverQuest' and 'Neverwinter Nights'. Any one of these games is enough to illustrate that video games are the next great artistic medium to emerge after movies. It is time for video game criticism to catch up. Obviously, video game criticism faces the elemental external difficulty that all groundbreaking, youthbased movements face: sheer ignorance, suspicion and even hostility from the publishers who still hold the purse-strings (a system even the wonders of the Internet have yet to untangle). However, this is a problem that tends to solve itself economically. As media become more popular and significant, they generate more advertising revenue, and publishers feel the need to make at least a pretence of paying attention.

92

Reading Comprehension However, video games have several unique aspects and circumstances that hobble even passable attempts at serious (if still more tech-minded than literary) criticism. One of these aspects can be summed up as ambiguity - both technological and narrative. Video game technology is not unified and is constantly being updated. Consoles are all different and have different games produced uniquely for them. PCs have wildly different components and processing capacities, changing the way the game looks and plays between computers. (Some games cannot even be played on low-end systems.) Games produced for both PCs and a console can be very different on the differing systems - lacking entire scenes or gameplay features, for example. In this sense, most people never really play a video game the way it was meant to be played back when it was being created on super-duper production computers. With the variations in systems and in audio/video capabilities, it could be said that video games are not a medium but media. An even more significant element of ambiguity is that video games are, obviously, participatory. Some games are extremely interactive; even those that are fairly basic maze-running games challenge the user to come up with a personalized solution. The bottom line is that the playing experience tends to be unique for each player (which indeed is a major appeal of the medium). This can become extreme in online persistent worlds where the game never ends; open-ended games that allow 'play' to continue even when the main story is over; and games that allow (or are hacked to allow) players to create their own games with the code. The old moronic anti-critic complaint, 'Did you even see the same [movie, work of art, TV show, etc.] I did?' can actually become valid in the world of video games. The answer is likely to be 'no', to a significant degree. It's an entertainment boon and a critical bane.

ªScore Enhancerª 422.

What, according to the author, is the relationship between movies and video games? 1) 2) 3) 4) 5)

Movie criticism is good, but video game criticism is not. Video games are more important than movies as art forms. They are both art forms and both need good criticism in order to be good. Video games require good criticism in order to be good, but movies do not. Movie criticism is taken seriously, because movies are art, while video games are not.

ªScore Maximiserª 423.

Which of the following words would the author not use to describe the current state of video game criticism? 1) hostile

424.

2) patronizing

3) superficial

4) undiscerning

5) non-literary

What is the point that the author is trying to make in this passage? 1) Video game criticism is necessarily flawed, because of the ambiguity inherent in the medium. 2) Because video games are still a nascent medium, they don't have the serious criticism they deserve. 3) Video games are the next artistic medium to emerge after movies, and it's time that critics realized this. 4) Critics need to acknowledge that video games are an art form, which is made difficult by the ambiguity inherent in the medium. 5) People need to stop criticizing video games because of their violent content, and start recognizing that they are an artistic medium in their own right.

93

Reading Comprehension ªScore Enhancerª 425.

Which of the following can be inferred about the author? a] b] c] d] e] 1) 4)

426.

He enjoys playing video games. He does not like movies. He used to be a journalist. He thinks that PCs are an unsuitable platform for video games. He thinks that 'Planescape: Torment' and 'Baldur's Gate' are the best video games. [a] and [c] 2) [a] and [b] 3) [b], [c] and [d] [a], [c] and [d] 5) [a], [d] and [e]

Which of these factors would result in an increase in 'ambiguity' (as defined by the author) in a video game? 1) The game allows the players to play its levels in any order they want. 2) The game is designed to run only on certain consoles and PCs with high-end graphics. 3) The game allows the players to choose their skill level at the start, which determines the type of challenges they will face. 4) The game is released in two versions - one for low-end systems and one for highend ones - in which the former lacks certain features of the latter. 5) All of the above.

There has been resistance among both islanders and scholars to acknowledging the reality of self-inflicted environmental damage to Easter Island before the Dutch explorer Jacob Roggeveen's arrival in 1722, despite all the detailed evidence that I have summarized. In essence, the islanders are saying, 'Our ancestors would never have done that,' while visiting scientists are saying, 'Those nice people whom we have come to love would never have done that.' Three specific objections or alternative theories have been raised. First, it has been suggested that Easter's deforested condition seen by Roggeveen in 1722 was not caused by the islanders in isolation but resulted in some unspecified way from disruption caused by unrecorded European visitors before Roggeveen. It is perfectly possible that there were indeed one or more such unrecorded visits: many Spanish galleons were sailing across the Pacific in the 1500s and 1600s, and the islanders' nonchalant, unafraid, curious reaction to Roggeveen does suggest prior experience of Europeans, rather than the shocked reaction expected for people who had been living in total isolation and had assumed themselves to be the only humans in the world. However, we have no specific knowledge of any pre-1722 visit, nor is it obvious how it would have triggered deforestation. Even before Magellan became the first European to cross the Pacific in 1521, abundant evidence attests to massive human impacts on Easter: extinctions of all the land bird species, disappearance of porpoises and tuna from the diet, declines of forest tree pollen in Flenley's sediment cores before 1300, deforestation of the Poike Peninsula by around 1400, lack of radiocarbon-dated palm nuts after 1500, and so on. A second objection is that deforestation might instead have been due to natural climate changes, such as droughts or El Nino episodes. It would not surprise me at all if a contributing role of climate change does eventually emerge for Easter, because we shall see that climatic downturns did exacerbate human environmental impacts by the Anasazi, Maya, Greenland Norse, and probably many other societies. At present, we lack information about climate changes on Easter in the relevant period of A.D. 900-1700: we don't know whether the climate got drier and stormier and less favourable to forest survival (as postulated by critics), or wetter and less stormy and more favourable to forest survival. But there seems to me to be compelling evidence against climate change by itself having caused the deforestation and bird extinctions: the palm trunk casts in Mt. Terevaka's lava flows prove that the giant palm had already survived on Easter for several hundred thousand years; and Flen-ley's sediment cores demonstrate pollen

94

Reading Comprehension of the palm, tree daisies, toromiro, and half-a-dozen other tree species on Easter between 38,000 and 21,000 years ago. Hence Easter's plants had already survived innumerable droughts and El Nino events, making it unlikely that all those native tree species finally chose a time coincidentally just after the arrival of those innocent humans to drop dead simultaneously in response to yet another drought or El Nino event. In fact, Flenley's records show that a cool dry period on Easter between 26,000 and 12,000 years ago, more severe than any worldwide cool dry period in the last thousand years, merely caused Easter's trees at higher elevation to undergo a retreat to the lowlands, from which they subsequently recovered. A third objection is that Easter Islanders surely wouldn't have been so foolish as to cut down all their trees, when the consequences would have been so obvious to them. As Catherine Orliac expressed it,' Why destroy a forest that one needs for his [i.e., the Easter Islanders'] material and spiritual survival?' This is indeed a key question, one that has nagged not only Catherine Orliac but also my University of California students, me, and everyone else who has wondered about self-inflicted environmental damage. I have often asked myself, 'What did the Easter Islander who cut down the last palm tree say while he was doing it?' Like modern loggers, did he shout 'Jobs, not trees!'? Or: 'Technology will solve our problems, never fear, we'll find a substitute for wood'? Or: 'We don't have proof that there aren't palms somewhere else on Easter, we need more research, your proposed ban on logging is premature and driven by fear-mongering'? Similar questions arise for every society that has inadvertently damaged its environment. When we return to this question, we shall see that there is a whole series of reasons why societies nevertheless do make such mistakes.

ªScore Enhancerª 427.

Which of these is not one of the alternative theories / objections regarding the environmental damage done to Easter Island? 1) The damage was a result of natural causes. 2) Unknown European explorers caused the damage sometime before 1722. 3) A cool, dry period between 26,000 and 12,000 years ago was responsible for the extinction of the forests. 4) The damage could not have been self-inflicted, as the Easter Islanders would not have been foolish enough to cut down all their trees. 5) None of these.

428.

What is the author's main objection to the second alternative theory? 1) There is no proof that climate change contributed to deforestation on Easter Island. 2) Climate change is unlikely to cause complete deforestation and bird extinc tions as seen on Easter Island. 3) It is too much of a coincidence to assume that Easter Island trees went extinct only after the arrival of humans. 4) There is evidence that cool, dry periods cause trees to grow on lower altitudes on Easter Island, not go completely extinct. 5) Given that Easter Island tree species had already survived many climate changes, it is unlikely that these changes were the sole cause of their eventual extinction.

95

Reading Comprehension ªScore Maximiserª 429.

What is the author's opinion of Easter Islanders? 1) He thinks that they ruined their own environment, due to unknown reasons. 2) He thinks that they knowingly ruined their own environment, and then blamed the Europeans for it. 3) He thinks they are bad people who deliberately ruined their own environment. 4) He thinks that they are nice people, who were nevertheless foolish enough to damage their own environment. 5) He considers them to be very nice people, who would never knowingly damage their environment.

ªScore Enhancerª 430.

Which of the following cannot be inferred about Easter Island? 1) 2) 3) 4) 5)

It was officially 'discovered' by Europeans in the 18th century. Mt. Terevaka and Poike Peninsula are places on Easter Island. By the year 1722, there were no forests on the island. Currently, no people live on Easter Is land. Plants like toromiro and tree daisies used to grow on it.

ªScore Maximiserª 431.

'Easter Island is only one of many other societies which have inadvertently damaged their own environment.' The author of this passage: 1) 2) 3) 4) 5)

would definitely agree with this statement. would agree, but with some qualifications. would not agree with this statement at all. does not have any opinion on this topic. may agree with this statement - nothing more certain can be inferred from the limited data in the passage.

People who believe that the mind can be replicated on a computer tend to explain the mind in terms of a computer. When theorizing about the mind, especially to outsiders but also to one another, defenders of artificial intelligence (AI) often rely on computational concepts. They regularly describe the mind and brain as the 'software and hardware' of thinking, the mind as a 'pattern' and the brain as a 'substrate,' senses as 'inputs' and behaviours as 'outputs,' neurons as 'processing units' and synapses as 'circuitry,' to give just a few common examples. Those who employ this analogy tend to do so with casual presumption. They rarely justify it by reference to the actual workings of computers, and they misuse and abuse terms that have clear and established definitions in computer science - established not merely because they are well understood, but because they in fact are products of human engineering. An examination of what this usage means and whether it is correct reveals a great deal about the history and present state of artificial intelligence research. And it highlights the aspirations of some of the luminaries of AI - researchers, writers, and advocates for whom the metaphor of mind-as-machine is dogma rather than discipline. Much artificial intelligence research has been based on the assumption that the mind has layers comparable to those of the computer. Under this assumption, the physical world, including the mind, is not merely understandable through sciences at increasing levels of complexity - physics, chemistry, biology, neurology and psychology - but is actually organized into these levels. Moreover, much work in AI has assumed that the layers of the mind and brain are separable from each other in the same manner that the computer is organized into many layers of abstraction, so that each layer can be understood on its own terms

96

Reading Comprehension without recourse to the principles of lower levels. These assumptions underlie the notion that the mind is a 'pattern' and the brain is its 'substrate.' If this notion is true, then the processes that give rise to the mind must consist of some basic rules and procedures implemented in the brain. The mind, then, is a program, and the brain is but a computer upon which the mind is running. In this understanding, the brain must contain some basic functional unit whose operations enable the implementation of the procedures of the mind. For those AI researchers interested in actually replicating the human mind, the two guiding questions have thus been (1) What organizational layer of the mind embodies its program? and (2) At what organizational layer of the brain will we find the basic functional unit necessary to run the mind-program? Especially for researchers who believe 'strong AI' is tenable - that is, those who believe that computers can be programmed to be intelligent, conscious and emotional - their aims and methods can be understood as a progression of attempts to answer these two questions. But when closely examined, the history of their efforts is revealed to be a sort of regression, as the layer targeted for replication has moved lower and lower. The earliest AI efforts aimed for the highest level, attempting to replicate the rules underlying reason itself. In 1955, Alan Newell and Herbert Simon created Logic Theorist, a program that modelled human problem-solving methods in order to prove mathematical theorems. When it successfully solved many of the theorems in Bertrand Russell's Principia Mathematica, Simon famously told students, 'Over Christmas, Al Newell and I invented a thinking machine.' The next year, leaders in the burgeoning field held the first AI conference in Dartmouth; their conference proposal asserted the foundational creed of AI: that 'every aspect of learning or any other feature of intelligence can be so precisely described that a machine can be made to simulate it.' Implicit in this statement are answers to the fundamental questions: (1) Reason itself, the highest level of the mind, embodies the mind's program, and (2) The basic functional unit of the brain is irrelevant. The approach of 'precisely describing' intelligence and learning was applied to a wide array of simple problems, meeting with great success until the late 1970s, at which time researchers began to realize that many seemingly simple problems - such as recognizing objects in an image, navigating a robot around an obstacle course, or understanding a children's story - could not be solved using methods that 'precisely described' intelligence and learning. Later research in image recognition has met with more success, while children's stories are still too difficult for computers, despite decades of effort.

ªScore Enhancerª 432.

In the passage the word 'substrate' means: 1) 2) 3) 4) 5)

A complementary organ The organ that executes the impulses sent by the mind The organ that provides the surface that implements the processes of the mind The organ that works separately from the mind None of these

In 1885, Czar Alexander III of Russia gave the czarina an Easter present that astonished her. When your wife lives in a palace with 900 rooms, delighting her with a gift is no easy task. And at first, Czarina Marie Fedorovna could have been excused for being underwhelmed by Alexander's offering: It was a plain enamelled egg - the traditional Russian Easter gift - 2½ inches high. Inside the egg, though, she found a yolk made of gold; inside that, an exquisite golden hen on a bed of golden straw; inside that, a miniature diamond crown; and inside that, a tiny ruby pendant. She'd never seen anything like it. No one had. It so captivated the family that its maker, Carl Gustavovich Fabergé, earned the right to display the royal seal.

97

Reading Comprehension Nearly every Easter thereafter, Alexander III gave a new Fabergé creation to Marie; when he died, his son Czar Nicholas II continued each spring to present an egg, one to his wife, Alexandra, and one to his mother. Between 1885 and 1916, 50 'imperial eggs' were made for the czars. Then the tradition ended: In 1917, eggs were the last thing on the mind of a czar worried about his very survival. Over the years the Fabergé eggs had grown more elaborate, each with a theme calculated to charm its recipient. When Marie's sons went to sea in 1890 on a Russian cruiser, the Azova Egg the following spring contained a gold and platinum replica of the ship floating on aquamarine, with diamonds for portholes. The Danish Palaces Egg from that period was so complex - featuring watercolour miniatures of palaces from the czarina's youth in Denmark - that it took a year to make. The 1913 Winter Egg, made of rock crystal and designed to imitate ice, was so perfectly etched and polished that it seemed freezing to the touch. Marie called Fabergé the 'greatest genius of our time.' (What did Tolstoy ever do for her?) Luscious though Fabergé eggs might be, Toby Faber's book Fabergé's Eggs is not just about glittering baubles. He argues that these objects have something to teach us about history. Noting, for instance, that Carl Fabergé borrowed designs from the court jewellers of Louis XV and XVI, Mr. Faber observes that the French and Russian dynasties, both coming to gory ends, had both also debauched themselves in riches. While the Romanovs were ogling the latest gem-encrusted Fabergé fantasy, their subjects were farming with medieval tools. But Mr. Faber wisely doesn't turn his story into a grim portrait of unfeeling excess. Instead, he tells a vivid, engrossing tale, describing, for instance, the rise and fall of Rasputin, the czarina's confidant, and giving a harrowing account of the final miserable months of Nicholas II and his family. We see them, prisoners in Siberia, hand-decorating Easter eggs that had been donated by sympathetic villagers, before being shot. In 1918, a year after the Russian Revolution, Lenin's commissars seized Fabergé's company and looted dozens of imperial eggs from the deposed czar's possessions. The communists sold the eggs abroad during the 1920s to raise desperately needed hard currency. Other eggs had been smuggled out of the country long before. Mr. Faber tracks these masterpieces as they turn up around the world, from Shanghai flea markets to the private collections of heiresses. It was investor-aficionado Malcolm Forbes who, in modern times, 'almost single-handedly' made the eggs into some of the world's most precious objects, Mr. Faber says. Between 1965 and 1985, bidding at auctions and striking private deals, Forbes purchased nine imperial eggs for ever-increasing sums. Mr. Faber wonders whether Fabergé's prestige today is in part the effect, not the cause, of the price that Mr. Forbes was willing to pay. Many people predicted that after Malcolm Forbes's death (in 1990) prices would collapse. They didn't. In 1994 the Winter Egg sold for $5.6 million; eight years later the emir of Qatar bought it for $9.6 million. Then, in 2004, the Forbes family engaged Sotheby's to put its Fabergé hoard - all nine eggs - up for sale. A Russian oil magnate named Viktor Vekselberg swooped in before the auction could even take place and bought the lot for $90 million. 'There's a satisfying symmetry,' Mr. Faber writes, 'inherent in the idea of eggs that were once ordered by the czar, the individual at the apex of an aristocratic society, being brought back to Russia by his modern-day successors, the oligarchs who now bestride the Russian economy.' Maybe Mr. Vekselberg will someday give his collection to a Russian museum - say, the famous one in St. Petersburg that was once an imperial palace - and the circle will be complete.

98

Reading Comprehension ªScore Enhancerª 433.

Which of the following is implicit in the author's parenthetical remark 'What did Tolstoy ever do for her?'? 1) 2) 3) 4) 5)

Tolstoy and Fabergé were rivals. Unlike Fabergé, Tolstoy was not a master jeweller. The general opinion is that Tolstoy was the greatest genius of Czarina Marie Fedorovna's time. Unlike Fabergé, Tolstoy had never created something impressive for the Czarina Marie Fedorovna. Both [3) and [4).

ªScore Maximiserª 434.

What is the author's opinion of Fabergé Eggs? 1) He considers them to be beautiful and fascinating, but also a symbol of the decadence of Russian royalty. 2) He considers them to be beautiful and fascinating, and lauds how Toby Faber ties them to the history of Russian monarchy. 3) He considers them to be beautiful and fascinating, and laments that Toby Faber's book on them goes off on irrelevant tangents. 4) He considers them to be beautiful and fascinating, and traces their history from the hands of Russian monarchs to their return to modern Russia. 5) While he considers them be beautiful and fascinating, he wonders whether their prestige is more a result of hype rather than their intrinsic worth.

435.

What does the author mean by the conclusion 'and the circle will be complete'? 1) Just as the Fabergé Eggs belonged to the czars, the rulers of Russia then, they are now owned by a Russian oligarch, a member of the group at the apex of Russian society now. 2) If the oligarch who now owns the Fabergé Eggs donates them to a Russian museum, they will have returned to the place they originated from. 3) Some of the Fabergé Eggs have returned to Russia; if all of them do, and if they are donated to the St. Petersburg Museum, they will have returned to their place of origin. 4) If some of the Fabergé Eggs wind up at the Russian palace that is now a museum, they will have returned to the place they started from, i.e., the czar's palace. 5) The czars originally commissioned the Fabergé Eggs as gifts, but their value has been increasing dramatically; if their current owner donates them, they will once again become gifts.

436.

Which of the following, if true, would weaken Mr. Faber's speculation whether Fabergé's prestige today is in part the effect, not the cause, of the price that Mr. Forbes was willing to pay for some of the eggs? 1) Fabergé Eggs were well known and admired throughout Europe right from the beginning. 2) Fabergé has always been famous for the masterly jewelled objects he created, even apart from the eggs. 3) The eggs were considered highly valuable long before Mr. Forbes bought them, though no one else could afford to pay the kind of prices he paid. 4) The eggs are made of many precious stones and metals, which are highly valuable in themselves, not to mention the highly prized craftsmanship that went into making the eggs. 5) All of the above.

99

Reading Comprehension Whether made of highly polished metal or of glass with a coating of metal on the back, mirrors have fascinated people for millennia: ancient Egyptians were often depicted holding hand mirrors. With their capacity to reflect back nearly all incident light upon them and so recapitulate the scene they face, mirrors are like pieces of dreams, their images hyper-real and profoundly fake. Mirrors reveal truths you may not want to see. Give them a little smoke and a house to call their own, and mirrors will tell you nothing but lies. 'In a sense, mirrors are the best "virtual reality" system that we can build,' said Marco Bertamini of the University of Liverpool. 'The object "inside" the mirror is virtual, but as far as our eyes are concerned it exists as much as any other object.' Dr. Bertamini and his colleagues have also studied what people believe about the nature of mirrors and mirror images, and have found nearly everybody, even students of physics and math, to be shockingly off the mark. In a series of studies, Dr. Bertamini and his colleagues have interviewed scores of people about what they think the mirror shows them. They have asked questions like, Imagine you are standing in front of a bathroom mirror; how big do you think the image of your face is on the surface? And what would happen to the size of that image if you were to step steadily backward, away from the glass? People overwhelmingly give the same answers. To the first question they say, well, the outline of my face on the mirror would be pretty much the size of my face. As for the second question, that's obvious: if I move away from the mirror, the size of my image will shrink with each step. Both answers, it turns out, are wrong. Outline your face on a mirror, and you will find it to be exactly half the size of your real face. Step back as much as you please, and the size of that outlined oval will not change: it will remain half the size of your face (or half the size of whatever part of your body you are looking at), even as the background scene reflected in the mirror steadily changes. Importantly, this half-size rule does not apply to the image of someone else moving about the room. If you sit still by the mirror, and a friend approaches or moves away, the size of the person's image in the mirror will grow or shrink as our innate sense says it should. What is it about our reflected self that it plays by such counterintuitive rules? The important point is that no matter how close or far we are from the looking glass, the mirror is always halfway between our physical selves and our projected selves in the virtual world inside the mirror, and so the captured image in the mirror is half our true size. Dr. Bertamini's colleague Rebecca Lawson suggests imagining that you had an identical twin, that you were both six feet tall and that you were standing in a room with a movable partition between you. How tall would a window in the partition have to be to allow you to see all six feet of your twin? The window needs to allow light from the top of your twin's head and from the bottom of your twin's feet to reach you, Dr. Lawson said. These two light sources start six feet apart and converge at your eye. If the partition is close to your twin, the upper and lower light points have just begun to converge, so the opening has to be nearly six feet tall to allow you a full-body view. If the partition is close to you, the light has nearly finished converging, so the window can be quite small. If the partition were halfway between you and your twin, the aperture would have to be - three feet tall. Optically, a mirror is similar, Dr. Lawson said, 'except that instead of lighting coming from your twin directly through a window, you see yourself in the mirror with light from your head and your feet being reflected off the mirror into your eye.' This is one partition whose position we cannot change. When we gaze into a mirror, we are all of us tethered eternally to our doppelganger on the other side.

100

Reading Comprehension ªScore Enhancerª 437.

Why does Dr. Bertamini call mirrors 'virtual reality systems'? 1) 2) 3) 4) 5)

438.

Because they accurately simulate reality. Because they depict reality in counterintuitive ways. Because the images they depict are more 'realistic' than reality itself. Because they can fool our eyes into thinking that what they show is real. Because they depict reality, whereas electronic virtual reality systems do not.

What does the word 'doppelganger' signify in the last sentence of the passage? 1) A ghostly apparition 3) An unreal double of a person 5) A sinister omen in human form

439.

2) A clone of a human being 4) An evil counterpart of a person

The twin example is used as an analogy for the mirrors in order to: 1) show that basically a mirror is nothing but a partition between 'two people' who look the same. 2) show why you can see the whole of your twin through the hole in the partition between the two of you. 3) show that the size of an opening in the partition between you and your twin should be exactly half your size. 4) show why the image of someone the same size as you would be half your size on a partition halfway in between. 5) show that the size of the hole in the partition between you and your twin varies depending on how close it is to you.

ªScore Maximiserª 440.

Suggest a suitable title for the passage. 1) Misconceptions about Mirrors 3) Mirrors don't Lie 5) A House of Smoke and Mirrors

2) The Mystery of Mirrors 4) The Man in the Mirror

As a young man I enjoyed listening to a particular series of French instructional programmes. I didn't understand a word, but was nevertheless enthralled. Was it because the sounds of human speech are thrilling? Not really. Speech sounds alone, stripped of their meaning, don't inspire. We don't wake up to alarm clocks blaring German speech. We don't drive to work listening to native spoken Eskimo. Speech sounds don't give us the chills, and they don't make us cry. But music does emanate from our alarm clocks in the morning, and fill our cars, and give us chills, and make us cry. According to a recent paper by Nidhya Logeswaran and Joydeep Bhattacharya from the University of London, music even affects how we see visual images. In the experiment, 30 subjects were presented with a series of happy or sad musical excerpts. After listening to the snippets, the subjects were shown a photograph of a face. Some people were shown a happy face - the person was smiling - while others were exposed to a sad or neutral facial expression. The participants were then asked to rate the emotional content of the face on a 7-point scale, where 1 mean extremely sad and 7 extremely happy. The researchers found that music powerfully influenced the emotional ratings of the faces. Happy music made happy faces seem even happier while sad music exaggerated the melancholy of a frown. A similar effect was also observed with neutral faces. The simple moral is that the emotions of music are 'crossmodal', and can easily spread from one sensory system to another. Although it probably seems obvious that music can evoke emotions, it is to this day not clear why. Why doesn't music feel like listening to speech sounds, or animal calls, or garbage disposals? Why

101

Reading Comprehension is music nice to listen to? Why does music get blessed with a multi-billion dollar industry, whereas there is no market for 'easy listening' speech sounds? In an effort to answer, let's ask why I was listening to French instructional programmes in the first place. The truth is, I wasn't just listening. I was watching them on public television. What kept my attention was not the meaningless-to-me speech sounds, but the young French actress. Her hair, her smile, her mannerisms, her pout… I digress. The show was a pleasure to watch because of the humans it showed, especially the exhibited expressions and behaviours. Music is exquisitely emotionally evocative, which is why a touch of happy music makes even unrelated pictures seem more pleasant. It would be plausible to conclude that the artefact of music should contain some distinctly human elements. The question, of course, is what those elements are. One candidate is our expressive speech - perhaps music is just an abstract form of language. However, most of the emotion of language is in the meaning, which is why foreign languages that we don't understand rarely make us swoon with pleasure or get angry. That's also why emotional speech from an unfamiliar language isn't featured on the radio! But there is a second auditory expressive behaviour we humans carry out - our bodily movements themselves. Human movement has been conjectured to underlie music as far back as the Greeks. As a hypothesis this has the advantage that we have auditory systems capable of making sense of the sounds of people moving in our midst. Some of these movements trigger positive emotions - they conjure up images of pleasant activities - while others might be automatically associated with fear or anxiety. (The sound of running makes us wonder what we're running from.) If music were speech-driven, then it is missing out on the largest part of speech's expressiveness - the meaning. But if music sounds like human expressive movements, then it sounds like something that, all by itself, is rich in emotional expressiveness, and can be easily interpreted by the auditory system. Regardless of whether music is emotional intonation from speech or a summary of expressive movements - or something else altogether - the new research by Logeswaran and Bhattacharya adds yet more fuel to the expectation that music has been culturally selected to sound like an emotionally expressive human. While it is not easy for us to see the human ingredients in the modulations of pitch, intensity, tempo and rhythm that make music, perhaps it is obvious to our auditory homunculus.

ªScore Maximiserª 441.

According to the passage, which of the following factors definitely influence the way in which an individual might perceive a particular song or piece of music? 1) His or her familiarity with the language in which the song is sung. 2) His or her emotional state at the time the song is sung or the music is played. 3) The emotion that is sought to be expressed in the music by its composer. 4) All of the above. 5) Only [1) and [2).

ªScore Enhancerª 442.

The point that the author wishes to make in the second and third paragraphs is that: 1) the evocative nature of music enhances the emotional colouring of events or pictures that may otherwise seem commonplace. 2) because music is so emotionally evocative, it enables the listener to comprehend deeper meanings in pictures or events. 3) music makes us emotional in such a way that we tend to see meanings where there are none.

102

Reading Comprehension 4) music distorts our ability to think clearly and look at things objectively. 5) music has the ability to influence our perceptions of things, but there is little correlation between the mood of a piece and that of a person. DIRECTIONS for questions 54 to 56: Five alternative summaries are given below each text. Choose the option that best captures the essence of the text.

ªScore Maximiserª 443.

Various countries have set standards in legislation of emissions in the form of concentration levels that are believed to be low enough to protect public health. Source emission standards are also specified to limit the discharge of pollutants into the air, so that air-quality standards will be achieved. However, the nature of the problem requires the implementation of international environmental treaties, and to this end 49 countries agreed in March 1985 on a United Nations convention to protect the ozone layer. This 'Montreal Protocol', which was renegotiated in 1990, calls for the phaseout of certain chlorocarbons and fluorocarbons by the end of the century and provides aid to developing countries in making this transition. In addition, several international protocols have been aimed specifically at reducing the incidence of acid rain. 1) Various countries have set up legislative standards to specify concentration limits and emission standards in the interest of public health. Furthermore, several international treaties have been signed for environmental protection, like the Montreal Protocol signed at a UN Convention. 2) Taking the cue from the signing of several international treaties like the Montreal Protocol, many countries are now setting up legislative standards to deal with environmental pollution by specifying concentration and emission limits in a bid to protect public health. 3) Various countries have initiated legislation standards to curb environmental pollution including setting limits for concentration and emission levels, besides signing environmental treaties, and in this regard a UN Convention against ozone layer depletion has been signed, apart from protocols such as the Montreal Protocol. 4) Environmental protection measures are necessary to stop ozone layer depletion and acid rain, and in this regard a lot of commendable work is being done both by the governments of individual countries and by the United Nations through the signing of various protocols and legislation. 5) Various countries have now set up effective legal standards to counter environmental problems such as source emissions, ozone layer depletion and acid rain. Through facilitating the signing of protocols at the international level, the United Nations has also indicated its commitment to environmental protection.

444.

Even though no less a figure than Neil Armstrong has been quoted recently as saying that the challenges to land on Mars are not as difficult as the Apollo pioneers faced, a manned mission to Mars seems beyond the reach of affordable technology and political will. What was possible in the 1960s to a technically less advanced generation may be out of reach to this and future generations, simply because of the change in attitudes and priorities and the vast increase in the thresholds of risk that are now seen as acceptable. This is one overhead that the Apollo programme did not have to face. Apollo could not be reproduced in the early 21st century, simply because the world we live in is so utterly different from that of only 40 or 50 years ago; sights have lowered as fears have increased. 1) In spite of the insistence of Neil Armstrong, the first human being to land on the moon, landing on Mars has now become an unaffordable proposition. Scientific attitudes have changed, and man has become fearful of the universe. 2) Neil Armstrong may think differently, but landing on Mars seems an improbable proposition

103

Reading Comprehension today. Attitudes and priorities have changed and the fears of the modern world have lowered ambitions. 3) Neil Armstrong may think differently, but the achievement of the Apollo pioneers cannot be replicated today by a landing on Mars, a technologically and politically unaffordable option, as attitudes have changed and caution has lowered ambitions. 4) Neil Armstrong insists that a landing on Mars is possible today, but scientists today are more cautious than they were when Apollo was launched. We are now in the 21st century and such a landing has become unviable due to political and technological constraints. 5) Neil Armstrong's assertion that a Mars landing is possible today has been met with stark criticism from the scientific community, which regards such a mission impossible due to technological and political constraints. 445.

Over the past century, violent images from World War II concentration camps, Cambodia, Rwanda, Darfur, Iraq, and many other times and places have been seared into our collective consciousness. These images have led to a common belief that technology, centralized nation-states, and modern values have brought about unprecedented violence. But now that social scientists have started to count bodies in different historical periods, they have discovered that the romantic theory gets it backward: far from causing us to become more violent, something in modernity and its cultural institutions has made us nobler. In fact, our ancestors were far more violent than we are today. Indeed, violence has been in decline over long stretches of history, and today we are probably living in the most peaceful moment of our species' time on earth. 1) Images of violence from across the modern world have led to a belief that modern progress, systems and values are responsible for this violence. However, social scientists have already discovered that it is quite the opposite, and that our ancestors were more violent than we are. 2) Images of violence from the modern world have fostered a belief that modern progress and value systems are responsible for this, whereas the truth is quite opposite, as is being discovered by social scientists. 3) That the outcome of modernity has been unprecedented violence is an erroneous notion created through grisly imagery. Social scientists are now discovering that our ancestors were more violent than we are, and violence has been on the decline through the ages. 4) Grisly images have fooled us into believing that the modern world is terribly violent. While violence in the modern world is on the rise, our ancestors were no saints in this regard. 5) Images of gory violence have deliberately been implanted into our minds by those who want to make us believe in the decadence of the modern world, thus making us blind to the atrocities committed by our ancestors.

DIRECTIONS for questions 446 to 506: The passages given below is followed by a set of questions. Choose the best answer to each question. What is an Art Museum for? As the word "Curator" implies, the first and most essential function of such a Museum is to take care of ancient or unique works of art which are no longer in their original places or no longer used as was originally intended, and are therefore in danger of destruction by neglect or otherwise. This care of works of art does not necessarily involve their exhibition. If we ask, why should the protected works of art be exhibited and made accessible and explained to the public, the answer will be made, that this is to be done with an educational purpose. But before we proceed to a consideration of this purpose, before we ask, Education in or for what? a distinction

104

Reading Comprehension must be made between the exhibition of the works of living artists and that of ancient or relatively ancient or exotic works of art. It is unnecessary for Museums to exhibit the works of living artists, which are not in imminent danger of destruction; or at least, if such works are exhibited, it should be clearly understood that the Museum is really advertising the artist and acting on behalf of the art dealer or middleman whose business it is to find a market for the artist; the only difference being that while the Museum does the same sort of work as the dealer, it makes no profit. On the other hand, that a living artist should wish to be "hung" or "shown" in a Museum can be only due to his need or his vanity. For things are made normally for certain purposes and certain places to which they are appropriate, and not simply "for exhibition"; and because whatever is thus custom-made, i.e., made by an artist for a consumer, is controlled by certain requirements and kept in order. Whereas, as Mr. Steinfels has recently remarked, "Art which is only intended to be hung on the walls of a Museum is one kind of art that need not consider its relationship to its ultimate surroundings. The artist can paint anything he wishes, any way he wishes, and if the Curator and Trustees like it well enough they will line it up on the wall with all the other curiosities." We are left with the real problem, Why exhibit? as it applies to the relatively ancient or foreign works of art which, because of their fragility and because they no longer correspond to any needs of our own of which we are actively conscious, are preserved in our Museums, where they form the bulk of the collections. If we are to exhibit these objects for educational reasons, and not as mere curios, it is evident that we are proposing to make such use of them as is possible without an actual handling. It will be imaginatively and not actually that we must use the mediaeval reliquary, or lie on the Egyptian bed, or make our offering to some ancient deity. The educational ends that an exhibition can serve demand, accordingly, the services not of a Curator only, who prepares the exhibition, but of a Docent who explains the original patron's needs and the original artists' methods; for it is because of what these patrons and artists were that the works before us are what they are. If the exhibition is to be anything more than a show of curiosities and an entertaining spectacle it will not suffice to be satisfied with our own reactions to the objects; to know why they are what they are we must know the men that made them. It will not be "educational" to interpret such objects by our likes or dislikes, or to assume that these men thought of art in our fashion, or that they had aesthetic motives, or were "expressing themselves." We must examine their theory of art, first of all in order to understand the things that they made by art, and secondly in order to ask whether their view of art, if it is found to differ from ours, may not have been a truer one.

ªScore Enhancerª 446.

Which of the following does the author mean when he says that a living artist exhibits his work only to satisfy his need or vanity? 1) A living artist exhibits his work either to make money or to showoff his talents. 2) A living artist exhibits his work in order to be critically evaluated by an audience. 3) A living artist exhibits his works to know the monetary and artistic worth of his creations. 4) A living artist exhibits his work to know where he stands with respect to his con temporaries.

447.

Which of the following can be a suitable title for the passage? 1) What is the function of art? 3) Why study works of art?

2) Why exhibit works of art? 4) Should living artists exhibit their art?

105

Reading Comprehension ªScore Maximiserª 448.

According to the author in order to understand a work of art from another era or culture we should I. II. III. 1)

understand the art theory of that era. understand the motivations of the artists and the patrons of that era. understand their relevance in contempo rary times. Only I 2) Only II 3) Both I and II 4) I, II and III

The United Nations General Assembly has described intervention as dictatorial interference in the political independence and territorial integrity of a sovereign state. Traditionally, intervention was prohibited by international law. Non-intervention has been an especially important principle for liberal statesmen and moralists with a commitment to universal human rights. On the one hand, Liberals have provided some of the strongest reasons to abide by a strict form of the non-intervention doctrine. On the other hand, those same principles of universal human dignity have provided justifications for overriding the principle of non-intervention. Arguments against intervention have taken the form of both direct principles and indirect, or procedural, considerations. The most important direct consideration for the liberals was that non-intervention reflected and protected human dignity. According to Kant, non-intervention could enable a polity the necessary territorial space and political independence in which free and equal citizens could work out what their own way of life would be. A free government achieved by means of intervention would not be authentic or self-determining but determined by others and not one that local citizens had themselves defined through their own actions. Secondly, as argued by John Stuart Mill, intervention undermines the authenticity of domestic struggles for liberty. A people given freedom by a foreign intervention would not, he argued, be able to hold on to it. Intervention, therefore, would have produced not freedom and progress, but a civil war with all its attendant violence or the interveners would have to continually send in foreign support. A third argument against intervention was difficulties of transparency. Historically, it has proven difficult to identify authentic "freedom fighters." Particular national regimes of liberty and oppression are difficult for foreigners to "unpack," reflecting, sometimes, complicated historical compromises and contracts of a Burkean sort among the dead, the living and the yet to be born. Michael Walzer, as did Mill, acknowledges that sovereignty and non-intervention ultimately depend upon consent. If the people welcome an intervention, or refuse to resist, something less than aggression has occurred. But since we cannot make those judgments reliably in advance, domestic revolutions need to be left to domestic citizens. Fourth, the necessary "dirty hands" of violent means often become "dangerous hands" in international interventions. International history is rife with interventions justified by high-sounding principles ending the slave trade or suttee or introducing law and order and civilized behaviour - turning into self-serving, imperialist "rescues" in which the intervener stays to profit and control. Requiring that the intervener be impartial, looking for something more than a unilateral decision, and respecting the multilateral processes of international law are important procedural considerations in weighing the justice of an intervention.

106

Reading Comprehension The indirect reasons for non-intervention are the rules of international law among sovereign civilized states prohibiting intervention. These rules were painstakingly achieved compromises among diverse moralities. The mere process of achieving consent made them legitimate.

ªScore Enhancerª 449.

Which of the following is the author primarily concerned with? 1) To show that the very arguments in favour of non-intervention in the political in dependence and territorial integrity of a sovereign state hold true for intervention as well. 2) To outline the various arguments in favour of non-intervention in the political independence and territorial integrity of a sovereign state. 3) To highlight the importance of Kant and Mill in international laws on non-intervention in the political independence and territorial integrity of a sovereign state. 4) To argue in favour of non-intervention in the political independence and territorial integrity of a sovereign state.

ªScore Maximiserª 450.

Which of the following can be inferred from the third argument in favour of non-intervention? I. Historically, within a country, there have been different groups fighting for its liberation. II. Over a period of time, groups fighting for a country's liberation might resort to actions that differ in values from those of their founding fathers. III. Struggles for liberation involve compromises between foreigners, freedom fighters and oppressors. 1) Only I 2) Only II 3) Both I and II 4) I, II and III

451.

Which of the following CANNOT be inferred from the indirect reasons that the author states in favour of non-intervention? I.

Many countries that have agreed to current international laws prohibiting intervention are not in favour of them. II. Not all countries that have agreed to current international laws prohibiting intervention are in favour of them. III. Framing international laws that govern many countries is tough and involves a lot of compromise. 1) Only I 2) Only II 3) Only III 4) Both I and III

In the early Platonic dialogue, Crito, Socrates makes a compelling argument as to why he must stay in prison and accept the death penalty, rather than escape and go into exile in another Greek city. He personifies the Laws of Athens, and, speaking in their voice, explains that he has acquired an overwhelming obligation to obey the Laws because they have made his entire way of life, and even the fact of his very existence, possible. They made it possible for his mother and father to marry, and therefore to have legitimate children, including himself. Having been born, the city of Athens, through its laws, then required that his father care for and educate him. Socrates' life and the way in which that life has flourished in Athens are each dependent upon the Laws. Importantly, however, this relationship between citizens and the Laws of the city are not coerced. Citizens, once they have grown up, and have seen how the city conducts itself, can choose whether to leave, taking their property with them, or stay. Staying implies an agreement to abide by the Laws and accept the punishments that they mete out. And, having made an agreement that is itself just, Socrates asserts that he must keep to this agreement that he has made and obey the Laws, in this case, by staying and accepting the death penalty. Importantly, the contract described by Socrates is an implicit one: it is implied by his choice to stay in Athens, even though he is free to leave.

107

Reading Comprehension In Plato's most well-known dialogue, Republic, social contract theory is represented again, although this time less favourably. In Book II, Glaucon offers a candidate for an answer to the question "what is justice?" by representing a social contract explanation for the nature of justice. What men would most want is to be able to commit injustices against others without the fear of reprisal, and what they most want to avoid is being treated unjustly by others without being able to do injustice in return. Justice then, he says, is the conventional result of the laws and covenants that men make in order to avoid these extremes. Being unable to commit injustice with impunity (as those who wear the ring of Gyges would), and fearing becoming victims themselves, men decide that it is in their interests to submit themselves to the convention of justice. Socrates rejects this view, and most of the rest of the dialogue centres on showing that justice is worth having for its own sake, and that the just man is the happy man. So, from Socrates' point of view, justice has a value that greatly exceeds the prudential value that Glaucon assigns to it. These views, in the Crito and the Republic, might seem at first glance inconsistent: in the former dialogue Socrates uses a social contract type of argument to show why it is just for him to remain in prison, whereas in the latter he rejects social contract as the source of justice. These two views are, however, reconcilable. From Socrates' point of view, a just man is one who will, among other things, recognize his obligation to the state by obeying its laws. The state is the morally and politically most fundamental entity, and as such deserves our highest allegiance and deepest respect. Just men know this and act accordingly. Justice, however, is more than simply obeying laws in exchange for others obeying them as well. Justice is the state of a well-regulated soul, and so the just man will also necessarily be the happy man. So, justice is more than the simple reciprocal obedience to law, as Glaucon suggests, but it does nonetheless include obedience to the state and the laws that sustain it. So in the end, although Plato is perhaps the first philosopher to offer a representation of the argument at the heart of social contract theory, Socrates ultimately rejects the idea that social contract is the original source of justice.

ªScore Maximiserª 452.

Which of the following is the primary purpose of the passage? 1) 2) 3) 4)

453.

To To To To

prove that the social contract was better understood by Socrates than by Plato. demonstrate how Socrates repudiated Plato's position on the origins of justice. reconcile the contradictions in the social contract as represented by Plato. evaluate the representation of the social contract in the Platonic dialogues.

Which of the following best captures the difference between Glaucon's and Socrates' conception of justice? 1) The former viewed it as arising from the point of view of a selfish man while the latter viewed it as arising from the point of view of a just man. 2) The former viewed it as arising out of man's innate self-interest while the latter viewed it as something that is to be sought for its own intrinsic worth. 3) The former viewed is as a way of curbing the extremes while the latter viewed it as a way of reaching the sublime. 4) The former viewed it as arising out of the social contract while the latter thought it upended the social contract.

108

Reading Comprehension 454.

Which of the statements represent Socrates' view of the relationship between justice and the social contract? I. II. III. 1) 3)

455.

The social contract arises out of one of the many obligations that a just, self-regulated man will undertake. Justice does not arise from the social contract; rather the social contract arises out of justice. Justice is one of the attributes only of a well-regulated soul and is a part of the social contract. Only I 2) Only II Both I and III 4) I, II and III

Which of the following pairs of words best characterizes the representation of the social contract in the Crito and the Republic, respectively? 1) Implicit, Explicit 3) Exalted, Utilitarian

2) Principled, Agnostic 4) Subjective, Objective

Hindsight bias—the irrational belief that past outcomes were predictable—is a well-understood psychological phenomenon. Our research suggests that this bias is becoming stronger, thanks largely to an abundance of visual information, including re-creations and simulations. But in measuring it, we’ve also discovered its near opposite, what we call the propensity effect: Visualization may also, in certain circumstances, make people hyper-confident of impending events’ outcomes. Study participants were presented with traffic situations: some received a text description with diagrams, and others watched a computer animation. The amounts and types of information varied within each group. Some people examined normal traffic conditions; others saw or read about a driver error but not the resulting accident; still others saw or read about the driver error and the resulting accident. Hindsight bias more than doubled for the subjects who watched the computer animation. The propensity effect was significantly greater for those who watched the driver error but not the accident: They were more likely to say they could see a serious accident coming than those who actually saw it occur and then were asked if they had seen it coming. You experience the propensity effect when, say, a baseball that’s hit hard gives you that momentary feeling of “just knowing” it’s going out of the park. People misattribute visual processing of motion to higher-order judgments, such as predicting outcomes. So far, propensity has been tested only in relation to trajectory events (cars heading toward an accident), but movement seems to be a key factor in sparking the effect. When we gave subjects still photos of the same traffic situations that they could page through at their own pace—in effect making flipbook animations—the propensity effect wasn’t present. Computer-animated visualization is appealing because it can help make sense of highly complex information, but it’s also, quite literally, a point of view. The information can be conveyed with certain emphases, shown from certain angles, slowed down, or enlarged. (In a sense, all this is true of text as well, but with subtler effects.) Animations can whitewash the guesswork and assumptions that go into interpreting reconstructions. By creating a picture of one possibility, they make others seem less likely, even if they’re not. When an objective reading of evidence is critical—as it is in a courtroom and in many business contexts— both the deepening hindsight bias and the propensity effect can be pernicious. A manager with the tools to animate financial data sets, such as sales forecasts, can easily—on the basis of the story the visualization tells—misidentify trends, place blame where it doesn’t belong, or become overconfident about an action plan. Or a sophisticated visualization of customer feedback data might make it appear, for example,

109

Reading Comprehension that a new smartphone had tanked because it lacked user-friendliness, though pricing and availability were also responsible. A false sense of certainty about what went right or wrong can make managers unwilling to consider strategic alterations or search for new ways to attack problems. All of this is not to suggest doing away with animations. In many cases, they really help penetrate complexity. But further research is needed to understand the consequences of using them to depict data and to know how to control for variables.

ªScore Enhancerª 456.

Which of the following is implied about the effect of movement on propensity effect? I. Trajectory events tend to trigger people to predict outcomes more than stationary events. II. Trajectory events exaggerate people’s sense of sense of certainty about their predictions. III. Trajectory events tend to make people overestimate the actual distance that the object in motion will travel 1) Both I and II 2) Both II and III 3) Both I and III 4) I, II and III

457.

Which of the following does the passage suggest about the effect of computer-animated visualization on our ability to interpret complex data? 1) Computer-animated visualizations can be misleading since they lack the sophisti cation to take subtler effects into consideration thus limiting our interpretation of complex data. 2) Computer-animated visualization may bias our interpretation of complex data in one particular direction at the expense of others. 3) Computer animated-visualizations are better used in situations that demand a sub-jective reading than in those that demand an objective reading. 4) Computer-animated visualizations are less applicable in business scenarios when compared to others since there is usually more than one variable to influencing a situation.

ªScore Maximiserª 458.

The main purpose of the passage is to 1) Explain the difference between hindsight bias and propensity effect as experienced by human beings. 2) Outline the effects of visualization on the human tendency to predict outcomes of or reasons behind events. 3) Argue for the need for more research on the effect of using animations to depict and analyze data in business situations. 4) Question the relevance of using animation to depict and interpret data by pointing out their effect on hindsight bias and propensity effect.

110

Reading Comprehension The symbolic content of an art is originally bound up with its practical function, but is not necessarily lost when under changed conditions the art is no longer practiced of necessity but as a game or sport; and even when such a sport has been completely secularized and has become for the profane a mere recreation or amusement it is still possible for whoever possesses the requisite knowledge of traditional symbolism to complete this physical participation in the sport, or enjoyment of it as a spectacle, by an understanding of its forgotten significance, and so restore, for himself at least, the “polar balance of physical and metaphysical” that is characteristic of all traditional cultures. The position of archery in Turkey, long after the introduction of firearms had robbed the bow and arrow of their military value, provides us an excellent example of the ritual values that may still inhere in what to a modern observer might appear to be a “mere sport”. Here archery had become already in the fifteenth century a “sport” under royal patronage, the sultans themselves competing with others in the “field”. In the sixteenth century, at the circumcision festivals of the sons of Muhammad II, competing archers shot their arrows through iron plates and metal mirrors, or shot at valuable prizes set up on high posts: the symbolisms involved are evidently those of “penetration”, and that of the attainment of solar goods not within the archer’s direct reach; we may assume that, as in India, the “doctrine” implied an identification of the archer himself with the arrow that reached its mark. The Indian literature contains an almost embarrassing wealth of matter in which the symbolic values of archery are conspicuous. The penetration of obstacles is a common feat as it has been noted above in Turkish practice. An archer performs the difficult feat of piercing a hundred planks at a distance of some fifty yards; the archer is blindfolded and mounted on a moving wheel; when it comes round so that he faces the target, the cue is given by the sound of a blow struck on the target with a stick; and guided by the sound, he lets fly and pierces all the planks. The archer represents the “Gnosis of the Way”, while the given cue is that of “Adoptive Gnosis” and can be regarded as a “reminder” of the end to be reached; the bundle of planks signifies the “trunks or aggregates of greed, ill-will and delusion”; the “intention” or “aim” is Nirvâna. The literature on the practice of archery as a “sport” in Japan shows little this “sport” has the character of mere recreation that the notion of sport implies in secular cultures. The consummation of shooting is in the release; the Stance, Preparation, Posture, Raising the Bow, Drawing, and Holding, all these are but preparatory activities. The actual release of the arrow, like that of the contemplative, takes place suddenly indeed, but almost unawares, is spontaneous, and as it were uncaused. If all the preparations have been made correctly, the arrow, like a homing bird, will find its own goal; just as the man who, when he departs from this world “all in act”, having done what there was to be done, need not wonder what will become of him nor where he is going, but will inevitably find the bull’s eye, and passing through that sun door, enter into the empyrean beyond the “murity” of the sky. Thus Japanese archery is more than a “sport” in the Western sense; it belongs to Bushido, the Way of the Warrior. Further, the Seven Ways are based upon spontaneous principles, and not upon mere reasoning. Thus one sees how in a traditional society every necessary activity can be also the Way, and that in such a society there is nothing profane; a condition the reverse of that to be seen in secular societies, where there is nothing sacred. We see that even a “sport” may also be a yoga, and how the active and contemplative lives, outer and inner man can be unified in a single act of being in which both selves cooperate.

111

Reading Comprehension ªScore Maximiserª 459.

Which of the following best captures the meaning of the word secular as used in the passage? 1) Not connected to religion 3) Equally concerned about all religions

2) Impartial to all religions 4) Lacking value systems

Physicists at the Fermi National Accelerator Laboratory are reporting that they have discovered a new clue that could help unravel one of the biggest mysteries of cosmology: why the universe is composed of matter and not its evil-twin opposite, antimatter. If confirmed, the finding portends fundamental discoveries at the new Large Hadron Collider outside Geneva, as well as a possible explanation for our own existence. In a mathematically perfect universe, we would be less than dead; we would never have existed. According to the basic precepts of Einsteinian relativity and quantum mechanics, equal amounts of matter and antimatter should have been created in the Big Bang and then immediately annihilated each other in a blaze of lethal energy, leaving a big fat goose egg with which to make stars, galaxies and us. And yet we exist, and physicists (among others) would dearly like to know why. Sifting data from collisions of protons and antiprotons at Fermilab’s Tevatron, which until last winter was the most powerful particle accelerator in the world, the team, known as the DZero collaboration, found that the fireballs produced pairs of the particles known as muons, which are sort of fat electrons, slightly more often than they produced pairs of anti-muons. So the miniature universe inside the accelerator went from being neutral to being about 1 percent more matter than antimatter. This result may provide an important input for explaining the matter dominance in our universe. It was Andrei Sakharov, the Russian dissident and physicist, who first provided a recipe for how matter could prevail over antimatter in the early universe. Among his conditions was that there be a slight difference in the properties of particles and antiparticles known technically as CP violation. In effect, when the charges and spins of particles are reversed, they should behave slightly differently. Over the years, physicists have discovered a few examples of CP violation in rare reactions between subatomic particles that tilt slightly in favor of matter over antimatter, but not enough to explain our existence. The new effect hinges on the behavior of particularly strange particles called neutral B-mesons, which are famous for not being able to make up their minds. They oscillate back and forth trillions of times a second between their regular state and their antimatter state. As it happens, the mesons, created in the proton-antiproton collisions, seem to go from their antimatter state to their matter state more rapidly than they go the other way around, leading to an eventual preponderance of matter over antimatter of about 1 percent, when they decay to muons. Though the situation is fairly encouraging, whether this is enough to explain our existence is a question that cannot be answered until the cause of the still-mysterious behavior of the B-mesons is directly observed. The observed preponderance is about 50 times what is predicted by the Standard Model, the suite of theories that has ruled particle physics for a generation, meaning that whatever is causing the B-meson to act this way is “new physics” that physicists have been yearning for almost as long. The most likely explanations were some new particle not predicted by the Standard Model or some new kind of interaction between particles. Luckily, this is something we should be able to poke at with the Large Hadron Collider.

112

Reading Comprehension Nevertheless, physicists will be holding their breath until the results are confirmed by other experiments. Joe Lykken, a theorist at Fermilab, said, “So I would not say that this announcement is the equivalent of seeing the face of God, but it might turn out to be the toe of God.”

ªScore Enhancerª 460.

Which of the following is true based on the information given in the passage? I. II. III. IV.

B-mesons precede muons Muons precede B-mesons B-mesons are formed from muons Muons are formed from B-mesons

1) Only II 461.

2) Only III

3) I and III

4) I and IV

Which of the following best represents Joe Lykken’s analogy in the last line of the passage? 1) While we he exists. 2) While we exists. 3) The sight 4) The sight

might not as yet get to see how God looks, we at least now know for sure that might never get to see how God looks, we at least now know for sure that he of God’s toe does not prove the existence of God. of God’s toe provides an illusion of the existence of God.

ªScore Maximiserª 462.

According to the passage, why do the new discoveries at the Large Hadron Collider cast doubts over the Standard Model? I.

The new discoveries have found the preponderance of matter over anti-matter to be 50 times more than that predicted by the Standard Model. II. The new discoveries have found a new particle that was not predicted by the Standard Model. III. The new discoveries have found new particle interactions that were not predicted by the Standard Model. 1) Only I

2) Only II

3) Only III

4) I, II and III

Long before “sustainable” became a buzzword, intellectuals wondered how long industrial society could survive. After surveying predictions from the mid-19th century until today, the historian Arthur Herman, identifies two consistently dominant schools of thought. The first school despairs because it foresees inevitable ruin. The second school is hopeful — but only because these intellectuals foresee ruin, too, and can hardly wait for the decadent modern world to be replaced by one more to their liking. Every now and then, someone comes along to note that society has failed to collapse and might go on prospering, but the notion is promptly dismissed in academia as happy talk from a simpleton. What made Homo sapiens so special? It wasn’t our big brain, because Neanderthals had a big brain, too. Nor was it our willingness to help one another, because apes and other social animals also had an instinct for reciprocity. At some point, after millions of years of indulging in reciprocal back scratching of gradually increasing intensity, one species, and one alone, stumbled upon an entirely different trick. Adam gave Oz an object in exchange for a different object. The evidence for this trick is in perforated seashells from more than 80,000 years ago that ended up far from the nearest coast, an indication that inlanders were bartering to get ornamental seashells from coastal dwellers. Unlike the contemporary Neanderthals, who apparently relied just on local resources, those modern humans could shop for imports.

113

Reading Comprehension The extraordinary promise of this event was that Adam potentially now had access to objects he did not know how to make or find; and so did Oz. People traded goods, services and, most important, knowledge, creating a collective intelligence. Ten individuals could know between them ten things, while each understanding one. As they specialized and exchanged, humans learned how to domesticate crops and animals and sell food to passing merchants. Traders congregated in the first cities and built ships that spread goods and ideas around the world. Rulers like to take credit for the advances during their reigns, and scientists like to see their theories as the source of technological progress. But they’ve both got it backward: traders’ wealth builds empires, and entrepreneurial tinkerers are more likely to inspire scientists than vice versa. From Stone Age seashells to the steam engine to the personal computer, innovation has mostly been a bottom-up process. Forget wars, religions, famines and poems for the moment, this is history’s greatest theme: the metastasis of exchange, specialization and the invention it has called forth, the ‘creation’ of time. The modern world is a history of ideas meeting, mixing, mating and mutating, and the reason that economic growth has accelerated so in the past two centuries is down to the fact that ideas have been mixing more than ever before. Our progress is unsustainable only if we stifle innovation and trade, the way China and other empires did in the past. Is that possible? Well, European countries are already banning technologies based on the precautionary principle requiring advance proof that they’re risk-free. Americans are turning more protectionist and advocating Byzantine restrictions like carbon tariffs. Globalization is denounced by affluent Westerners preaching a return to self-sufficiency, but with new hubs of innovation emerging elsewhere, and with ideas spreading faster than ever on the Internet, bottom-up innovators will prevail.

ªScore Enhancerª 463.

Which of the following can be inferred from the first paragraph? 1) Throughout modern history, majority of intellectuals have tended to have a pessimistic outlook than an optimistic one. 2) Throughout modern history, majority of intellectuals have believed that the world is headed for ruin in the long run. 3) Throughout modern history, predictions of doom were not backed by data pointing in that direction. 4) Throughout modern history, optimists have been always in the minority and have been ridiculed by others for their positive beliefs.

464.

Which of the following does the author mean and refer to by the phrase bottom-up process? 1) 2) 3) 4)

The demands of trade have created the need for scientific and technological innovation. Scientific and technological innovations have fuelled the rise and spread of trade. Innovation has been possible by the interaction between scientists and traders. Rulers and scientists have not directly engaged in trade and hence not contributed to innovation.

114

Reading Comprehension ªScore Maximiserª 465.

Which of the following best captures the main idea of the passage? 1) Western protectionism notwithstanding, innovation, driven by other parts of the world, would ensure that human beings will continue to survive and thrive. 2) The future will belong to non-western countries driven by their zeal to innovate, and the ability of the Internet to spread ideas faster than at any other time in our history. 3) The usual predictions of unsustainability notwithstanding, man will continue to thrive, driven by the ability of trade to spur innovation through the exchange of ideas. 4) Trade has and, if not curbed, will continue to ensure that mankind thrives by fostering innovation and exchange of ideas; the usual predictions of doom not withstanding.

466.

Which of the following words would the author be most likely to use to describe the current tendency among European countries towards banning new technologies? 1) skeptical

2) alarmist

3) pessimistic

4) ambivalent

It was possible for Aristotle, starting from the premise that a man, being actually cultured, may also become literate, to ask whether there is a necessary or merely an accidental connection of literacy with culture. Such a question can hardly arise for us, to whom illiteracy implies, as a matter of course, ignorance, backwardness, unfitness for self-government: for us, unlettered peoples are uncivilized peoples, and vice versa. As against the complacent prejudice, there is no necessary connection of literacy with culture, and that to impose our literacy (and our contemporary “literature”) upon a cultured but illiterate people is to destroy their culture in the name of our own. For the sake of brevity we shall assume without argument that “culture” implies an ideal quality and a good form that can be realized by all men irrespective of condition: and, since we are treating of culture chiefly as expressed in words, we shall identify culture with “poetry”; not having in view the kind of poetry that nowadays babbles of green fields or that merely reflects social behavior or our private reactions to passing events, but with reference to that whole class of prophetic literature that includes the Bible, the Vedas, the Edda, the great epics. Of these “books” many existed long before they were written down, many have never been written down, and others have been or will be lost. While the merely literate are often very proud of their literacy, such as it is, it is only by men who are “not only literate but also cultured” that it has been widely recognized that “letters” at their best are only a means to an end and never an end in themselves, or, indeed, that “the letter kills.” A “literary” man, if ever there was one, the late Professor G. L. Kittredge writes: “The ability of oral tradition to transmit great masses of verse for hundreds of years is proved and admitted. To this oral literature education is no friend. When a nation begins to read, what was once the possession of the folk as a whole, becomes the heritage of the illiterate only, and soon, unless it is gathered up by the antiquary, vanishes altogether.” Mark, too, that this oral literature once belonged “to the whole people, the community whose intellectual interests are the same from the top of the social structure to the bottom,” while in the reading society it is accessible only to antiquaries, and is no longer bound up with everyday life. A point of further importance is this: that the traditional oral literatures interested not only all classes, but also all ages of the population; while the books that are nowadays written expressly “for children” are such as no mature mind could tolerate; it is now only the comic strips that appeal alike to children who have been given nothing better and at the same time to “adults” who have never grown up.

115

Reading Comprehension A master who can himself not only read, but also write good classical Latin and Greek, remarks that “there is no doubt of the quantitative increase in literacy of a kind, and amid the general satisfaction that something is being multiplied it escapes enquiry whether the something is profit or deficit.” He is discussing only the “worst effects” of enforced literacy, and concludes: “Learning and wisdom have often been divided; perhaps the clearest result of modern literacy has been to maintain and enlarge the gulf

ªScore Maximiserª 467.

Which of the following is the primary purpose of the passage? 1) To advocate a return to the oral traditions as a way of creating a purer culture and achieving truer wisdom. 2) To argue that contemporary notions about the cultural superiority of literate people over the illiterate are grossly misplaced. 3) To argue against the imposition of literacy by bringing to light its withering effects on indigenous culture, traditions and wisdom. 4) To outline the effect of literacy on in digenous people and argue for a culture that is inclusive of oral traditions.

Innovate or die. The phrase, popularized in Silicon Valley in the nineteen-nineties, has since become a mantra throughout the business world, and nowhere has it been more popular than on Wall Street, which in recent years has churned out a seemingly endless stream of new ways to manage capital and slice and dice risk. But, while Silicon Valley’s innovations have brought enormous benefits to society, the value of Wall Street’s innovations seems a lot less clear. The Valley gave us the microprocessor, Google, and the iPod. The Street gave us the C.D.O, the A.B.S, and the C.D.S —not to mention the kind of computerized trading that enabled the latest stock market crash. Not surprisingly, then, the whole notion of “financial innovation” is being looked at with a gimlet eye, and Congress is now considering various ways to rein in the banking industry’s excesses. Not all of Wall Street’s concoctions have been pointless or destructive, of course. Take junk bonds, whose use Michael Milken pioneered in the nineteen-eighties. They got a bad name when Milken went to prison for securities fraud. But his insight that high-yield bonds could be a good investment—that, historically, the rewards outweighed the risks—allowed new companies, including eventual giants like Turner Broadcasting and M.C.I., as well as countless smaller businesses, to raise billions in capital that previously would have been out of their reach. Today, almost two hundred billion dollars’ worth of junk bonds are sold every year; they’re an integral part of the way Wall Street does what it’s supposed to do: channel money from investors to productive enterprises. Raghuram Rajan, a former chief economist at the I.M.F. and a finance professor at the University of Chicago, says, “There’s a lot of stuff that does a lot of good that we take for granted, because it’s just become part of our everyday financial lives.” Unfortunately, the benefits of good financial innovations have, of late, been swamped by the costs of the ones that went bad. Some of these ideas, as it happens, were reasonable ones, within limits. But limits aren’t something that Wall Street knows much about: in recent years, it has shown an uncanny knack for taking reasonable ideas to unreasonable extremes. Of late economists have argued that financial innovation often leads to financial instability: investors get interested in a new product that seems to offer high returns, and, precisely because it’s new, underestimate the chance that this product will eventually blow up. As Rajan argued in a prescient 2005 paper, financial development, which was supposed to make the system safer, could in fact make it riskier. The fundamental problem with innovation was that it made investors and executives forget the need to think for themselves.

116

Reading Comprehension The cost of all these mad-scientist endeavors can be measured in the trillions of dollars that vaporized when the housing bubble burst. But another cost has been the damage done to the whole notion of financial innovation. “The real question is how do we keep the good parts of innovation without being stuck with the bad,” Rajan says. But even a potentially useful idea like the creation of a carbonpermit market to fight global warming is already being dismissed as Wall Street’s “next big scam.” And while the Yale economist Robert Shiller has long advocated using markets to help individuals protect themselves against things like declining house prices or future unemployment, the chances of that happening now seem smaller than ever. Someday, perhaps, we’ll be in the mood to experiment again. But that will happen only when Wall Street remembers that the phrase is “Innovate or die,” not “Innovate and die.”

ªScore Enhancerª 468.

According to the passage, which of the following is the underlying reason behind the instability caused by financial innovation? 1) Investors tend to be risk-seeking when faced with higher returns and risk-averse when faced with lower returns. 2) Tempted by higher returns, investors have tended to not evaluate risk and invest excessively in new financial products. 3) The absence of a regulatory body that will stringently evaluate new financial products before they are introduced into the market. 4) Investors and executives have collectively lacked the ability to evaluate the inherent benefits and risks of new financial products.

469.

Which of the following does the passage suggest is one of the biggest negative fallouts of Wall Street’s latest debacle? 1) The latest financial crisis might unleash a barrage of regulations that will strangle financial innovation. 2) An increase in global warming due to the lack of belief in carbon-permit markets, a Wall Street innovation, as a way to fight global warming. 3) Over the longer term the markets might be unable to perform their primary function – channelizing funds from investors to productive enterprises. 4) The lack of faith in financial innovation as a whole might lead to potentially good ideas being relegated to the dustbin.

ªScore Maximiserª 470.

Which of the following can be a suitable title for the passage? 1) 2) 3) 4)

Innovation: Bane or Boon? The pros and cons of financial innovation Innovate, The Right Way The future of financial innovation

117

Reading Comprehension In evolutionary biology, an organism is said to behave altruistically when its behaviour benefits other organisms, at a cost to itself. The costs and benefits are measured in terms of reproductive fitness, or expected number of offspring. So by behaving altruistically, an organism reduces the number of offspring it is likely to produce itself, but boosts the number that other organisms are likely to produce. This biological notion of altruism is not identical to the everyday concept. In everyday parlance, an action would only be called ‘altruistic’ if it was done with the conscious intention of helping another. But in the biological sense there is no such requirement. Indeed, some of the most interesting examples of biological altruism are found among creatures that are (presumably) not capable of conscious thought at all, e.g. insects. For the biologist, it is the consequences of an action for reproductive fitness that determine whether the action counts as altruistic, not the intentions, if any, with which the action is performed. Altruistic behaviour is common throughout the animal kingdom, particularly in species with complex social structures. For example, vampire bats regularly regurgitate blood and donate it to other members of their group who have failed to feed that night, ensuring they do not starve. In numerous bird species, a breeding pair receives help in raising its young from other ‘helper’ birds, who protect the nest from predators and help to feed the fledglings. Vervet monkeys give alarm calls to warn fellow monkeys of the presence of predators, even though in doing so they attract attention to themselves, increasing their personal chance of being attacked. In social insect colonies (ants, wasps, bees and termites), sterile workers devote their whole lives to caring for the queen, constructing and protecting the nest, foraging for food, and tending the larvae. Such behaviour is maximally altruistic: sterile workers obviously do not leave any offspring of their own — so have personal fitness of zero — but their actions greatly assist the reproductive efforts of the queen. From a Darwinian viewpoint, the existence of altruism in nature is at first sight puzzling, as Darwin himself realized. Natural selection leads us to expect animals to behave in ways that increase their own chances of survival and reproduction, not those of others. But by behaving altruistically an animal reduces its own fitness, so should be at a selective disadvantage vis-à-vis one which behaves selfishly. To see this, imagine that some members of a group of Vervet monkeys give alarm calls when they see predators, but others do not. Other things being equal, the latter will have an advantage. By selfishly refusing to give an alarm call, a monkey can reduce the chance that it will itself be attacked, while at the same time benefiting from the alarm calls of others. So we should expect natural selection to favour those monkeys that do not give alarm calls over those that do. But this raises an immediate puzzle. How did the alarm-calling behaviour evolve in the first place, and why has it not been eliminated by natural selection? How can the existence of altruism be reconciled with basic Darwinian principles?

ªScore Enhancerª 471.

Which of the following is the primary purpose of the passage? 1) To highlight the shortcomings in the definition of altruism according to evolutionary biology by pointing out its contradictions with Darwinian principles of altruism. 2) To argue for a need to re-evaluate the definition of altruism according to evolutionary biology since it is inconsistent with Darwinian principles of natural selection. 3) To illustrate the definition of altruism according to evolutionary biology and point out its contradiction with Darwinian principles of natural selection. 4) To demonstrate that the Darwinian prin ciples of natural selection are inconsistent since they do not explain altruistic behaviour as defined by evolutionary biology.

118

Reading Comprehension ªScore Maximiserª 472.

Which of the following statements are correct according to the passage? I. II. III. 1) 3)

473.

In several bird species, the whole community, rather than parents, looks after the needs of the children. Vervet monkeys are the most altruistic of all monkey species, and several instances of their altruism have been recorded. Some bees spend their whole life caring for the queen, and consequently do not reproduce. Both I & II 2) Both I & III Only III 4) None of the statements are correct

Which of following statements, if true, reconciles the conflict between Darwinism and altruistic behaviour as explained in the passage? I.

Altruistic behaviours that help other species have been found to be consistent with the modern theories of evolution. II. Altruism and natural selection work to ensure survival of a species as a whole and not its individual organisms. III. Natural selection does not retain or eliminate behavioural patterns but is limited only to physical characteristics. 1) Only I 2) Only II 3) Only III 4) Both II & III

What is history? Most prosaically, it is the human past and our organized representations of that past. We can of course write about the chronology of non-human events—the history of the solar system, the history of the earth’s environment over a billion-year expanse of time. But the key issues in the philosophy of history arise in our representations of the human past. And history is fascinating for us, because, in Marx’s words, “Men make their own history, but not in circumstances of their own choosing”. That is to say: history reflects agency—the choices by individuals and groups; and it reflects constraining structures and circumstances. So historical outcomes are neither causally determined nor entirely plastic and accidental. Therefore it is open to the historian to attempt to discover the historical circumstances that induced and constrained historical agents to act in one way rather than another— thus bringing about a historical outcome of interest. So we might begin by saying that history is a temporally ordered sequence of events and processes involving human doings, within which there are interconnections of causality, structure, and action, within which there is the play of accident, contingency, and outside forces. But we might also say: there is no such thing as “history in general.” The description just provided suggests that there is a comprehensible collection of historical processes that might be characterized as a “total” human history: population growth, urbanization, technological innovation, economic differentiation, the growth of knowledge and culture, etc. But this impression is highly misleading. It suggests a degree of order and structure that history does not possess. There are only specific histories: histories of various conditions or circumstances of interest to us. Historical space is dense: at any given time there are countless human actions and social processes underway in the world. So to single out the history of something specific—agriculture, the French Revolution, modern science, Islam—is unavoidably to select, from the full complexity of events and actions, a limited set of related historical features that will be traced through a process of development. And this in turn raises the point that “history” depends partly on “what occurred” and partly on “what we are interested in”. This point does not undercut the objectivity of judgments about the past. Events and actions happened in the past, separate from our interest in them. But organizing them into a narrative about “religious awakening” or “formation of the absolutist state” imposes an interpretive structure on them that depends inherently on the observer’s interests. There is no such thing as “perspective-free history”.

119

Reading Comprehension ªScore Maximiserª 474.

Which of the following is the central idea of the passage? 1) Historical studies are misleading and are only valid when applied to specific contexts and viewed through specific lenses. 2) There is no such thing as a universal history since it is impossible to structure and order specific histories occurring simultaneously in different circumstances. 3) Human beings make their own history but they do so within the bounded limits of the constraints placed on them by societal structure and circumstances. 4) History is constituted by historical interpr etation and traditions of historical interest— even though the underlying happenings themselves are not.

475.

Which of the following is the reason why the author suggests that there is no such thing as “history in general”? 1) The idea of a history in general means imposing a non-existent order and causality on a large number of complex human actions. 2) The idea of a history in general foregoes the accidental and the unrelated in favour of a narrative based on temporal and causal order. 3) A history in general is impossible to define since there can be no causal relationship between people and events belonging to different times, places and contexts. 4) A history in general presupposes an objective view of events, which is possible only within the framework of our interpretive structure.

476.

Which of the following statements is true according to the passage? I.

Our judgements about the past cannot be objective since they are always based on the perspective through which we view the past. II. The actual events in the past are independent of the interpretive structure through which we want to view them. III. Historical studies can never capture events in all of their complexity. 1) Only I 2) Only II 3) Both II and III 4) II and III Directions for Questions 56 to 58: The passage given below is followed by a set of questions. Choose the most appropriate answer to each question. Colonialism is not a modern phenomenon. World history is full of examples of one society gradually expanding by incorporating adjacent territory and settling its people on newly conquered territory. The ancient Greeks set up colonies as did the Romans, the Moors, and the Ottomans, to name just a few of the most notorious examples. Colonialism, then, is not restricted to a specific time or place. Nevertheless, in the sixteenth century, colonialism changed decisively because of technological developments in navigation that began to connect more remote parts of the world. Fast sailing ships made it possible to reach distant ports while sustaining closer ties between the centre and colonies. Thus, the modern European colonial project emerged when it became possible to move large numbers of people across the ocean and to maintain political sovereignty in spite of geographical dispersion.

120

Reading Comprehension The difficulty of defining colonialism stems from the fact that the term is often used as a synonym for imperialism. Both colonialism and imperialism were forms of conquest that were expected to benefit Europe economically and strategically. The term colonialism is frequently used to describe the settlement of places such as North America, Australia, New Zealand, Algeria and Brazil that were controlled by a large population of permanent European residents. The term imperialism often describes cases in which a foreign government administers a territory without significant settlement; typical examples include the scramble for Africa in the late nineteenth century and the American domination of the Philippines and Puerto Rico. The distinction between the two, however, is not entirely consistent in the literature. Some scholars distinguish between colonies for settlement and colonies for economic exploitation. Others use the term colonialism to describe dependencies that are directly governed by a foreign nation and contrast this with imperialism, which involves indirect forms of domination. The confusion about the meaning of the term imperialism reflects the way that the concept has changed over time. Although the English word imperialism was not commonly used before the nineteenth century, Elizabethans already described the United Kingdom as “the British Empire”. As Britain began to acquire overseas dependencies, the concept of empire was employed more frequently. Thus, the traditional understanding of imperialism was a system of military domination and sovereignty over territories. The day-to-day work of government might be exercised indirectly through local assemblies or indigenous rulers who paid tribute but sovereignty rested with the British. The shift away from this traditional understanding of empire was influenced by the Leninist analysis of imperialism as a system oriented towards economic exploitation. According to Lenin, imperialism was the necessary and inevitable result of the logic of accumulation in late capitalism. Thus, for Lenin and subsequent Marxists, imperialism described a historical stage of capitalism rather than a trans-historical practice of political and military domination. The lasting impact of the Marxist approach is apparent in contemporary debates about American imperialism, a term which usually means American economic hegemony, regardless of whether such power is exercised directly or indirectly.

ªScore Enhancerª 477.

Which of the following is the primary purpose of the passage? 1) To show the similarities between colonialism and imperialism in practice despite the difference in their theoretical definitions. 2) To argue that colonialism and imperialism have always gone hand in hand with military and economic exploitation. 3) To explore the definitions of the terms colonialism and imperialism, their us age, evolution and inter-linkages. 4) To argue that colonialism and imperialism are not new phenomena but have always existed since ancient times.

478.

Which of the following have been cited in the passage as the difference(s) between colonialism and imperialism? I. Colonies are directly governed while imperialism involves indirect control. II. Colonizers move and settle a large number of people from their own country into colonies while imperialism does not involve large settlement in colonies. III. Unlike imperialism, colonialism involves only administrative control and not military control. 1) Both I and II 2) Both II and III 3) Both I and III 4) I, II and III

121

Reading Comprehension 479.

According to the passage, how do the Marxist-Leninist analyses of imperialism differ from earlier definitions of imperialism? 1) From the Marxist-Leninist perspective, imperialism was primarily an economic phe-nomenon and not a military one as per the earlier definitions. 2) From the Marxist-Leninist perspective, imperialism was an outcome of capitalism and not a political- military domination as was believed earlier. 3) While imperialism was traditionally viewed as being a political and military domination, from the Marxists-Leninist perspective, it was the domination of capitalism over communism. 4) Traditional definitions of imperialism, unlike the Marxist-Leninist analyses, failed to take into account the American economic hegemony, which is a form of imperialism.

Directions for questions 3 to 5: The passage given below is followed by a set of questions. Choose the most appropriate answer to each question. Wisdom is not the same as knowledge, and so it seems odd it has attracted the attention of science. There is such a thing as “wisdom studies” now, and researchers and neuroscientists are in search for the latest information about wisdom. Scientists treat wisdom the way they treat anything else. They break it down into its smallest components to identify and test, and they attempt to figure out how it works, how to obtain it, and what it is. According to researchers there are eight attributes of wisdom: Emotional Regulation, Knowing What’s Important, Moral Reasoning, Compassion, Humility, Altruism, Patience, and Dealing with Uncertainty. Tests are designed, studies are lined up, and college undergrads short of cash or in need of class credit are recruited as lab rats in our pursuit of wisdom. The problem is that wisdom is elusive, and the act of reducing it down to a binary code seems ridiculous. Take a common test for moral reasoning: A trolley is out of control and will kill five people unless you pull the lever for the trolley to switch tracks, resulting in the death of one person. What do you do? Researchers later switched it up to find people’s moral threshold: How far would you go to save those five people? How much would you participate in that one person’s death? Would you kill him or her with your own hands? The problem with the test is that it has only two answer choices: yes and no. Everyone in the world knows how you’re supposed to answer: you are supposed to kill the one to save the five. What’s more interesting is now that we have neuro-imaging, we can watch a person’s brain at work, their emotional response, how rationality has to override disgust. A person might give the right answer on the survey and still be the type of person who doesn’t intervene when he or she hears a cry for help. There is a danger in seeing this as a map of isolated points rather than a threedimensional, pulsing, dynamic network of neural coordination, one that is constantly changing, and changeable, one that is weighted with different inputs depending on our previous experiences, our learning, our mood that day, the general uncertainty or anxiety we may be feeling, our life circumstances at any time, our age and stage of life — a network that is, in a word, idiosyncratic. The problem with many behaviourists is their simplistic reduction of our desires, motivation, and reasoning. And so while the neuro-imaging might teach us that our brains are much more complex than the standard self-reporting tests reveal, the experiments break down when they try to encompass all we bring to every decision, or when they figure out why knowing the wise or moral thing to do does not lead one to actually do that thing. We crave wisdom — worship it in others, wish it upon our children, and seek it ourselves. But there’s a difference between admiring wisdom and emulating it. That’s perhaps the best illustration of the difference between knowledge and wisdom.

122

Reading Comprehension ªScore Enhancerª 480.

Which of the following best captures the essence of the author’s main argument against ‘wisdom studies’? 1) Studies using binary tests with predictable answers cannot capture the complexity of the psycho logical processes behind real-time human decision-making. 2) Studies using undergrad students will tend to produce results that validate researchers’ hypothesis rather than reflect real-time decision-making. 3) Studies using tests cannot account for the many variables that are involved in real-time decisionmaking scenarios. 4) Studies using binary tests tend to skew the findings in one particular direction, ignoring the grey areas of real-time human decision-making.

ªScore Maximiserª 481.

Which of the following words is the author most likely to use to describe scientific models that deconstruct wisdom? 1) Rational

482.

2) Simplistic

3) Causal

4) Inane

Which of the following would be a suitable title to the passage? 1) Can we understand wisdom? 3) The science behind wisdom.

2) Our knowledge of wisdom 4) Bounded knowledge, elusive wisdom

Directions for questions 11 to 13: The passage given below is followed by a set of questions. Choose the most appropriate answer to each question. To use the language of the 18th-century economist Adam Smith, the value of the arts “in use” precedes their value “in exchange”. Once something is deemed desirable, the market can indeed establish its commercial price. But although the market can trade in the products of culture, it cannot express the value of culture as a process, or what it does. A cultural economics that captures the value of the arts has to understand value in use, and that involves broader ways of understanding ourselves and our world, for instance, anthropology and environmentalism. The value in use of the arts is that they help a society make sense of itself. They generate the symbols and rituals that create a common identity—that is why art and religion are so closely linked. Like religion, the arts give access to the spiritual. Art is a link to previous generations, and anchors us to history. These anthropological arguments show why the government, as guarantor of the public realm, should take responsibility for ensuring that everyone has access to this language, and that it is both preserved and developed. For, as the environmentalists argue, it is necessary to intervene when a resource is at risk. The precautionary principle tells us we have a duty to future generations to ensure that our cultural assets are passed on to them. We also have a selfish interest in sustaining the richness and diversity of those assets. Creativity occurs through the interaction of different forms—life forms, or art forms. Culture creates social capital, expressed as trust generated by a shared understanding of the symbols that the arts generate, and a commitment to the values they represent. It sustains the legitimacy of social institutions by ensuring that they are accepted, not imposed. Societies with an equitable distribution of cultural assets will be more cohesive, and more creative. Wellbeing, which is the true end of economic activity, depends on the quality of life that culture sustains. The word “culture”, after all, means “growth”.

123

Reading Comprehension Social capital—like economic capital—requires both regulation and investment. That the educated and well-off have greater access to the arts is not an argument for abandoning intervention to secure a more equitable distribution of cultural experience. Rationally, the government should be putting more funding into the arts because of the social capital they generate. There is a sound economic argument that when the market fails to provide certain kinds of goods thought useful, then it is necessary to intervene—health and education are the usual examples. The economics of the arts are particularly prone to market failure, for it is not easy to make the advances in productivity that technology facilitates in manufacturing. A symphony played on a synthesiser is not an efficiency gain. It seems particularly ironic, then, that the creator and first chairman of the post-war Arts Council was the economist John Maynard Keynes. He believed that in a recession, governments should stimulate the economy. He also understood the use value of the arts. The decision taken in 1940 that led to long-term funding of the arts was not taken on economic grounds, or for reasons of health, social inclusion or the prevention of crime. But it was a rational decision, based on a rational argument: that we are supposed to be fighting for civilisation.

ªScore Enhancerª 483.

Which of the following best summarizes the anthropological arguments in support of the arts as outlined in the first paragraph? 1) The value of the arts does not lie in their commercial value but in their use by a society to further its shared values. 2) The arts are what distinguish one country from another, thus breeding the diversity upon which creativity thrives. 3) The arts are a society’s means of making sense of its history and traditions, creating a common identity. 4) The arts are the means through which a society evolves its common religious and cultural identity.

484.

According to the passage, which of the following is the reason why cultural assets should be equitably distributed? I.

Cultures enable the creation and propagation of shared values that confer legitimacy to social institutions. II. Culture generates and sustains a way of life that promotes well-being. III. Culture enables the forging of a cohesive national identity that sets a country apart from others. 1) Both I and II 2) Both II and III 3) Both I and III 4) I, II and III 485.

Which of the following is a governmental legislation that the author would most likely be indifferent to? 1) 2) 3) 4)

Setting up institutions to preserve and propagate traditional art forms. Making the entry to museums and art galleries free of cost. Revoking the tax rebate given to popular films to support non-commercial cinema. Supporting artists with financial endowments.

124

Reading Comprehension The idea that some characteristics of an organism are explained by the organism’s intrinsic nature, whilst others reflect the influence of the environment is an ancient one. It has even been argued that this distinction is itself part of the evolved psychology of the human species. Innateness must be clearly distinguished from heritability, at least in the scientific sense of that term. The idea that heritability scores measure the degree to which a characteristic is innate is a vulgar fallacy. Heritability is a statistical measure of the sources of individual differences in a population. While heritability itself is well understood, its relationship to the innate/acquired distinction remains highly controversial. The belief that a trait is innate is today commonly expressed by saying it is ‘in the genes’. But genes play an essential role in the production of every trait. Consequently, it will not do to say simply that innate traits are ‘caused by genes’ whilst acquired traits are ‘caused by the environment’. Any relationship between genetic causation and the innate/acquired distinction will be far more complex than this. Recent philosophical analyses of the innate/acquired distinction can be classified into four types. The first identifies innate traits with those characteristics of an entire species and identifies acquired traits with those that vary between populations and individuals. A second type of analysis identifies innate traits with those that can be explained by natural selection. The third, and currently the most influential, identifies innate traits with those produced by a particular patterns of interaction between genes and environment. A fourth, quite different, type of analysis suggests that labelling a trait ‘innate’ is a way to indicate that it lies outside the domain of psychology. Finally, there is a tradition of scepticism about the innate/acquired distinction. Sceptics argue that it confounds a number of distinctions that are better kept separate, or, perhaps equivalently, that there is no one property of a trait that corresponds to its being innate. Philosophical scepticism about innateness draws on a longstanding tradition of scepticism in developmental psychobiology. This research tradition interprets many of its own results as demonstrating the inadequacy of the innate/acquired distinction, and it is critical of the results and interpretations of results found in neo-nativist research. Philosophical defenders of the distinction would benefit from looking at these cases, in which the distinction is purportedly inapplicable or unhelpful, in addition to the cases used by neo-nativists to exemplify the distinction. One thing seems clear, which is that efforts to clarify the distinction in psychology by appeal to the underlying genetics have not been successful. The innate/acquired distinction does not seem to get much grip on the findings of lower-level developmental sciences such as molecular developmental biology. Once again, this suggests that the distinction may be best understood via its actual use in psychological research.

ªScore Maximiserª 486.

Which of the following statements would the author most likely agree with? I. II. III. 1)

Every genetically inherited trait cannot be classified as being innate. All traits that are not genetically transmitted are traits acquired from one’s environment. All traits are fundamentally genetic since genes play a role in the production of every trait. Only I 2) Only III 3) Both I and II 4) I, II and III

125

Reading Comprehension 487.

Why does the author ask the philosophical defenders of the distinction to look at the research carried out in the field of developmental psychobiology? 1) The research shows that there is nogenetic basis to classify traits as being either innate or acquired. 2) The research proves that the neo-nativist distinction between traits as being innate or acquired is incorrect. 3) The research exemplifies the neo-nativist distinction between traits as being either innate or acquired. 4) The research shows the difficulty to conclusively classify traits as being either innate or acquired.

488.

Which of the following best captures the central idea of the passage? 1) The distinction between innate and acquired traits is largely a psychological one since it has not been supported by genetics or molecular developmental biology. 2) The distinction between innate and acquired traits holds no water since studies have shown that no single property of a trait corresponds to its being innate. 3) Psychology still remains the only area within which the distinction between innate and acquired traits might be understood, as other areas have failed to clarify the distinction. 4) The age-old debate between innate and acquired traits has not yet been resolved due to the reliance on genetics and molecular biology instead of psychological research.

Directions for Questions 41 to 43: The passage given below is followed by a set of questions. Choose the most appropriate answer to each question. Directed to art, interpretation means plucking a set of elements from the whole work. The task of interpretation is virtually one of translation. The interpreter says, “Look, don’t you see that X is really – or, really means – A? That Y is really B? That Z is really C?” What situation could prompt this curious project for transforming a text? History gives us the materials for an answer. Interpretation first appears in the culture of late classical antiquity, when the power and credibility of myth had been broken by the “realistic” view of the world introduced by scientific enlightenment. Once the question that haunts post-mythic consciousness – that of the seemliness of religious symbols – had been asked, the ancient texts were, in their pristine form, no longer acceptable. Then interpretation was summoned, to reconcile the ancient texts to “modern” demands. Thus, the Stoics, to accord with their view that the gods had to be moral, allegorized away the rude features of Zeus and his boisterous clan in Homer’s epics. What Homer really designated by the adultery of Zeus with Leto, they explained, was the union between power and wisdom. In the same vein, Philo of Alexandria interpreted the literal historical narratives of the Hebrew Bible as spiritual paradigms. The story of the exodus from Egypt, the wandering in the desert for forty years, and the entry into the promised land, said Philo, was really an allegory of the individual soul’s emancipation, tribulations, and final deliverance. Interpretation thus presupposes a discrepancy between the clear meaning of the text and the demands of (later) readers. It seeks to resolve that discrepancy. The situation is that for some reason a text has become unacceptable; yet it cannot be discarded. Interpretation is a radical strategy for conserving an old text, which is thought too precious to repudiate, by revamping it. The interpreter, without actually erasing or rewriting the text, is altering it. But he can’t admit to doing this. He claims to be only making it intelligible, by disclosing its true meaning. However far the interpreters alter the text (another notorious example is the Rabbinic and Christian “spiritual” interpretations of the clearly erotic Song of Songs), they must claim to be reading off a sense that is already there.

126

Reading Comprehension Interpretation in our own time, however, is even more complex. For the contemporary zeal for the project of interpretation is often prompted not by piety towards the troublesome text (which may conceal an aggression), but by an open aggressiveness, an overt contempt for appearances. The old style of interpretation was insistent, but respectful; it erected another meaning on top of the literal one. The modern style of interpretation excavates, and as it excavates, destroys; it digs “behind” the text, to find a sub-text, which is the true one. The most celebrated and influential modern doctrines, those of Marx and Freud, actually amount to elaborate systems of hermeneutics, aggressive and impious theories of interpretation. All observable phenomena are bracketed, in Freud’s phrase, as manifest content. This manifest content must be probed and pushed aside to find the true meaning – the latent content – beneath. Actually, they have no meaning without interpretation. To understand is to interpret. And to interpret is to restate the phenomenon, in effect to find an equivalent for it. Thus, interpretation is not (as most people assume) an absolute value, a gesture of mind situated in some timeless realm of capabilities. Interpretation must itself be evaluated within a historical view of human consciousness. In some cultural contexts, interpretation is a liberating act. It is a means of revising, of trans-valuing, of escaping the dead past. In other cultural contexts, it is reactionary, impertinent, cowardly, stifling.

ªScore Maximiserª 489.

According the author, the ancient texts began to be interpreted by moderns in order to: 1) 2) 3) 4)

490.

Which of the following is suggested or implied as a reason why interpreters of ancient texts claim that they are ‘reading off a sense that is already there’ rather than altering it? I. II. III. 1) 3)

491.

understand and reveal their true hidden meaning, which lies buried behind allegory. bridge the gap between their literal and their transcendental meaning. make them compatible with contemporary sensibilities from which they greatly differ. transform them from the realm of the pagan to the realm of the spiritual.

To give legitimacy to their interpretation The texts are considered too sacred to alter To make their interpretations appeal to a large audience Both I and II 2) Both II and III Both I and III 4) I, II and III

According to the author, which of the following is the key difference between interpretation in our own time and the earlier acts of interpretation? 1) Earlier interpreters treated the religious content of the texts with a certain piety, whereas modern interpreters have no qualms about openly blaspheming them. 2) Earlier interpreters respected the original text and found other meanings in it; today a text itself is supposed to have no value, only its interpretation does. 3) Earlier interpreters evaluated ancient texts from a spiritual point of view, whereas modern interpreters analyze them from a Marxist and Freudian point of view. 4) Earlier interpreters never claimed that they were altering a text but modern inter preters are open about the fact that they are re-writing the text.

127

Reading Comprehension We all thought that the fall of the Berlin Wall, and of the Iron Curtain, would dismantle barriers, but 20 years later, there are new and subtler walls and social dividers around many neighbourhoods globally. Not just the big ones – Belfast’s “peace walls” and the new US-Mexico and Israel-West Bank divisions – but all the residential gated development, in places often with low crime rates and significant local affluence. These micro fortifications now accommodate around four million US households and a thousand such developments are scattered across England. In our affluent societies, the ascription of disorder outside the home requires us to launch ourselves into sites of incalculable risk, unpredictable social encounter and a degraded public realm that worries or depresses us. We find ourselves moving between micro-bordered worlds, from one safe zone to another; what the Dutch theorist Lieven de Cauter calls a “capsular civilisation”. A fear of personal harm and the pursuit of status have created a built environment in a generalised state of alarm, further feeding the rationale by which such citadel spaces were engineered and generating a sense of in-group affluence and out-group danger. Where is the alternative vision for more equitable, pro-social, joyful urban experiences? Such dividends will only come through reducing the inequality exclusion felt by so many but, as Herbert Gans said about desegregating American cities in the 1960s, it seems unfeasible that popular politics will assist an affluent majority to vote for what it will see as its own impoverishment. We are scared of the wrong things. We fear violence at the hands of strangers but are more likely to be attacked by partners; we are alerted to terrorists without a history or acknowledgment of geo-political injustice; we focus on the rare and the cruel without understanding social complexity or its context. The world outside the front door appears as a random, vicious trap. The walling, gating and locking-down of the domestic sphere is the ultimate endpoint of this fear, but we also need to recognise the logic by which political and corporate life reaps dividends. Developers have an interest in highlighting the communality of life in gated zones while drawing attention to the risks of living outside them. These fears aren’t without foundation, yet it is social disparities under neoliberalism that have generated these increased risks of personal harm at the hands of the excluded while offering market opportunities for private security products. And politicians seek political capital by selling themselves as saviours from terror, economic variability, immigration, drugs, crime and the blowback of ecological catastrophe. 492.

Which of the following is the author primarily concerned with? 1) Bringing to light the gross misconceptions behind, and consequences of, the affluent class living in the gated communities and alienating other social classes. 2) Evaluating the validity of the reasons behind the affluent class of the developed world choosing to live in gated communities. 3) Elaborating and arguing against the reasons behind the rise of gated communities housing the affluent across the developed world. 4) Arguing for a more equitable distribution of urban space between the affluent and other social classes.

493.

According to the author, the fears of the affluent about their personal safety are: I. II. III. 1) 3)

exaggerated and misplaced. not based on actual understanding of the external world. exploited by the ruling and business class. Both I and II 2) Both II and III Both I and III 4) I, II and III

128

Reading Comprehension 494.

Which of the following can be a suitable title to the passage? 1) Prisoners by choice 3) Affluent fears

2) Then walls, now gates 4) The rise of the gated community

Directions for Questions 47 to 49: The passage given below is followed by a set of questions. Choose the most appropriate answer to each question. While physical books may be on the road to obsolescence, the road will almost certainly be a long and winding one and books and book reading, at least as we’ve defined those things in the past, are in their cultural twilight. As a society, we devote ever less time to reading printed words, and even when we do read them, we do so in the busy shadow of the Internet. Some thinkers welcome the eclipse of the book and the literary mind it fostered. Mark Federman, an education researcher, has argued that literacy, as we’ve traditionally understood it, “is now nothing but a quaint notion, an aesthetic form that is as irrelevant to the real questions and issues of pedagogy today as is recited poetry — clearly not devoid of value, but equally no longer the structuring force of society”. The time has come, he said, for teachers and students alike to abandon the “linear, hierarchical” world of the book and enter the Web’s “world of ubiquitous connectivity and pervasive proximity” — a world in which “the greatest skill” involves “discovering emergent meaning among contexts that are continually in flux”. Clay Shirky, a digital-media scholar, suggests that we shouldn’t waste our time mourning the death of deep reading — it was overrated all along. “No one reads War and Peace,” he wrote, singling out Tolstoy’s epic as the quintessence of high literary achievement. Such proclamations seem a little too staged to take seriously. Although it may be tempting to ignore those who suggest the value of the literary mind has always been exaggerated, that would be a mistake. Their arguments are another important sign of the fundamental shift taking place in society’s attitude towards intellectual achievement. Their words also make it a lot easier for people to justify that shift — to convince themselves that surfing the Web is a suitable, even superior, substitute for deep reading and other forms of calm and attentive thought. In arguing that books are archaic and dispensable, Federman and Shirky provide the intellectual cover that allows thoughtful people to slip comfortably into the permanent state of distractedness that defines the online life. Our desire for fast-moving, kaleidoscopic diversions didn’t originate with the invention of the World Wide Web. It has been present and growing for many decades, as the pace of our work and home lives has quickened and as broadcast media like radio and television have presented us with a welter of programmes, messages and advertisements. The Internet, though it marks a radical departure from traditional media in many ways, also represents a continuation of the intellectual and social trends that emerged from people’s embrace of the electric media of the twentieth century and that have been shaping our lives and thoughts ever since. The distractions in our lives have been proliferating for a long time, but never has there been a medium that, like the Net, has been programmed to so widely scatter our attention and to do it so insistently. On the Net, there are windows within windows within windows, not to mention long ranks of tabs primed to trigger the opening of even more windows. Multitasking has become so routine that most of us would find it intolerable if we had to go back to computers that could run only one program or open only one file at a time. And yet, even though the question may have been rendered moot, it remains as vital today as it was thirty-five years ago. It points to “a conflict between two different ways of working and two different understandings of how technology should be used to support that work”. In the choices we have made, consciously or not, about how we use our computers, we have rejected the intellectual tradition of solitary, single-minded concentration, the ethic that the book bestowed on us. We have cast our lot with the juggler.

129

Reading Comprehension 495.

The author says that Clay Shirky’s statement makes it difficult to take the latter seriously because: 1) 2) 3) 4)

496.

disparaging Tolstoy seems calculated to gain attention rather than make a point. deep-reading is not over-rated like Shirky would like us to believe. War and Peace is a masterpiece despite whatever Shirky has to say and spread of the Internet. there is no evidence to say that War and Peace will not be read digitally.

Which of the following statements does the author make about the Internet vis-à-vis traditional media? I.

Unlike other media, which entertain, the Internet is seminal in its allowing us to multitask and dispersing our attention span. II. The Internet has exponentially multiplied the ability of the electronic media to distract our attention. III. The Internet is the latest in the list of mass mediums, which ever since their inception, have been consistently chipping away at our attention span. 1) Both I and II 2) Both II and III 3) Both I and III 4) I, II and III 497.

Which of the following best describes the author’s attitude towards the effect the Internet has had on our mindset? 1) nostalgic

2) indifferent

3) ambivalent

4) disappointed

Directions for questions 1 to 3: The passage given below is followed by a set of questions. Choose the most appropriate answer to each question. The question of whether languages shape the way we think goes back centuries; Charlemagne proclaimed that "to have a second language is to have a second soul". But the idea went out of favour with scientists when Noam Chomsky's theories of language gained popularity in the 1960s and '70s. Dr. Chomsky proposed that there is a universal grammar for all human languages-essentially, that languages don't really differ from one another in significant ways. And because languages didn't differ from one another, the theory went, it made no sense to ask whether linguistic differences led to differences in thinking. Now, a flurry of new cognitive science research is showing that in fact, language does profoundly influence how we see the world. In the past decade, cognitive scientists have begun to measure not just how people talk, but also how they think, asking whether our understanding of even such fundamental domains of experience as space, time and causality could be constructed by language. Patterns in language offer a window on a culture's dispositions and priorities. For example, English likes to describe events in terms of agents doing things. English speakers tend to say things like "John broke the vase" even for accidents. Speakers of Spanish or Japanese would be more likely to say "the vase broke itself". Such differences between languages have profound consequences for how their speakers understand events, construct notions of causality and agency, what they remember as eyewitnesses and how much they blame and punish others. So does language shape cultural values, or does the influence go the other way, or both? Languages, of course, are human creations, tools we invent and hone to suit our needs. Simply showing that speakers of different languages think differently doesn't tell us whether it's language that shapes thought or the other way around. To demonstrate the causal role of language, what's needed are studies that directly manipulate language and look for effects in cognition.

130

Reading Comprehension One of the key advances in recent years has been the demonstration of precisely this causal link. It turns out that if you change how people talk, that changes how they think. If people learn another language, they inadvertently also learn a new way of looking at the world. When bilingual people switch from one language to another, they start thinking differently, too. And if you take away people's ability to use language in what should be a simple nonlinguistic task, their performance can change dramatically, sometimes making them look no smarter than rats or infants. The structures that exist in our languages profoundly shape how we construct reality, and help make us as smart and sophisticated as we are. Language is a uniquely human gift. When we study language, we are uncovering in part what makes us human, getting a peek at the very nature of human nature. As we uncover how languages and their speakers differ from one another, we discover that human natures too can differ dramatically, depending on the languages we speak. The next steps are to understand the mechanisms through which languages help us construct the incredibly complex knowledge systems we have. Understanding how knowledge is built will allow us to create ideas that go beyond the currently thinkable.

ªScore Enhancerª 498.

According to the passage, which of the following thing(s) about a culture does its language tell us? I. II. III. 1) 3)

The social structure that governs the culture. The importance given to human actions in that culture. The judicial processes of a culture. Only I 2) Only II Only III 4) None of the three

ªScore Maximiserª 499.

Which of the following is implied from the performance of people in non-linguistic tasks when prevented from using language? 1) Without language we might be incapable of interpreting and organizing the reality of the external world. 2) Language shapes the way we think in ways that we have not yet been able to fully understand. 3) The boundaries of our language are perhaps the boundaries of our thought. 4) Everything that can be thought cannot be said.

500.

Which of the following can be a suitable title for the passage? 1) 2) 3) 4)

Language and Culture Linguistics and Thought Does Language Shape the Way We Think? A Philosophy of Language

131

EXPLANATORY ANSWERS Arithmetic

1 1 1 1 1 1 1 1 1 1 – + – + – + – + – )} 5 7 6 8 7 9 8 10 9 11

ªScore Enhancerª 1.

2.

3.

The given number can be expressed as n = 10 2009 – 1 2 2009 – 1) 2 ∴ n = (10 = 10 4018 – 2(10 2009)(1) + 1 = 10 2009(10 2009 – 2) + 1 Now, 102009 –2 is 99 ...... 998 with a total of 2009 digits. Multiplying this by 10 2009 will add 2009 zeroes following the 8, and adding 1 will change the right-most 0 to 1. ∴ Sum of digits = 2008(9) + 8 + 1 = 18081 Hence, [2].

5.

x = 11 (

as

1



1



1



1

1

1

1 55

1 66

1

,

1

1

1 1 , 7 7 and 8 8 . Now we can compare

1

5 5 and 6 6 by raising both to the 30 th power. Thus we must compare 5 6 and 6 5 and we will find that 56 is bigger. In similar fashion we can compare all the 1



200 10 × 20

1

1

1

1



1



1



1

5 5 < 6 6 < 7 7 < 8 8 . Hence, (4).

1 1 1 1 1 1 1 1 + + + + + + + + 3 8 15 24 35 48 63 80

6.

The shortest path from F to A will comprise of two edges eg. F – B – A Thus, the longest path should not contain the vertex B. There are several paths possible, but the longest path comprises of 6 edges eg. A – E – H – D – C – G – F. (observe that it does not contain the vertex B). Let x be the length of each edge of the cube. 6x 2x + = 120 2 1 ∴ x = 24 cm Hence, (3).



7.

1 ) 99

∴ 5x = 120

Let the initial mixture is 100 units by weight, and contains 30 units of A and 70 units of B. After C is added, A is still 30 units, and is 12% of the full mixture; 30 = 250 units, 12 and C must be 150 units. Hence ratio of B to C by weight is 70 : 150, i.e. 7 : 15. Hence, (1).

hence the mixture must now be 100 ×

= 11 (

1 1 1 1 1 1 + + + + + + 1×3 2× 4 3×5 4×6 5×7 6×8

1 1 1 + + ) 7×9 8 × 10 9 × 11

= 11{



The numbers 5 5 , 6 6 , 7 7 , 8 8 can be re-written

and show that 5 5 > 6 6 > 7 7 > 8 8 . Now if we take their reciprocals, the inequality will reverse and hence

= 1 unit. On the first two days, (10 + 10) units of work is done. Next 2 days (12 + 12) units of work is done. Next 2 days (14 + 14) units of work is done. Next 2 days (16 + 16) units of work is done. Next 2 days (18 + 18) units of work is done. Next 2 days (20 + 20) units of work is done. ∴ Total work done in 12 days = 20 + 24 + 28 + 32 + 36 + 40 = 180 units. On the 13 th day, 22 units of work can be done. ∴ Work will be completed in 13 days. Hence, [5].

4.

 144    = 7.2.  110 

Hence, (1).

10 workers can complete the work in 20 days. Let the total work be 10 × 20 = 200 units. ∴ Work completed by 1 worker in 1 day =

11 1 1 1 11 (1 + – – )= 2 2 10 11 2

=

((55)15!)188 ≡ ((4)15!)188 ≡ [(42)x]188 ≡ (–1)even no ≡ 1, when divided by 17. So the remainder is 1. Hence, [1].

1 1 1 1 1 1 1 1 (1 – + – + – + – 2 3 2 4 3 5 4 6

8. +

132

After step 1, value of a = a + b After step 2, new value of = |(a + b) – b| = a Thus, after step 2 of every iteration, the value of ‘a’ remains the same. In step 3 of every iteration, the new value of ‘b’ is the difference between ‘a’ and the present value of ‘b’.

9.

Iteration 1: a = 5; b = 8 – 5 = 3 Iteration 2: a = 5; b=5–3=2 Iteration 3: a = 5; b = 5 – 2 = 3 Iteration 4: a = 5; b=5–3=2 Iteration 99: a = 5; b = 5 – 2 = 3 Iteration 100: a = 5; b = 5 – 3 = 2 Hence, (3).

13.

66 = 2×3×11, therefore, any factorial greater than 11 will be divisible by 66. Thus, the last 4 terms of the given expression (3! + 6! + 12! + 24! + 48! + 96!) give a remainder 0 when divided by 66. Now, the first two terms sum up to 3! + 6! = 6 + 720 = 726 = 66×11 which is also divisible by 66. The overall remainder is, therefore, 0. Hence, (1).

7 appears in the thousands place 770 times (>700).

14.

Total numbers in the given range = 7769 – 1001 + 1 = 6769 ( 2 × 42, Abundant f(84) = 1 + 2 + 3 + 4 + 6 + 7 + 12 + 14 + 21 + 28 + 42 + 84 = 224 > 2 × 84, Abundant. f(32) = 1 + 2 + 4 + 8 + 16 + 32 = 63 < 2 × 32, Deficient. Only option [4] contains one deficient and one abundant number. Hence, [4].

24.

B City Q

Let Alok and Bimol meet for the first time at A and for the second time at B. The time taken by Alok to reach A =

10 = 2 hours. 5

∴ The time taken by Bimol to reach A = 2 + 1 = 3 hours

Let P and Q be the centres of the circles in the upper and lower halves of the square respectively. m ∠ PBC = m ∠ PKC = 90°

134

⇒ m ∠ BPK = 180 – m ∠ BCK = 180 – 45 = 135°

i.e., 1 +

Similarly, m ∠ HPK = m ∠ KQD = m ∠ KQF = 135° and m ∠ BPH = m ∠ DQF = 90° Since the speed is constant, the ratio of the time taken = Ratio of the distance travelled Distance, B-K-F = (135 + 135)m = 270m and distance B-H-K-D-F = (90 + 135 + 135 + 90)m = 450m; where m =

∴ Required ratio =

i.e.,

n(n + 1) ≡ 1 (mod M) 2

n(n + 1) ≡ 0 (mod M) 2

n(n + 1) is divisble by M 2 M = 44. So we have to find the least value of n for

i.e.,

2π . 360

n(n + 1) is divisible by 44, 2 i.e. n(n + 1) is divisible by 88. The least value of n for which this is true is n = 32. Hence, (4).

which

27 3 = 45 5

Hence, [3]. 28.

The set consists of numbers 99 × 98 × 97, 96 × 95 × 94… 6 × 5 × 4, 3 × 2 × 1. Note that each of these numbers is divisible by 3. Also, every alternate terms has got two even factors, one of which is also divisible by 4. So these alternate numbers will be divisible by 2 × 4 × 3 = 24. There are 16 such terms. The remaining (alternate) numbers have got only one even factor. So they will be divisible by 24 if and only if this factor is also divisible by 8. So we have to look for numbers where the middle factor is divisible by 8. But the numbers are of the form 3k (3k – 1) (3k – 2) so we want to find factors of 8 which are of the form 3k – 1. They will occur after every 24 numbers – 8, 32, 56, 80. I.e. the numbers 9 × 8 × 7, 33 × 32 × 31, 57 × 56 × 55 and 81 × 80 × 79 will also be divisible by 8. So, in all, 16 + 4 = 20 numbers are divisible by 24. Hence, (1).

29.

The given statement can be restated simply as “If k is an odd no. divisible by 3 then n is not prime.” So if ‘n is prime’ then we know that k is ‘not an odd number divisible by 3’ (because in that case n would not have been prime). …… (i) But that does not imply that k is an even number not divisible by 3. E.g. k could be 5 or 6. So, [I] cannot be concluded. (2) cannot be concluded because the given statement does not put any condition on n if k is not an odd number divisible by 3. (3) begins with “If n is prime and k is divisible by 3” Let us see what we can conclude in this case – If n is prime we know from (i) that k is ‘not an odd number divisible by 3’. So if k is divisible by 3, it should not be an odd number. So, k is an even number divisible by 3. So k is divisible by 6. So, (3) can be concluded. (4) cannot be concluded because if n is not prime we cannot conclude anything about k. Hence, (3).

Arithmetic ªScore Maximiserª 25.

Given, the sum of the ages is 35 and each one’s age is a prime number. So check that following five combinations are possible: 3 13 19 (I) 5 13 17 (II) 5 11 19 (III) 5 7 23 (IV) 7 11 17 (V) Out of these, see that in the middle column (which represents the age of the middle brother) the numbers 11 and 13 appear two times, but 7 appears only once. Thus the age of the middle brother should be 7, for only then the ages of others can be uniquely obtained. In that case, the age of the eldest brother comes out to be 23 years. Hence, [3].

26.

f(83) = 6 83 + 8 83. Binomial = (7 – 1) 83 + (7 + 1) 83 On expanding f(83) using Binomial theorem, alternate terms cancel out and we are left with f(83) = 2[783 + 83C 2 7 81 + .... +83(7)] Now, all terms except the right-most are divisible by 49 = 7 2 . Hence, required remainder = remainder when (14 × 83) is divided by 49. On dividing, we obtain the remainder = 35. Hence, [5].

27.

Let us trace the path of the 1 st child’s cap (at the end of rounds 1, 2, 3 …) – 2, 4, 7, 11… At the end of the nth round, the 1 st cap will be with

 n(n + 1) the [1 + (1 + 2 + 3 + … n)] = 1 + 2 

  

th

child.

The cap will come back to the first child only when 1+

n(n + 1) is of the form Mk + 1, 2

135

30.

Let the time for a swing be t, the length be l, then we

33.

can say t = k

2a a = km. BC 2 2 and CK are the tangents to a circle from a common external point C. Then, BC = (a – 1)km and CK =

Time for the first pendulum of length 21 = 2 seconds per swing. Hence 2 = k

31.

32.

Let the side of the square ACEG be ‘a’ km.

and so k

a

For the second pendulum, t = k = x = 2 = Hence per 60 seconds, it will make = 45 swings. Hence (1).

∴ a–1=

Both x2 and 4y2 are non-negative and hence the maximum value each of them can take is 100. Now consider 4y2 < 100. Hence y can range from -5 to +5: For y = 0, x2 < 100 i.e x can range from -10 to +10 (21 values) For y = ±1, x2 < 96 i.e x can range from -9 to +9 (19 values) For y = ±2, x2 < 84 i.e x can range from -9 to +9 (19 values) For y = ±3, x2 < 64 i.e x can range from -8 to +8 (17 values) For y = ±4, x2 < 36 i.e x can range from -6 to +6 (13 values) For y = ±5, x2 < 0 i.e x can take only the value 0 (1 value) Thus, overall we can have 21 + (19 Î2) + (19Î2) + (17Î2) + (13Î2) + (1Î2) = 159 combinations. Hence, (2).

Distance B-C-K-G-F = 2 × BC + CG

⇒ a = (2 + 2 ) km.

= 2 × (1 + 2 ) +

In the latter case, Amit travels 2 2 km more and, hence, takes

2

= 2 hours more.

Hence, [1].

34.

un =

2 n2 − 1

=

1 1 − n −1 n +1

∴ u 2 + u 3 + u 4 + …. + u 100 = + .... + = 1+

= x(x 2 – 10) + (y – 1)y(y + 1) ... (i) For the possible values of x and y, we have 2

2 2 2

⇒ 273 = x(x 2 – 10) + y(y 2 – 1)

2

2 (2 + 2 ) = (4 + 4 2 ) km

Distance B-C-D-E-F = 2a = (4 + 2 2 ) km.

Let the number be 300 + 10x + y such that 0 < x, y < 9. Then, 300 + 10x + y = 27 + x 3 + y 3

x x x – 10 x(x – 10) 0 0 –10 0 1 1 –9 –9 2 4 –6 –12 3 9 –1 –3 4 16 6 24 5 25 15 75 6 36 26 156 7 49 39 273 8 64 54 432 9 81 71 639

2

1 1 1 1 1 1 − + − + − 1 3 2 4 3 5

1 1 1 1 − + − 98 100 99 101 1 1 1 − − ≈ 1.5 – 0.01 – 0.01 = 1.48 2 100 101

Hence, [4].

y (y – 1)y(y + 1) 0 0 1 0 2 6 3 24 4 60 5 120 6 210 7 336 8 504 9 720

35.

From the above table, the only possible values of x(x 2 – 10) and (y – 1) y(y + 1) which satisfy (i) are 273 and 0, i.e., x = 7 and y = 0 or 1. Hence, the numbers are 370 and 371. Hence, (3).

136

Numbers from 100 to 1000 include all the three-digit numbers and the number 1000. Akshay must have written only those three-digit numbers correctly which do not have 6 and/or 9 as their digit/s. The number of three-digit numbers not having 6 and/ or 9 as a digit = 7 × 8 × 8 = 448. Thus, Akshay wrote 448 + 1 = 449 numbers correctly in his assignment. Hence, [4].

Algebra

⇒ 2xy + 2yz + 2zx <

ªScore Enhancerª ⇒ xy + yz + zx <

36.

We know the x co-ordinate of the vertex. This is nothing but the arithmetic mean of the roots. ( Q Graph of a quadratic function is always symmetrical about a line parallel to the x or y-axis). ∴ If x 1, x 2 are roots of f(x) = 0 then,

39.

−b a

38.

⇒ x = 10[ (m 2 + 52 ) – 5] …………………for some 'm'

Since x is a natural number.,

we get

40.

To get the term a2c 2e2 we must select 2a’s, 2c’s and 2e’s each form a different bracket. Also, the coefficient of a2c2e2 is the number of ways in which this selection can be done. Now, ‘e’ cannot be selected from the first 4 brackets. So, ‘e’ must be selected from the last two brackets (only 1 way). After 2 ‘e’ have been selected 2 ‘c’s can be selected in 1 way (3 rd & 4 th bracket) and then 2 a’s can be selected in 1 way (first 2 brackets). So the coefficient of a 2c 2e 2 is 1. Hence, [1].

41.

f(0) = c = 4

1 –a–b 3

Then, 2

2

1  1  1  x 2 + y 2 + z 2 =  − (a + b )  +  + a  +  + b  3 3 3      

(144 + 25) = 13.

Also, x = 10 × (13 – 5) = 80. Thus, y = m 2 = 144. Hence, [5].

(x + y + z)2 = 1

1 1 + a, z = +b 3 3

( y + 52 ) should be a

natural number. Thus, we need to find a Pythagorean triplet which includes 5. (3,4,5) and (5,12,13) are the only possible Pythagorean triplets. Using the first triplet, we do not get a natural no for the above expression. If m = 12,

As the values of a, b, c cannot be uniquely determined, we cannot find the minimum value of f(x). Hence, [5].

then x =

x2 100

⇒ (x + 50) = 10 ( y + 25)

c −42 = = –14 a 3

Let y =

x2 100

⇒ x 2 + 100x = 100y ⇒ x 2 + 100x + 2500 = 100y + 2500 ⇒ (x + 50) 2 = 100(y + 25)

∴ Product of roots = –14. Hence, [2]. 37.

x% of x =

Therefore, y = x +

∴ b = 5a .... (1) Now, f(–2) = –2f(3) ∴ 4a – 2b + c = –2[9a + 3b + c] = –18a – 6b – 2c ∴ 22a + 4b + 3c = 0 .... (2) Substituting (1) in (2) we get 42a + 3c = 0 ∴

1 3

Hence, [3].

x1 + x 2 = –2.5 2

∴ x 1 + x 2 = –5 =

2 ( Q (x + y + z) 2 = 1) 3

2

=

1 2 1 2 – (a + b) + (a + b) 2 + + a + a2 9 3 9 3

+

1 2 + b + b2 9 3

f(x) = (x +

=

1 2 – (a + b – a – b) + a 2 + b 2 + (a + b) 2 3 3

So the minimum value of f(x) is when x =

∴ x2 + y2 + z2 = ⇒ x2 + y2 + z2 >

1 + a 2 + b 2 + (a + b) 2 3

b 2 b2 ) + (4 – ). (Rearranging the terms). 2 4

value is 4 – So b = +6.

1 3

137

−b and the 2

b2 . Equating this to –5 we get b2 = 36. 4

b 3 3 can be either − or but cannot c 2 2 be determined uniquely. However, we can observe that f(x) = x 2 + 6x + 4 = (x + 3)2 – 5 or x2 – 6x + 4 = (x – 3)2 – 5. So the roots

⇒ (3y + 2) 2 = 0 ⇒ y = −

So the value of

are either –3 + 5 or 3 +

Thus, xy =

We know x =

Let the number of correct answers be c and the number of wrong answers be w. Hence we need solutions to 4c – w = 15 (where c and w are non-negative integers and c + w < 20). The possible cases are:

From each option, we get the value of xy and, hence, we can find the value of y. Substituting the values of x and y in the given expression, we can get the correct answer.

4 1

5 5

6 9

48.

7 13

x

The maximum possible combined score is 88 if both get all right. For every question that either one gets wrong, the effect is of -5 marks (going from a + 4 to a –1) and hence the total score must always be of the form (88 – 5k). 22 is not of this form. Hence, (1).

Answers to questions 44 and 45: Let the distance from A to B be d. Hence the fare for Mr Pant is k + 1.5 (d – 100) = 1100, while that for Mr Khot is 2k + 1.5 (d – 200) = 1150. This gives us k = 200 and d = 700. 44 – (3), 45 – (1).

47.

Let the two digit number be xy. It can be considered as 10x + y. The reversed number will be 10y + x and hence we can say that (10x + y) – 2(10y + x) = 13, i.e. 8x – 19y = 13. But x and y both must be digits, i.e. integers between 0 and 9, and hence the only values which satisfy the above condition are x = 4 and y = 1. The original number is therefore 41, which is prime. Hence, (2).

Let

49.

2

x

9y + 12y + 7 3

⇒ (2k – 3) 2 = 0 ⇒ k =

x

9y 2 + 12y + 7 3

Now, x =

x2

p 1 = q = 3p; then, q 3 ⇒

= k; then,

p+6 p+6 2 2 = ⇒ 3p + 6 = 3 ⇒ p = 2 and q = 6. q+6 3

4k 2 – 12k + 9 = 0



Let the two-digit ‘zeroth number’ be 10x + y. Then, 1 < x < 9, 0 < y < 9 and x 2 + y 2 = 10k.

possible y2 must values of y end in 1 1 9 3, 7 2 4 6 4, 6 3 9 1 1, 9 4 16 4 2, 8 5 25 5 5 6 36 4 2, 8 7 49 1 1, 9 8 64 6 4, 6 9 81 9 3, 7 ∴ The required number = 17. Hence, (4). Alternatively, From 1 to 9, there are two sets of numbers whose squares end in 1, 4, 6, 9. Thus, for every 1, there are two 9s; for every 4, there are two 6s; and so on. ∴ 8 × 2 = 16 such pairs exist. For 5, the possible number is 55. Thus, the total number of ‘zeroth numbers’ = 16 + 1 = 17.

Thus, there are four ways in which a score of 15 can be achieved and hence there could be a maximum of four people in the group. Hence, (2).

46.

3 . 2

Hence, (1).

c w

43.

3 −2 × = –1. 2 3

Hence, (2). Alternatively,

5 . In either case, the

difference between the roots is 2 5 .

42.

2 3

=

3 2

Now,

3 . 2

q +1 1 = ⇒ r = 13 r +1 2

r +2 3 s+4 3 = ⇒ s = 8 and = ⇒ t = 12. s+2 2 t +4 4

3 9y 2 + 12y + 7 =1 ⇒ 2 3



138

p + q + r 21 = . Hence, (4). s+t 20

Algebra

2

y corresponding to   = 0 2, 1 2, 2 2… 10 2. 2

ªScore Maximiserª

50.

So, x can take 11 possible integer values. Hence, (1).

1 An+1 = A n +   (n – 1). So, 5 53.

1 A55 = A 54 +   (53) 5 1 A54 = A 53 +   (52)… 5

=

1 A7 = A 6 +   (5) 5

= =

1 A6 = A 5 +   (4) 5 1 1 A55 = A 5 +   (4 + 5 + … + 53) = 55 +   1425 5 5 = 340. Hence, [2].

54.

f(0) = c = 4 b

f(x) =(x+ 2 )2 + (4 –

b2 ). (Rearranging the terms). 4

So the minimum value of f(x) is when x =

value is 4 –

⇒a=

−b and the 2

So b = +6. b 3 3 can be either − or but cannot c 2 2 be determined uniquely. However, we can observe that f(x) = x 2 + 6x + 4 = (x + 3)2 – 5 or x2 – 6x + 4 = (x – 3)2 – 5. So the roots

So the value of

52.

x=

ªScore Enhancerª 55.

Hence, (4).

1 + 2b and b + 1 < 51 ⇒ b < 50

Geometry

5 . In either case, the

difference between the roots is 2 5 .

Let c = b+1 then the equation becomes a2 + b2 = (b + 1)2 Since, a is a natural number, 2b + 1 should be a perfect square. Since, 2b + 1 is odd, the values should also be odd. Satisfying b < 50 and the possible values of 2b + 1 are 1, 9, 25, 49 and 81. Since b is a natural number, 1 is not a possible value. Therefore, the triplets (a, b, c) will be (3, 4, 5); (5, 12, 13); (7, 24, 25); (9, 40, 41) Hence, there are 4 possible triplets with the given conditions. Hence, (2).

b2 . Equating this to –5 we get b2 = 36. 4

are either –3 + 5 or 3 +

63 + Now the denominator of the fraction above has 2010 terms out of which the minimum is 622009. Hence the value is greater than 2010 × 62 2009. This value is definitely greater than the numerator, which is only 62 × 62 2009. Thus this fraction is less than 1 and m is between 63 and 64. Hence, (3).

Adding these we get:

51.

=

m=

4 − y2 ⇒ y2 = 4(1 – x) ⇒ 1 – x should be a perfect 4 2

y square =   2

E B

C

G

F

A

D

As areas of the kites are equal,

So, possible values of x are 1, 0, –3, –8, –15, –24… –99

139

1

1 2 × GE × BD = 2 EF × AC ⇒ GE = EF

This implies that E must be the mid point of BC ( Q CE = CF, BE = BG). ∴ A( ∆ BAE) = A( ∆ EAC) and

The length along the curved arc from C to A will subtend an angle of 120° at the centre (i.e. it is one third of the circumference) and hence will have a length 2 π r/3. Hence the ratio of speeds of ants at C and at A will be the ratio of distances travelled in the same time, i.e. 2 π r/3 / r = 2 π /3. Hence, (4).

A( ∆ CAF) = A( ∆ FAD) ∴ Area of square = 2 × Area of kite = 200 sq. units. Hence, [1]. 56.

If 50% of the water present in the tank is removed, then the water level will also decrease by 50%. Let the initial water level be ‘x’ m. ∴ decrease in water level =

59.

As seen in the above answer ∠ Y = 30°. Hence, (2).

60.

Let the radii of the two smaller circles be a and b. Hence the radius of the larger circle will be a + b. ∴p = πa2, r = πb2 and the total area of the larger circle will be π(a + b)2. The remaining area q + s = π(a + b)2 – πa2 – πb2 = 2πab. Now, from the symmetry of the figure, q = s = πab. We can see that pr = qs = π2a2b2. Hence, (2).

x m. 2

x = 30% of 8 2 ∴ x = 4.8 m Hence, (3).

Thus,

61. 57.

C P A

O S

X

Q

If we draw triangle ABC and take the midpoint of the longest side (say BC) as a point D, then we can see that we form two right triangles as shown in the figure, Hence length of AD must be 3 cm (which becomes the height of the triangle) and hence area of triangle ABC = ½Î3Î8 = 12.

B

M R

Y D

We also know that the area of a triangle =

where

a, b and c are the lengths of the sides and R is the PQ = QX = 2 cm ⇒ MQ = 2 2 cm. Similarly,

circumradius. Hence we can say that 12 =

MS = 4 2 cm. So, XY = XQ + QM + MS + SY = 2 + 2 2 + 4 2 + 4 = 6 + 6 2 cm. So, OX =

gives us R =

1 XY = 3( 2 + 1) cm. 2

Hence, (4).

62.

58.

D

B

= 4.166 cm. Hence, (2).

AC = BC ⇒ m ∠ CAB = m ∠ CBA Also, OA = OB ⇒ m ∠ OAB = m ∠ OBA ∴ m ∠ OAC = m ∠ OBC ∴ ∆ CAF ≅ ∆ CBE (by ASA test) ∴ AF = BE = BO + OE = 7 + 3 = 10 cm. Hence, (3).

A

C

which

E

Consider the point D at the 9 o’clock position. Each of the arcs AB, BC and CD have the same length and the same central angle (60°). Hence by inscribed angle theorem, ∠ DAC, ∠ CAB and ∠ BAE are all 30°. Hence if the radius of the clock is r, l (AD) = 2r and hence

63.

l (AC) will be r 3 .

Let m ∠ ACB = x°, then as BC = BD m ∠ BDC = m ∠ BCD = m ∠ ACB + m ∠ ACD = (x + 20)° ⇒ m ∠ CBD = 180° – 2 × (x + 20)° = (140 – 2x)° (sum of all angles of a triangle is 180°)

Now, in ∆ ABC, BC = AC ⇒ m ∠ BAC = m ∠ ABC = 40 + m ∠ CBD

140

= (180 – 2x)° and 2 × (180 – 2x) + x = 180°

66.

C

⇒ 3x = 180° ⇒ x = 60°

∴ m ∠ BAC = 180° – 2x = 180° – 120° = 60°. Hence, [3].

A

Note that the question asks which statements are ‘necessary’ and NOT which statements are ‘sufficient’. Now if ∠ ACD is a right angle, AD will be the diameter of the circle that inscribes ∆ ACD. So, BC = AB = BD as all denote the radius of this circle. So, ∆ ABC has two sides (AB and BC) equal. So, I is necessary (although not sufficient) for ∠ ACD to be a right angle. However for ∠ ACD to be a right angle, ∠ CBD need

Geometry ªScore Maximiserª 64.

Observe that each odd integer > 2 is part of at least one Pythagorean triplet. (Suppose k is an odd number then k2 is also odd. Let k2 = m2 + (m + 1)2. Then, clearly, k, m and m + 1 form a Pythagorean triplet). Numbers 1 and 2 are not part of any of the triplets. What remains is all even numbers > 3. See that an even number is either a power of 2 or a multiple of an odd number. So all those even numbers, which are multiples of odds are also involved in some triplet. 4 is also a member of one triplet i.e., (3, 4, 5) so all of its multiples; and thus all powers of 2 (excluding 2 itself) are also involved in some triplet. So from the set of Natural Numbers, only 1 and 2 are not members of any triplet. So we need to draw triangles, having sides from {1, 2}. Note that only three distinct triangles can be drawn. Triangles with: (I) Sides 1, 1, 1 (II) Sides 2, 2, 2 (III) Sides 1, 2, 2 Hence, [3].

D

B

not necessarily be 120 o. E.g. We can have ∠ A, ∠ B, ∠ C = 50 o, 80 o, 50 o respectively and ∠ CBD = 100 o and ∠ ACD = 90 o. Hence, (1).

67.

A

B

P O

Q

Top View x

65.

1 AB = 2.4 m. OA = 3 m. So, OP 2 = (3) 2 – (2.4) 2. 2 So, OP = 1.8 m. So the maximum distance he can walk in the same direction = OQ = 7.2 – 2 (OP) = 3.6 m. Hence, (3). AP =

x 6-x 6-x Note that if we take the sides of the cubes as 'x' and '6 – x' and try to maximize x3 + (6 – x) 3 = 16 – 108x + 18x 2 then we can observe that this is an upward sloping quadratic (a > 0) and so, will attain maximum (in the given range) for the maximum possible value of x. Now x can be at most 4 (as one of the sides of the cuboid is 4). So the maximum possible total volume = 43 + 23 = 72 cu. m. Hence, [4]. Alternatively, We can consider only the 2 extreme possibilities: (i) Both cubes are of side 3m → volume = 3 3 + 3 3 = 54 cu. m and (ii) The cubes are of sides 4m and 2m → volume = 4 3 + 2 3 = 72 cu. m.

68.

141

A

B E

H D

G

C

There are two possible candidates for the largest equilateral triangle, ∆ DEC and ∆ BFG as shown in the figure. ∆ DEC clearly has a side of 1. Let the side of BFG be x. Now FDG is a 45-45-90 triangle and hence l(HD) will be

x . Also HB will be the height of the 2

equilateral triangle and hence l(HB) will be

Let the side of the square be 2a m, then PQ = 2a m, QC = a m and OQ = (2a – 3) m

x 3 . 2

∴ In the right-angled ∆ OQC, OC 2 = OQ 2 + QC 2 i.e., 25 = (2a – 3)2 + a 2

x x 3 Hence l(BD) will be l(HD) + l(HB) = + . But 2 2 l(BD) =

 1+ 3  we can say that x  2  = and hence x =  



⇒ 4a 2 + a 2 – 12a + 9 – 25 = 0

2 as BD is the diagonal of the square. Hence

⇒ 5a 2 – 12a – 16 = 0

 2 2     1+ 3   

+ 12 ± 144 + 4 × 5 × 16 2×5

⇒ a =

2.828 ≈ 1.03. Hence, (3). 2.732

⇒ 2a =

12 + 4 29 12 + 4 25 32 ≈ = = 6.4 m 5 5 5

The closest option is [4]. Hence, [4].

69.

Modern Mathematics ªScore Enhancerª 71.

x 1 Thus, x + y + 2 = 2

By symmetry, the longer diagonal AC of the kite is the diameter of the circular sheet. The lengths of the sides of the kite are in the ratio 3 : 3 : 4 : 4. Let AB = 3x, then BC = 4x ∴ A( ABCD) = 2 ×

⇒ 2x = x + y + 2 ⇒ x = y + 2. y

Also

1 × AB × BC 2

= 3x × 4x = 12x and the area of the circular sheet

1 C 2 = 11

C2 1 = C2 11

2y + 4

∴ 11y(y – 1) = (2y + 4)(2y + 3) Thus 7y 2 – 25y – 12 = 0 ∴ (7y + 3)(y – 4) = 0

5x 275 2 22 5x × = x × 2 14 7 2

−3 or y = 4 7 But y cannot be –ve. So y = 4 ⇒ x=4+2=6 ∴ Total number of balls = 6 + 4 + 2 = 12. Hence, [3]. ∴ y =

12 ∴ The required percentage = 100 – 275 × 100 14

=

C

2 x+ y+2

y

i.e.,

2

= πr 2 =

Let number of red balls be x and number of blue balls be y. So in all there are x + y + 2 balls in the box.

107 428 ≈ 39% . × 100 = 275 11

Hence, [3].

72.

70.

EF = 8 m ⇒ PF = 4 m (perpendicular from the centre bisects EF at P)

142

t10 = t 8 + t 9 ... (i) ∴ t 8 = t 10 – t 9 ∴ t 10 – t 9 = 124 Also, t 11 = t 10 + t 9 ... (ii) ∴ t 10 + t 9 = 430 Solving (i) and (ii) simultaneously. 2(t 10) = 554 ∴ t 10 = 277 ∴ t 9 = 153 Now, t 12 = t 11 + t 10 = 430 + 277 t12 = 707 Hence, [2].

... (i)

73.

log ba +

series as the second level differences are the same. Alternatively, we can see that f(n+2) = f(n) + 2n + 3 = f(n) + (n +1) + (n+2). But we also have f(1) = f(0) + (0+1). Putting these together we can show that for all whole numbers n, f(n+1) = f(n) + (n+1).

1 log a b +

= log ba +

= log ba +

1 log a b +

1 log a b + log

a2

1 log a b

Either way, we can show that f(n) =

b

Hence (1) is true. f(4) = 11, f(3) = 7 and f(2) = 4, f(4) = f(3) + f(2). Hence 3f(4) = 3 f(3) x 3 f(2) .So (3) is true. Hence (2) is the only statement which is not necessarily true for all n. Hence (2).

1 log a b +

1 log a b 2

2 2 = log ba + 2 log b + log b = log ba + log b3 a a a = logb a + 2 logb 3 a = log ba +

77.

2 log ba 3

3 3 ∴ b5 = a 5 Now b = 32

78.

f(a + b) = f(a) + f(b) + 2 ab So f(2+3) = f(2) + f(3) + 2 × 2 × 3 i.e. 35 = f(2) + 15 + 12 Hence f(2) = 8 Now f(1+2) = f(1) + f(2) + 2 × 1 × 2 i.e. 15 = f(1) + 8 + 4. Hence f(1) = 3. Hence, (2).

79.

We can see that f(1) = 3 = 1 x 3, f(2) = 8 = 2 x 4, f(3) = 15 = 3 x 5 and so on. We can note that f(n) = n(n+2) = n2 + 2n. So, f(n) + 1 will be nothing but n2 + 2n + 1

3

= (32) 5 = a ⇒ (32 115)3 = a ⇒ 2 3 = a ∴ a = 8. Hence, [1]. 1 (3+7) = 5 2 T2 = S 2 – S 1 = 13 – 5 = 8 Since it is an A.P. the common difference is 3 So the difference between 10th and 15 th terms is 5 × 3 = 15. Hence, (1). Alternatively, We can find the 10th and 15th terms as T10 = S10 – S9 T1 = S1 =

=

= 35.

Hence, (4). 80.

If F(k) = G(k) then G(k) – F(k) = 0. This implies that (2k3 + k2 +7k + 15) – (2k3 – 3k2 – 5k + 7)=0 i.e. 4k2 + 12k + 8 = 0 or k2 + 3k + 2 = 0 i.e. k = -1 or k = -2. The required product is 2. Hence, (1).

1 (3 × 152 + 105 – 3 × 142 – 98) = 47 2 So, difference = 15.

76.

i.e. (n+1)2. Thus k =

1 (3 × 102 + 70 – 3 × 92 – 63) = 32 2

T15 = S15 – S14 =

75.

10 = log 10 – log 2 = 1 – 0.3= 0.7 2

Hence, (4).

∴ log ba =

=

log 4 = 0.6 Thus, 2 log 2 = 0.6 giving log 2 = 0.3 log 5= log

1 5 5 = log b a = log b a 3 3 This is given to be equal to 1.

74.

1 2 (n + n + 2). 2

For the graphs to intersect, we must have 2 log x = log 2x, i.e. log x2 = log 2x, i.e. x2 = 2x Note that the graph of log x is defined only for x > 0 So, x 2 = 2x ⇒ x = 2 So the graphs intersect at exactly one point (2, log 4) Hence, (2).

81.

P(3, 2) = 1 + 3 + 3 2 = 13 while Q(2, 3) = 1 – 2 + 2 2 – 2 3 = –5. ∴ J = P(3, 2) × Q (2, 3) = 13 × (–5) = – 65. Hence (3).

82.

P(x, m) will always be positive since x > 0. So the sign of the product depends on Q(y, n). Now Q(y, n) will be negative if y > 1 and n is odd (so that the last term is negative and of greater magnitude than the preceding term) which could be true in any of the first three statements. But if n is even, Q (y, n) will definitely be positive and so P(x, m)ÎQ(y, n) will always be positive. Hence, (4).

We can just observe the pattern f(0) = 1, f(1) = 2, f(2) = 4, f(3) = 7, f(4) = 11 and so on which is a quadratic

143

83.

If there are n children, there would be nC 3 triangles. and a + (q – 1)d =

n(n − 1)(n − 2) Hence C3 must be divisible by 3. Hence 6 is divisible by 3 or n(n – 1)(n – 2) is divisible by 18, i.e. by 9 and 2 both. Now when 3 consecutive numbers are multiplied, the product will always be divisible by 2. Also when three consecutive numbers are taken, exactly one of them will be divisible by 3, and in this case that number must be divisible by 9 also. So, either n or n – 1 or n – 2 must be divisible by 9. Hence, (4). n

84.

85.

86.

(i) – (ii) gives (p – q)d =

⇒ d =

∴ S pq =

m [10 + (m – 1)10] = 6125 2 m [10m] = 6125 ⇒ m 2 = 1225 ⇒ m = 35. 2

Let the first term and the common difference be ‘a’ and ‘d’ respectively. Then, a + (p – 1)d =

1 q

=

pq  2 + (pq − 1) 1  pq  1 + 1     pq  = 2  pq 2  pq 

=

pq 1 1 + = (pq + 1) . Hence, (2). 2 2 2

From the given data, we get the following Venn diagram:

⇒ n = 10. Thus, for every bouquet, he uses 10 more roses than the number of roses used for the previous bouquet. Let the number of bouquets made, when 6125 roses are used, be m; then,

87.

pq [2a + (pq – 1)d] 2

89.

25 [2a + (25 – 1)n] = 3125 2

∴ He can make 35 – 25 = 10 more bouquets. Hence, (2).

1 1 1 ⇒ a= = pq q pq

Only 2-digit, 3-digit, 4-digit or 5-digit numbers can satisfy the two given conditions. For each of the above cases, there are possibilities for the leftmost digit, namely 3, 5 and 7, and two possibilities for the rightmost digit, 4 and 9. Therefore, regardless of the number of digits, we will have 6 different combinations for the first and last digits taken together. Now, depending on the number of digits, we can find the total number of combinations that can be formed with that many digits. For a 5-digit number, there are3C3 × 3! possible arrangements for the remaining three digits. Hence, a total of 6 × 6 = 36 arrangements are possible. For a 4-digit number, for each of the above 6 combinations, there will be 3C2 ×2! possibilities for the remaining 2 digits. Hence, a total of 6 × 6 = 36 numbers are possible. For a 3-digit numbers, we will have 3C1 × 1! possible arrangements for the remaining digit. Hence, a total of 6 × 3= 18 numbers are possible. For a 2-digit number, 6 arrangements are possible. Hence, a total of 36 + 36 + 18 + 6 = 96 arrangements are possible. Hence, (3).

⇒ 25[10 + 24n] = 6250



1 1 – q p

88.

The number of roses in the first bouquet = 5. The number of roses in each subsequent bouquet is 5 + n, 5 + 2n, 5 + 3n, ... This forms an A.P. with a = 5 and d = n. Since the total number of roses used in 25 bouquets is 3125, S 25 =

... (ii)

1 pq

∴ a + (p – 1)

Each boy has to meet n girls and will require at least 4n minutes. Similarly each girl has to meet m guys and will take a minimum of 4m minutes. But m > n and hence 4m > 4n; thus the time needed for the event would be 4m minutes. Hence, (1). The only constraint here is that all vertices of the triangle cannot be selected from the points on one parallel line. Selecting 3 vertices randomly from the field = 15C3 = 455 Selecting 3 vertices from 3 points on line A =3C3 = 1 Selecting 3 vertices from 4 points on line B =4C3 = 4 Therefore total number of triangles that can be formed is = 455 – (1 + 4) = 450. Hence, (1).

1 p

... (i)

144

Since k is even, 4K is divisible by 8 and hence 4k + 4 is never divisible by 8. Hence true. Option (2): F(n) is always a perfect square and hence has an odd number of factors. Hence true. Option (3): For an even number F(n) = n2. Hence F(4n) = 16n 2 while F(2n) = 4n2. Hence true. Option (4): If m is even (e.g. 6) while n is odd (e.g. 5), this will not hold true. Hence false. Hence, (4).

From the Venn diagram, we get, 21 + 4 + x + y = 32 ⇒ x + y = 7 …(i) 12 + 4 + y + z = 24 ⇒ y + z = 8 …(ii) From (i) and (ii), (x + y + z) + y = 15 ⇒ x + y + z = 15 – y Thus, the total number of respondents surveyed (say N) = 21 + 12 + 14 + 4 + x + y + z = 51 + 15 – y = 66 – y

Now, from (i), 0 < y < 7 ⇒ 59 < N < 66. Hence, [2].

93.

F(36) = 36 2 while F(31) = 302. Hence F(36) – F(31) = 362 – 30 2 = (36 + 30)(36 –30) = 66 × 6 =11 × 3 × 3 × 2 × 2 This will be divisible by 9, 22 and 33, but not 15. Hence, (2).

94.

We have 28 < x 2 + 3x < 40. Solving the first part we get: x2 + 3x – 28 > 0 => (x + 7)(x – 4) > 0 => x > 4 or x < -7. Solving the second part we get: x2 + 3x – 40 < 0 => (x + 8)(x – 5) < 0 => -8 < x < 5 . Combining both we get: -8 < x < -7 or 4 < x < 5 So, 49 < x 2 < 64 or 16 < x 2 < 25. Hence, (2).

95.

The man could have climbed 8 steps by tossing 8 Tails (1 way), or 6Tails and 1 head ( 7C1 = 7 ways) or 4Tails and 2 Heads ( 6C2 = 15 ways) or 2 Tails and 3 Heads (5C3 = 10 ways) or 4 Heads (1 way), leading to a total of 34 possible sequences in all. Hence (4).

96.

P(x, 8) / Q(x, 8) = (1 + x + x 2 + x 3 + . . . + x 8)/(1 – x + x2 – x3 + . . . + x8)

Modern Mathematics ªScore Maximiserª 90.

91.

For all x such that |x – 3| < 0.5, x must lie in the range 2.5 to 3.5. Now for all real numbers such that 2.5 < x < 3.5, we have 6.25 < x2 < 12.25 and –2.75 < x 2 – 9 < 3.25. So, |x 2 – 9| < 3.25 …… (i). Note that we want the inequality |x2 – 9|< K to hold for ALL real values of x in the range 2.5 to 3.5. So one should not be tempted to mark (2) just because it is a ‘smaller value’ because the inequality |x2 – 9| < 2.75 will not hold for all the required values of x. E.g. For x = 3.49, we have |x – 3|= 0.01 which is < 0.5, but |x2 – 9|= 3.18 which is not less than 2.75. So the smallest value that K can take is 3.25 (from (i)). [It is the ‘smallest’ value because the inequality would also hold for K > 3.25, say for K = 7.]. Hence, (4). 5 digits out of 9 digits can be selected in 9C 5 ways. Now, there are 2 possibilities: Digits in Number 1

I II

Selection/ Arrangement

Digits in Number 2

Selection/ Arrangement

Total no. of possible pairs

C1 = 5

4

4! = 24

5 × 24 = 120

C2 × 2! = 20

3

3! = 6

20 × 6 = 120

5

1 2

5

97.

Total number of pairs using 5 selected digits = 120 + 120 = 240 ∴ Total number of pairs that can be formed using any 5 out of 9 digits = 9C5 × 240 = 30240 Hence, (4). 92.

=

Let us observe the pattern: F(1) = 0 F(2) = 4 F(3) = 4 F(4) = 16 F(5) = 16 F(6) = 36 F(7) = 36 F(8) = 64 F(9) = 64 F(10) = 100 Thus if n is even, F(n) = n2, while if n is odd, F(n) = (n – 1) 2 Option (1): F(n+2) – F(n) will always be of the form (k+2) 2 – k 2 where k is even. This is nothing but 4k+4.

=

log 2

+ log 3

= log {

Î

= log {

Î

. Hence, (1).

+ log 4 Î }

=

+ log 5

Î…Î

… (n terms)

}

log {(n + 1)!}3 . Hence, (4).

98.

Each of the n girls has 2m ways of writing her list (number of ways of selecting some or all out of m distinct objects). Each girl chooses independently of the others, and hence the total number of ways would be (2 m)n = 2 mn. Hence, (3).

99.

4g(x)2 = f(x) 2 + 2f(x)f(–x) + f(–x)2

145

⇒ 4g(x) 2 = [f(x) + f(–x)] 2 ⇒ g(x) =

⇒ g(–x) =

 f(x) + f( − x)  f(x) + f( −x) or –   2 2    f( − x) + f(x)  f( −x) + f(x) or –   2 2  

Australia was declared winner of group C. Similarly, in Group A, as S.Korea was declared runnerup it must have won one game and lost the other. The only possibility is S.Korea beat Argentina (upset), Netherlands beat both Argentina and S.Korea. Thus, the winner of Group A was Netherlands. Hence, [1].

i.e., g(–x) = g(x) ∴ If g(4) = 20 ⇒ g(–4) = 20 Hence, (1). 100.

Let log xy = a; then the given equation becomes a2 – 12a – 45 = 0. Solving it, we get a = 15 or a = –3.

104.

If New Zealand lost both matches in league 1, it means that the Spain-India game was an UPSET. : India ∴ In Group B, Winner Runner-up : Spain Similarly, if France lost both matches in League 1, it means that the France-China game was an UPSET and Germany won both matches. : Germany ∴ In Group D, Winner Runner-up : China ∴ From answers of 26 and 27 we get, Group E: Netherlands, Australia, Spain, China Group F: India, Germany, S.Korea, Pakistan Hence, [1].

105.

Device C requires one unit of Chip 4 and Chip 5, thus the company can use M2 to produce Chip 4 and M4 and M5 to produce Chip 5. In a day M2 produces 152 functional units of Chip 4 and M4 and M5 produce 276 functional units of Chip 5 combined, thus the daily requirement of 100 units of device C will be met and extra 52 units will also be produced. Within 7 days 1000 units of Chip 4 and Chip 5 will be produced, thus fulfilling the total contract. Hence, [4].

106.

The column ‘Diff’ denotes the difference in the number of points scored by the team and points scored against that team. So the sum of all the entries in this column should be 0. So, ‘Diff’ for team B is ‘–1’. Since the points scored by A, C and D against B are 15, 10, 15 respectively, the total points against B are 40.

For a = 15, log xy = 15 ⇒ x 15 = y ⇒ As y > x, x 15 > x ⇒ x 14 > 1 ⇒ x > 1 and hence, y > 1 (as y > x > 0)

For a = –3, log xy = –3 ⇒ x –3 = y As y > x, x –3 > x ⇒ x 4 < 1 ⇒ x < 1 and hence,

1 x3

>1

⇒ y > 1 (as y > x > 0) Thus, among the options the only possible value of y is 8. Hence, [3].

101.

102.

For the number of respondents travelling by cab to be the least among all the three categories, it should be maximum 23. Thus, we get three possibilities for the least number of respondents travelling by cab: I. 19 when x = 0, z = 1 and y = 7 II. 21 when x = 1, z = 2 and y = 6 III. 23 when x = 2, z = 3 and y = 5 and, hence, three possibilities for the total number of respondents. Hence, [3]. The movie Raju watched twice can be selected in 6C 1 = 6 ways. Out of the five shows of this movie, Raju can select any two in 5C2 = 5 × 2 ways. The remaining three movies he can see in 5C 3 × 3! = 5 × 4 × 3 ways. Thus, Raju can see the movies in (5 × 4 × 3)2 ways.

5! 5×4×3×2 =5×4×3× 2! 2 = (5 × 4 × 3) 2 Hence, [4].

60 ×

A B C D

A – –15

B 15– – 10–15 15–

C

D

15–10 –

–15

Total 40– 39–40



40–41

Diff 5 –1 –3 –1

Win–Loss 2–1 1–2

Since A has 2 wins and 1 loss, he must have won against one of C and D and lost against the other. If A wins against C, then C must win against D (1 win)

Data Interpretation

So, we get 2 cases: [Let A

ªScore Enhancerª 103.

From this we get a partially filled table as follows:

It is given that there was one upset in each of the groups in league 1. Now, Pakistan is the runner-up of Group C. This means it won 1 game and lost the other. If there is only one upset in the group the only possibility is that England beat Australia (upset), Australia beat Pakistan and Pakistan beat England. Hence, all 3 teams had same points at the end of league 1 and based on its WHR,

146

↑ D denote A wins against D]

Case (i) A ↑ C, C ↑ D, D ↑ A and Case (ii) C ↑ A, D ↑ C, A ↑ D. Consider Case (i), A scored 15 points against B and C each. Since his total points are 40, he must have scored 10 points against D.

So, C and A scored 15 and 10 points against D.

Summarizing, we get,

But total points against D = 41, so B must score 16 points against D which is not possible. So case (i) is impossible.

Batsman MH SKR JPD YS JHK

We fill the table according to case (ii) as follows:

A B C D

A – 10–15 15–10 10–15

B 15–10

C 10–15 15–10 – 15–12

– 10–15 15–14

D 15–10 14–15 12–15 –

Total 40–35 39–40 40–41

Diff 5 –1 –3 –1

Win–Loss 2–1 1–2 1–2 2–1

Hence total doubles scored =

108 × 1600 = 480 360 480 Hence, total triples scored = = 160 3 36 × 1600 = 160 Runs scored in fours = 360

Let the runs scored by MH in singles be x ∴ 424 = x + 2 × 10 × 5 + 3 × 10 × 5 + 4 × 10 + 6 × 10 ∴ x = 74. Hence, [2].

110.

Let the number of singles, doubles and triples be x. 298 = x + 2x + 3x + 10 × 4 + 20 × 6 298 = 6x + 160 6x = 138 x = 23. Hence, [3].

111.

C scored 6 points. Hence, [3]. Two combinations for D exist

For case I: Number of draws for A, C, D, F are as follows:

160 = 40 4 54 Runs scored in sixes = × 1600 = 240 360 Hence, total fours hit =

Hence, total sixes hit =

240 = 40 6

From statement 2: it is clear that MH, SKR, YS and JHK hit ten fours each. From statement 4: it is clear that YS hit

80

109.

320 = 160 2

Runs scored in triples =

72

Number of doubles scored = 160. 160 = 80 + 54 + x x = 36 Any of the three remaining players could have scored a maximum of 36 doubles. Hence, [1].

90° × 1600 = 400 360° 72° × 1600 = 320 3600°

54

108.

Total runs scored by the five batsmen

Runs scored in doubles =

0

Triples Fours Sixes 10 10 54 10 0 0 10 10 20 0 10 0

JHK scored 72 singles. Hence, [3].

Answers for questions 107 to 110:

Runs scored in singles =

Doubles

107.

Hence, [1].

= 424 + 310 + 296 + 298 + 272 = 1600

Singles

For case II: Number of draws for A, C, D, F are as follows:

2 × 40 = 20 sixes 4

From statement 3: Runs scored by JHK in singles = 272 – 160 – 40 = 72 From statement 1: Runs scored by SKR in doubles and triples = 310 – 40 = 270 Let the number of doubles and triples be x. 2x + 3x = 270 5x = 270 x = 54

147

Answers to questions 112 and 113:

116.

Let C be the number of correct attempts and W be the number of wrong attempts. From the options we have For [1]:For score of 70 (C.W)= (18,2) or (19,6) For [2]: For score of 68 (C,W)= (17,0) or (18,4) For [3]: For score of 55 (C,W)= (14,1) or (16,5) or (17,9) For [4]: For score of 60 (C.W)= (15,0) or (16,4) or (17,8) For [5]: For score of 63 (C,W)= (16,1) or (17,5) Hence, [4].

117.

A score of 20 is possible if (C,W)= (5,0), (6,4), (7,8), (8,12) and (9,16). Hence, at most 5 students could have

From the given data, we have

Junior Senior PG Total Percentage 112.

113.

Boys 450

Girls

600 60

400 40

Total

Percentage

100

10

The number of boys in senior and PG sections combined is 150. So the maximum number of girls in these two sections is 150. So the maximum number of students in these two sections = 150 + 150 = 300. Out of these, 100 students belong to the PG section. Hence the maximum number of students in Senior section is 200. Hence, [3].

scored 20. Hence,[4]. 118.

If ‘b’ number of boys passed the exam then ‘b – 60’ number of girls passed the exam.

x × 600 = 6x Hence, b = 100 & b – 60 =

Following is the possible number of units Joshi could have bought in the given period:

Jan Feb Mar Apr

x + 15 × 400 = 4x + 60 100

∴ 6x – 60 = 4x + 60 ⇒ x = 60 Now 60% of 600 = 360 ∴ Number of girls who passed = 360 – 60 = 300 ∴ In all 360 + 300 = 660 students passed. ⇒ 1000 – 660 = 340 students failed. Hence, [4].

Soap (Rs. 5) 2/1/2 2 1/0 2

Detergent (Rs. 14) 1/2/0 2 0/1 0

Joshi Shampoo Hair Oil (Rs. 7) (Rs. 9) 2/1/2 0/0/2 0 2 2/1 1/1 2 1

Total cost 42 52 30 35

Thus, in the month of April Joshi’s family definitely did not buy detergent. Hence, [4]. 119.

Following is the possible number of units Rao family could have bought in the given period:

Answers to questions 114 and 115: The difference between Sanjay and Hrithik’s ranks is 11. ∴ One of them will be ranked 1 and the other will be ranked 12. If Sanjay’s rank is 1, then Salman’s, Aamir’s Sunny’s, Shahrukh’s and Shahid’s rank will be 6, 9, 5, 3 & 8 respectively ... (I) If Sanjay’s rank is 12, Salman’s, Aamir’s, Sunny’s, Shahrukh’s and Shahid’s rank will be 7, 4, 8, 10 & 5 respectively ... (II) But Sunny’s rank is better than Shahid’s. ∴ Case (II) is not possible. ∴ The ranks are Salman – 6 Aamir – 9 Sunny – 5 Sharukh– 3 Shahid – 8 Sanjay – 1 Now, comparing Akshay, Saif, Bobby, Abhay & Kunal’s ranks with Hrithik’s rank, we get: Akshay– 2 Saif – 7 Bobby – 4 Abhay – 10 Hrithik – 12 Kunal – 11 114-[1] 115-[2]

Jan Feb Mar Apr

Soap (Rs. 5) 0/1/1 2/2/1 2 2/2/1

Shampoo (Rs. 9) 1/2/2 1/1/0 0 1/1/0

Rao Detergent (Rs. 14) 1/0/1 1/0/1 2 1/0/1

Rao Hair Oil (Rs. 7) 2/2/0 0/2/2 2 0/2/2

Total cost 37 33 52 33

If the Rao family buys detergent and hair oil every month, then they did not buy 2 units of Shampoo in the given period. Hence, [3]. 120.

148

The RPSF for Ground floor of building 'C' is 4700. Therefore, RPSF for sixth floor = 4700 + (50 × 6) = 5000 Therefore, the price of that flat = 5000 × 1400 = 70 lakhs. Also, the customer will pay 1% commission to the agent. ∴ Total amount = 70 lakhs + 1% of (70 lakhs) = 70.7 lakhs Hence, [3].

121.

122.

123.

The RPSF of flats on the sixth, seventh and the ninth floor of building 'B' are Rs. 4650, Rs. 4700 and Rs. 4800 respectively. To minimize the total cost, he will take the discount on the flat costing him the maximum i.e. on the flat on the ninth floor. Thus, RPSF of the flat on the ninth floor will be => 80% of 4800 ⇒ 3840 Total price of all the three flats = 1200 × (4650 + 4700 + 3840) = 158.28 lakhs Total commission = 1% of (158.28 lakhs) = 1.58 lakhs Hence, [4]. The price for a flat on the fourth floor of building 'A' is Rs. 67.2 lakhs. The price for a flat on the second floor of building 'C' is also Rs. 67.2 lakhs. Thus, the estate agent gets the same commission from both the flats. Hence, [5].

127.

Nandini got the highest points for the post of trainee, therefore she is fit for the same. Hence, [1].

128.

From the table shown the following candidates are selected for the given posts Trainee – Nandini Executive – Mohan Team Leader – Ojas (As Lakshmi is Project leader) Project Leader – Lakshmi ∴ Pawan won’t be selected for any post. Hence, [3].

Data Interpretation ªScore Maximiserª 129.

1-Netherlands v/s 2-Spain 1-Netherlands v/s 3-Australia 1-Netherlands v/s 12-China

Let the sales of company A in 2003 be 100 units Then in 2004 ⇒ 100 × 1.3 = 130 ⇒ ↑ by 30 Then in 2005 ⇒ 130 × 1.4 = 182 ⇒ ↑ by 52 Then in 2007 ⇒ 222.04 × 1.2 = 266.448 ⇒ ↑ by 44.40 ∴ In 2005 increase in sales is the highest. Hence, [3].

125.

v/s 3-Australia

2-Spain

v/s 12-China

3-Australia

v/s 12-China

∴ Winner of Group E: Spain Runner-up of Group E: Netherlands

Let sales of A in 2002 be 100 ∴ Sales of B = 200 Now in 2004 sales of A = 100 × 1.2 × 1.3 = 156 In 2004 sales of B = 200 × 1.05 × 1.08 = 227

Answers to question 130:

156 × 100 ≈ 69%. Hence, [5]. ∴ 227

Considering the efficiencies of all the machines we will have the following number of units of functional chips manufactured:

Hence, [2].

In 2007, the total sales of A and B will be 600 and 990 respectively. Let ‘x’ be the percentage increase in sales of both the companies in 2008.

Machine Chip 1 Chip 2 Chip 3 Chip 4 Chip 5 M1 108 225 M2 190 152 M3 90 207 M4 112 140 M5 240 136

 x   x  ∴ 600 +  100  600 + 990 +  100  990 = 1908     Solving for ‘x’, we get: x = 20. Hence, [3].

130.

Answers to questions 126 to 128: From the given information, the following table showing total points can be constructed: Post Trainee Executive Team Leader Project Leader

126.

2-Spain

Since each team was involved in one ‘upset’ game and one team lost all matches, the only possibility is Australia lost all matches, Netherlands beat China but lost to Spain, and Spain beat China.

Then in 2006 ⇒ 182 × 1.22 = 222.04 ⇒ ↑ by 40.04

124.

Group E games were:

Lakshmi 100 105 105 108

Mohan 105 108 88 89

Nandini 108 103 94 89

Ojas 94 89 105 105

Pawan 99 99 102 105

As Mohan got the highest points for the post of executive he will be fit for that post. Hence, [2].

149

To complete the order, the company has to produce 10000 units of Chip 1 and 10000 units of Chip 3. Chip 1 can be manufactured by M1, M3 and M4 while Chip 3 can be manufactured by M1 and M3. But a machine can produce only one type of chip a day. Thus to maximize the production and minimize the days we will have to operate M1 and M3 to produce 10000 units of Chip 3 first and then use these machines to produce Chip 1. M4 will simultaneously produce Chip 1. Now, in a day M1 and M3 produce 432 = (225 + 207) units of Chip 3, thus it will take

10000 = 23.148 days to 432

produce the required units of Chip 3. But as each machine produces only one type of chips in a day, we will operate M1 for 23 days and M3 for 24 days. Now, as M4 is simultaneously producing Chip 1, we have 112 × 24 + 108 (from M1) units of chips ready after 24 days i.e. 2796 units. Thus 7204 = (10000 – 2796) units of Chip 1 has to be manufactured and we have M1, M3 and M4 producing simultaneously. In a day three machines can produce 310 units of Chip1, thus

7204 = 23.238 days 310 i.e. 24 days. Thus to produce 10000 units of device A company will need a minimum of 48 days. Hence, [3]. 7204 units will be produced in

Team A B C D E F

Won

Loss

Draw

Draw 1 0 2 3/1 0 2

Points 9 6 6 5 2 2

Either 3 or 4. Hence, [5].

132.

In any of the above cases, the match between C and F has to be a draw. Hence, [4].

133.

The match between A and D ends as a draw. So we get the following table:

Points 9

A B C D E F

2 1 5 0

A X 0 0 1 0 0

B 2 X 2 0 0 0

C 2 0 X 1 0 1

D 1 2 1 X 0 1

E 2 2 2 2 X 0

F 2 2 1 1 2 X

C defeats B. Hence, [2]. 2

A scored 9 points. The only combination possible is 4 wins, 1 draw. B scored points in the range of 5 to 9. But it already had two losses, hence the possible combinations are 3 wins with 6 points or 2 wins, 1 draw and 2 losses with 5 points. However, if B scored 5 points, then C has to have scored 5 points. The only way C could have scored them is by 5 draws, which is impossible as E did not draw with anyone. Hence, B stood on second place with 6 points and three wins. Now, the only possible combination for C is 2 wins, 2 draws with 6 points.

Team A B C D E F

Loss 0 2 1 1/2 4 3

131.

Answer for questions 131 to 133: Team A B C D E F

Won 4 3 2 1/2 1 0

Won 4 3 2

Loss 0 2 1

Draw 1 0 2

134.

From the given condition we know that, if two or more students got equal marks then they did not attempt the same number of questions. Thus option [1] is eliminated as to get a score of 83 only one combination i.e. (21C,1W) is possible. If Ram attempts one more question he could have got it right or wrong, thus the condition will be satisfied if both the possibilities lead to a unique score. Now, for option [3], score is 68. One more attempt will give score of 72 (if correct) or 67 (if wrong). Both the scores are not unique; 72 = (18C, 0W) or (19C, 4W) and 67 = (17C, 1W) or (18C, 5W). Similarly options [4] and [5] cannot be the answer. For option [2] i.e. for score 70, Ram had attempted 18 correct and 2 wrong answers or 19 correct or 6 wrong but in second case he cannot attempt one more question. Thus if he had got one more question correct he would have scored 74 marks and if he had attempted it wrong he would have got 69. Both the scores are unique. Hence, [2].

135.

Following is the possible number of units sold in the month of March:

Points 9 6 6 5

0 2

In 15 matches, the total points scored will be 30. Hence, E scored 2 points. Only two combinations for 2 points are possible. 1 Win – 2 points 2 Draw – 2 points As E is placed ahead of F, in the league, E got 2 points by 1 win and F gained 2 points by two draws. D could score 5 points by 1 win, 1 loss and 3 draws or 2 wins, 2 losses and 1 draw.

150

Joshi Lele Deshmukh Rao

Soap (Rs. Detergent 5) (Rs. 14) 1/0 0/1 1/2 2/2 2/1/2 1/2/0 2 2

March Shampoo (Rs. 9) 2/1 0/1 2/1/2 0

Hair Oil (Rs. 7) 1/1 2/0 0/0/2 2

Total cost 30 47 42 52

We cannot determine which family did not buy Hair oil. Hence, [5].

136.

Following is the possible number of units sold in the month of April:

Joshi Lele Deshmukh Rao

Soap (Rs. 5) 2 0/1 1/2 2/2/1

Shampoo (Rs. 9) 2 1/2 0/1 1/1/0

ªScore Enhancerª

April Detergent (Rs. 14) 0 2/2 2/2 1/0/1

April Hair Oil (Rs. 7) 1 2/0 2/0 0/2/2

Data Interpretation 139.

bottles. Total cost 35 51 47 33

Percentage contribution of factory 1 =

140.

Let the total production of QRS Ltd. be x bottles. Let the percentage contribution of factory 6 be y From question 7, we know that the percentage contribution of factory 1 = 30% ∴ 5% of 30% of x = 30% of y% of x ∴ y = 5 ∴ Percentage contribution of factory 6 = 5% Hence, (2).

141.

Percentage contribution of factory 3 = 100 – (10 + 10 + 5 + 30 + 20) = 25% ∴ number of guava bottles produced by factory 3 = 30% of 25% of 4 lakh = 30,000. Hence, (2).

142.

Rachit and Nanu have a difference of atleast 3 marks in test 2. But Rachit’s total score is more. So his score must be atleast 4 more than Nanu’s in test 1. But Nanu stood 2nd in Test 1. So the only possibility is that Rachit scores 8 and the rest scored 4, 3, 2, 1. So, Astha’s score in Test 1 is 3. Hence, (1).

143.

Now the gap between Rachit and Nanu’s score in Test 2 must be exactly 3 as the gap can be at most 4 in test 1. So Nanu, Manav, Kartik and Rachit must have consecutive scores in test 2. So, difference in Kartik and Nanu’s scores in Test 2 is 2. Hence, (1).

144.

Considering Rachit's score in test 2 as 'x', we can write the final scores as follows:

ªScore Enhancerª From table-2, we know that 31 visitors read exactly two books. From table-1, total number of readings = 85 of which 31 visitors read two books. ∴ total number of readers = 85 – 31 = 54. Hence, [4].

Data Interpretation ªScore Maximiserª Let the total number of readers of each book be represented as B1, B2, B3, B4, & B5 and M & F represent Males & Females respectively. From the information given, B1 = 14 = 5M+9F B2 = 10 = 4M+6F B1 & B2 = 10 = 4M+6F B1 – (B1 & B2)

= 4

=

From the information given, B1 & B5 = 1 = B1 & B3 = 3 ∴ B1 & B3 = Hence, (1).

3

=

(6000× 100) 20000

= 30%. Hence, (2).

Data Interpretation

138.

100 = 6000 5

∴ Total production of factory 1 = 300 ×

From the table we can observe that every family could have bought Soap or Shampoo or Hair oil. Hence, [5].

137.

300 bottles of the grape drink produced by factory 1 accounts to 5% of its total production in that week.

1M + 3 F

Student

1M

Rachit Nanu Astha Manav Kartik

3 F

Rank Score Rank T1 T1 T2 1 2 3 4 5

8 4 3 2 1

4 1 5 2 3

Score T2

Total score

Final Rank

x x+3 2 x > 2 or x < –2. Thus, we cannot answer the question using statement I alone. From statement II: |x – 1| < 3 –3 < x – 1 < 3 –2 < x < 4. In this case, also, we cannot answer the question. Combining statements I and II: x > 2 and –2 < x < 4 i.e., 2 < x < 4 1 < x < 5. Thus we can answer the question using both the statements together. Hence, (3).

216.

Even with both statements together, there could be multiple cases; for example 1, 2, 3, 4, 5, 6, 7, 8, 9, 10, 11 or 0, 1, 2, 3, 4, 6, 8, 9, 10, 11, 12. Hence, (4).

Using statement I alone, In a 100 m race, Tina would take 10 sec to finish the race and Nita would take 10 + 20 = 30 seconds (Ratio = 3 :1) whereas in a 200 m race, Tina would take 20 seconds and Nita would take 40 second (Ratio = 2:1). So, statement I alone is not sufficient. Using statement II alone, For distance ‘x’ when Nitin goes x meters Tina goes x – 20 meters (as her speed is 10m/s). In the same

(x − 20) . time Nita goes x – 60 m. Ratio of speeds = (x − 60) This value depends upon the value of x. So, statement II alone is not sufficient.

Answers for questions 368 to 371: As per the data given, we get the following information:

Using statements I and II together, From the first statement about the race Nitin beats Tina by 20 m, that is, by 2 seconds (as Tina’s speed is 10 m/s) The second statement states that Nitin beats Tina by 2 sec. So both the statements are describing the same race.

Team 1 D F/G

Hence the ratio is 6 : 22. Hence, (3). 214.

Team 2 E Batsman + Wicketkeeper F/G Bowler

217.

From statement I we know that each team has exactly 2 batsmen, 2 bowler and 1 wicketkeeper. \one of A and J plays with team 1 while the other plays with team 2. Also, either B or I or both can play in team 2. Therefore, the data is insufficient to answer the question. From statement II, we know the both C and H plays in team 1. ∴ B also plays in team 1. A, J and I therefore play in team II. Thus, statement II alone is sufficient to answer the question alone. Hence, (1).

218.

From statement I, C plays either with E or H. If C plays in team 1, then H also plays in team 1. If C plays in team 2 i.e. with E, then H plays in team 1. In any case, H plays with team 1. Thus, the statement I alone is sufficient to answer the question.

From the 2 statements we conclude that when Nitin finishes the race Tina is 20 m behind and Nita is 60 m behind. Tina will further require 2 seconds to finish and Nita will finish 20 seconds after Tina. So Nita will take 20 + 2 = 22 seconds to finish the race in which she covers 60 m.  60  So Nita’s speed is   m/s.  22 

Batsman + Bowler Bowler

From the question, we get x + y + z = 12. From statement I: (x, y) = (1, 15) or (3, 5). But, since x + y + z = 12, (x, y) = (3, 5) and z = 4. Thus, we can find the value of xyz. From statement II: We know that (x + y + z) 2 = x 2 + y 2 + z 2 + 2(xy + yz + xz) 2 2 2 ∴ 144 = x + y + z + 2(47) 2 2 2 ∴ x + y + z = 50 2 2 2 ∴ x , y , z ∈ {1, 4, 9, 16, 25, 36, 49} The only possible combination is {9, 16, 25) …[since 9 + 16 + 25 = 50] ∴ x, y, z ∈ {3, 4, 5) Thus, using statement II alone, we can answer the question. Hence, (2).

Using statement II and the data, we have 2 cases:(a) If team 1 has 3 batsmen and team 2 has 3 bowlers, then: Team 1 → D, A, J, H, F/G Team 2 → E, I, B, C, G/F Thus, H plays in team 1. (b) If team 1 has 3 bowlers and team 2 has 3 batsmen, then: Team 1 → D, H, B, C, F/G Team 2 → E, A, J, I, G/F Thus, H plays in team 1 only. In both cases, H plays in team 1. Statement II is, therefore, sufficient to answer the question alone. Hence, (2).

160

219.

220.

Statement I tells us that Sanjay covers 14 km relative to the water i.e. the water must have covered the remaining 20 – 14 = 6 km, which would take 2 hours. Statement II is not sufficient, as we do not know whether Sanjay is moving in the same direction as the flow or in the opposite direction – in the first case the answer would be 2 hours, in the second it would be 5 hours. Hence, (1). From statement I, we cannot say anything as we don’t know if the cards are drawn with or without replacement. From statement II, we can calculate the required probability as

The 8 th student in the H group would have a cap from the same group as the 2nd student which is 'H'. The last (12 th) cap in the Slytherin group would now belong to the same group as the 3rd cap in the Slytherin group which is 'S'. Hence, 222-[2] 223-[4].

Logical Reasoning ªScore Maximiserª 224.

At the end of Round 23, the situation is A B C D 93 1 1 1 Now, when Round 24 starts, A will pass (4 × 23) + 1 = 93 coins and be left with 0. After that, B will pass 94 coins and be left with 0 and then C will pass 95 coins and be left with 0. Thus, at this point, A, B and C all have 0 coins and D has all 96. Thus, Deb wins the game. Hence, [4].

225.

We know that in spite of being an intelligent player, Raghav cannot identify the trench containing the treasure in 2 steps or less. Observe that if there are 3 trenches (say 1, 2, 3) then first tap on 1. If hot, the treasure is in 2. If warm, the treasure is in 3. So, if there are 3 trenches, then the treasure can definitely be identified in 2 steps. If there are 4 trenches (1 – 4), then: (i) If we start with 1 or 4: Say we start with 1. Then if it is warm, the treasure can be in 3 or 4 (2 more steps may be required). (ii) If we start with 2 or 3: Say we start with 2. Then if it is hot, the treasure can be in 1 or 3 (again 2 more steps may be required) So, we cannot be sure to get the treasure in 2 steps in case of 4 or more trenches. Since Raghav took 3 steps, there are at least 4 trenches. Hence, [2].

4 3 2 × × . Hence (1). 52 51 50

Logical Reasoning ªScore Enhancerª 221.

Let Abhishek, Bipin, Chetan and Deb be represented by A, B, C and D. A B C D Initially 24 24 24 24 1st Round –1 + 4 +1 – 2 +2 – 3 +3 – 4 Net Change 3 –1 –1 –1 1st Round –5 + 8 +5 – 6 +6 – 7 +7 – 8 Net Change 3 –1 –1 –1 We can see that total number of coins with ‘A’ increases by ‘3’ in each round and for all the others, it decreases by ‘1’. Number of coins with Bipin after 7 th round = 24 – 7 = 17 ( Q in each round number of coins decrease by 1) And number of coins that Abhishek will give him in 8 th round= 4 × 7 + 1 = 29 ∴ Total number of coins with Bipin = 17 + 29 = 46. Hence, [4].

Answers to questions 222 and 223:

Logical Reasoning

Note: (i) Each cap is now in a box with the same number, (ii) Caps 1, 4, 7, 10 of each group are now in the same new group as the same process repeats for each set of 3 caps. Similarly caps 2, 5, 8, 11 and caps 3, 6, 9, 12 are in the same groups. So it is enough to track the transfers of caps from boxes 1, 2, 3 of each group, as shown below:

ªScore Enhancerª

Originally G1 G2 G3 H1 H2 H3 R1 R2 R3 S1 S2 S3

1st transfer H1 R2 S3 G1

2nd transfer

Name Sam

4th transfer G1

H3 R1 S2 S3 G1

H2

Opposite to

Joe

G2

G2

G3

3rd transfer

Answers for questions 226 to 228: We first make an elementary table:

Joe

Cookies k k+3 2m m

Sachets x 2x y 2y

Milk No

H2 R3 S1 R2 S3 G1

G3 H1 S2 S3

Number of cookies with Sam must be m or 2m. But if it is 2m then we get 7m + 3 = 18 which is not possible. So, Sam has m cookies with m = 3 (5m + 3 = 18). Also, Raman adds milk so he cannot be opposite to Sam. And since Raman has more sugar than Arun, 2y > 2x. The only possibility then is

161

Now if Esha is next to Charan then we will have M A D B C E G (or the same in reverse) which is not possible as Bittu must be closer to Charan than to Divya. If Esha is next to Ganpat we have M A __ B __ E G which is not possible for the same reason. Esha can be next to Divya or Bittu - M A C B E D G or Ashay - M G D C B E A Hence, (2).

x = 1, 2x = y = 2, 2y = 4. We thus get the complete table:

Name Sam Arun Joe Raman

Opposite to Arun Sam Raman Joe

226-(3)

Cookies 3 6 6 3

Sachets 1 2 2 4

Milk No Yes

227-(2) 228-(3)

229.

Neither D nor F have prior experience, and neither is a doctor. So the only way to fulfill the given conditions is if A, who also has prior experience, goes as the doctor, while the 4th member also has prior experience. Thus the other doctors, C and G, must go on Trek I, and B must be on Trek II. Hence (3).

230.

If F and C go on Trek II, B must go on Trek I. Trek II requires the remaining 2 people to have prior experience and hence D and G also must go on Trek I. Now, any one of the three remaining people with prior experience (A, E and H) can be sent on Trek I and the other two on Trek II. Hence 3 ways. Hence (3).

231.

If A is the only doctor in his group, C and G must be in the other group. Thus B must be in the first group. Also, the other two with C and G must be having prior experience. Thus they must be E and H, while D and F must go with A and B. Hence (3).

232.

Since both A and C cannot go with B, they must be together. Hence the remaining doctor, G, must go with B. Also, each group has only one experienced person so far, and hence E and H must go to different groups. Similarly the fourth spot in each group must be taken by D and F, one to each group. Hence the options (1), (2) and (3) are all impossible, but option (4) could take place (for example Trek I - A, C, D, E and Trek II - B, F, G, H is a possible combination). Hence, (4).

233.

Since Mohit is farthest from Esha, he has to be at one of the corners and Esha cannot be at the centre. Also one of Ashay and Ganpat has to be at the other corner. So, the possible arrangements are: M A/G ___ ___ ___ ___ G/A OR A/G ___ ___ ___ ___ ___ G/A M Now (1) is possible by – M G B C D E A (2) and (3) are possible by – M G D C E B A But (4) is not possible because if Divya is at the centre, then Mohit will not be the only student sitting farthest from her. Hence, (4).

234.

The possible arrangements are: M _A_ ___ _B_ ___ ___ _G_ OR the same in reverse OR M _G_ ___ ___ _B_ ___ _A_ OR the same in reverse

235.

The pawn is initially at (1, 1). It can reach (4, 5) in a minimum of 5 moves with 3 tails and 2 heads. But it can also reach the same position in more moves by moving k more steps to the left and k more steps right. So total number of tails must be 3 + 2k. Similarly, number of heads must be 2 + 2m. So, the total number of moves must be odd and the difference between the number of heads and tails must be an odd number. Hence, (2).

236.

Since the pawn only jumps 2 steps up or down it can never reach the 4 th row. So (1) is not possible as the pawn cannot reach (5, 4) To reach (6, 5) Mini must get 5 + 2k tails and 2 + 2m heads. So, (2) is possible. To reach (6, 7) Mini must get 5 + 2k tails and 3 + 2m heads. So, the total number of moves must be an even number. So, (3) is not possible. To reach (4, 3) Mini must get 3 + 2k tails and 1 + 2m heads. So, the number of tails must be an odd number. So, (4) is not possible. Hence, (2).

Logical Reasoning ªScore Maximiserª Answers to questions 237 and 238: Using (ii), we can say that T can take four positions: 1, 2, 3,4. The possibilities are: 1

2

I

T

II

_

T

III

_

_

IV

R/S R/S

V

_

VI

_

3

R/S R/S

5

6

7

8

_

_

_

_

_

_

_

_

_

_

_

_

R/S R/S T T

R/S R/S _

4

_

R/S R/S _

_

_

_

_

T

_

_

_

_

_

_

T

R/S R/S

I is not possible as the fourth letter to the right of T will be H and, then, that will contradict (iii). Using II, the arrangment can be: _ T S R _ A _ H. Only P, E, M are left. M cannot be adjacent to A. But if this is followed, condition (ii) is violated.

162

Using III, the arrangement can be: _ _ T R S _ M H. P, A and E are left. A and E cannot be adjacent to M or condition (ii) is violated. Using IV, the arrangment can be: R ST _ H _M _ A and E cannot be adjacent to H and M and, thus, condition (ii) cannot be followed. Using V, the arrangement can be: _ S R T _ _ H E. M, P and A are left and can be adjusted under all conditions. Using VI, the arrangement can be: H __ T R S_ E M, P and A are left and can be adjusted under all conditions. Thus, we have the following four possibilities: 1

2

3

4

7

8

A

S

R

T P/M M/P H

E

H

P/M M/P T

5 R

6 S

A

X, Y and Z are the movies screened as that particular shows. X can neither be an action movie nor can it be a horror movie. ∴ it is a comedy movie – ‘Hangover’. Y cannot be an action movie as it is followed by an action movie. It cannot be a comedy movie because Hangover (comedy) is screened at the same time on screen 3. ∴ Y is a horror movie. Z cannot be a horror movie as it is followed by a horror movie. It cannot be a comedy movie because Hangover (comedy) is screened at the same time on screen 5. ∴ Z is an action movie. Hence, (1). 243.

If N is the 6th speaker, M must be the 5th or 7th speaker and hence in the afternoon session. P also must be in the afternoon session, along with one of O and Q. So R cannot be 5 th or 7 th. Also, R cannot be 1 st as R cannot be the first speaker in a session. Hence, (2).

244.

Option (1) is eliminated, as P cannot be in the morning session. Option (2) has both O and Q and is hence not possible. Option (4), too, can be eliminated, as it would mean that both O and Q are in the afternoon session. Hence, (3).

245.

If S and M are 1 st and 2 nd, the 3 rd must be N and the 4th must be one of O and Q. Hence options (1) and (4) can be eliminated. Also option (2) can be eliminated, as R cannot be the first speaker in a session. Hence, (3).

E

Now, we can answer all the questions. 237-(1) 238-(3)

Logical Reasoning ªScore Enhancerª 239.

Option (3) violates condition (d) and hence cannot be planted. Hence, (3).

240.

The situation is A C A _ _. Now clearly from condition (c) either the 4 th or 5 th row must contain Dahlias. Option (2) Gardenias is eliminated from condition (d); option (3) is eliminated from condition (g) and option (4) from condition (e). Option (1) does not violate any condition. Hence, (1).

241.

246. 0 1 2 3 4 5 6 7 8 9 10 11 12 13 14 1516 17 18 19 20 2122 23 24 25 26 2728

The situation is _ D _ D _. Since all the empty position are adjacent to Dahlias, Camellias can never be planted from condition (e), eliminating options (3) and (4). Option (1) violates condition (g). Hence, (2).

D(12) E(7)

Logical Reasoning

C(8)

G(2) F(5)

A(6)

As can be seen from the figure, the earliest completion is after 26 days, i.e. August 26 th. Hence, (3).

ªScore Maximiserª 242.

B(5)

247.

Refer to the table below:

0 1 2 3 4 5 6 7 8 9 10 11 12 13 14 1516 17 18 19 20 2122 23 24 25 26 2728 D(6)

Screen Screen Screen Screen Screen 4 1 2 3 5

E(7)

B(5) C(8)

G(2) F(5)

A(6)

Show 1 Show 2

Z

Show 3

Y

Transfo Show 4 rmers

Shutter

As can be seen from the figure, the reduction in D has no effect; the earliest completion is still after 26 days, i.e. August 26 th. Hence, (3).

Hangov er

X Die Hard

Show 5

163

252.

248. 0 1 2 3 4 5 6 7 8 9 10 11 12 13 14 1516 17 18 19 20 2122 23 24 25 26 2728 D(12) E(7)

B(5) C(8)

G(2) F(5)

A(3)

As can be seen from the figure, by reducing the time for A to half, we can complete the project in 23 days. As seen in the earlier question, a reduction in D to half the time would not change the time for the project. A reduction in C to half the time would yield a saving of only one day, as we could then finish in 25 days. Hence, (1). Answer for questions 249 to 251: The face values are 1, 2, 3, 4, 5, 6; the possible absolute differences are 1, 2, 3, 4, 5. In Round 2, only 1 team had an odd absolute difference (1, 3 or 5), therefore the other two differences are 4 and 2. The odd difference cannot be 1 since the team with the odd difference did not stand 3rd. A difference of 5 is possible if the face values are (1,6) but one of the teams has an absolute difference of 4, which is possible by (1,5) or (2,6) making an odd difference of 5 impossible. Thus, the absolute differences in round 2 are 4, 3 and 2 of Teams 1, 2 and 3 respectively (from the points table). It is also given that the absolute difference of only one team in Round 1 is the same as that of a team in Round 2. So, two teams have absolute differences of 1 and 5. An odd difference can occur only if the face values are odd and even. Both 1 and 5 are odd differences, which means two even and two odd face values. Since there are 3 even and 3 odd face values; the remaining face values are 1 odd and 1 even and thus the third absolute difference common to that of a team from Round 2 has to be odd and hence, 3. Thus, from the points table, the absolute differences of teams 1, 2 and 3 in Round 1 are 5, 1 and 3 respectively. Round 1 T1has an absolute difference of 5, this is possible only with face values (1,6). T3 has an absolute difference of 3, this is possible only with face values (2,5). Therefore T2 has face values of (3,4). Round 2 One of the members of T1 has scored a 5; this has to be in round 2.Since the absolute difference is 4 the face values have to be (1,5). T2 has an absolute difference of 3 this is only possible with face values (3,6). Therefore, T3 has face values (2,4). 249–1 250–3 251-3

164

Let DH, NN, LL, FF and RR represent the teams Devil Hunters, Nifty Ninjas, Laughing Lamas, Feral Falcons and Rocket Rangers respectively. From the number matches won, lost and drawn by each team and the fact that all teams won against RR and lost against NN, the following results can be deduced (the table is read vertically as NN won against DH). DH

DH NN LL FF RR

L L D W

NN W W W W

LL W L D W

FF D L D

RR L L L L

W

G1: NN - DH (1 – 0) It is given that NN played the first game and scored 1 goal. NN won all its matches and all other teams except DH scored 1 or more goals in their first matches. G2: FF - RR (2 - 1) Since no team plays consecutive matches, the next game has to be between two of LL, FF and RR and would be the first match of each of the teams. From the above table it can be seen that LL drew its game with FF and won its game against RR. But since, in its first match, it scored less than FF and as many as RR did in their first matches, the second game has to be between FF and RR. G3: NN – LL (2-1) Since no team plays consecutive matches, and DH and NN have already played, the next game has to be between LL & DH (1 – 2) or LL & NN (1 -2) and the game is LL’s first match and DH or NN’s second match . From the above table it can be seen that LL won against DH. So this game has to be between LL and NN. G4: DH - RR (2 - 0) The next game has to be between DH, RR, and FF and is each team’s second match. Since the next game is played by FF this game has to be between DH and RR. G5: FF - LL (1 - 1) FF’s opponent has to be NN or LL. From the table above, LL has won its match with DH and DH has already played its first two matches. The only match in which LL could have beaten DH is if both of them play their fourth matches against each other. Hence, DH has to play its third match with FF, which is also a draw and it has to be the third match of both teams. In this game FF’s opponent has to be either NN or LL; it has to be only LL since from the table the match is a draw, and its possible only if it is the second game of both teams.

Based on the games so far, and from the table above it can be deduced that G6: NN – RR (3 - 1) G7: FF – DH (2 - 2) G8: LL – RR (2 - 1) G9: NN – FF (2 – 0) G10: LL – DH (1 – 0) Also, NN, LL, DH, FF and RR have a GD of 6, 1, 0, – 1, –6 respectively. Hence, (2). 253.

It is given that the speeds of the train starting at 8A.M. and 8P.M. are the maximum. A goes to Park Street and C returns to Lincoln Street respectively at these timings. Hence, the time taken for these journeys is minimum. However, as the total time taken by all passengers A.B, C, D for the to and fro journey is the same. Therefore, the time taken by A on journey from Park street to Lincoln Street and the time taken by C on the journey from Lincoln street to Park Street should be maximum (i.e. for the trains at timings 7 P.M. and 9 A.M. respectively). Hence, (4).

254.

t1 and t3 are minimum and maximum respectively among t1, t2, t3, t4. t7 and t5 are minimum and maximum respectively among t5, t6, t7, t8. Therefore, t1 < t3. Also, value of t2 will fall between value of t1 and t3. Therefore, t1 < t2 < t3 (thus, option (3) is true) t1 = t7 and t3 = t5 (Since these trains have same speeds – as discussed in the earlier question and given data that trains taken by A and C and 8A.M. and 8 P.M. have maximum speeds) ∴ t1 × t5 = t7 t3 (thus option (4) is correct) If t4 = t6, then t2 = t8. Therefore, t2 = t8 is possible, since there is no constraint that will disallow t4 = t6 (thus option (1)is true). Also, t7 is minimum among t5, t6, t7 and t8. \t7 < t5 (thus option (2) is false). Hence, (2).

255.

t1 and t3 are minimum and maximum respectively among t1, t2, t3 and t4. t7 and t5 are minimum and maximum respectively among t5, t6, t7 and t8. And from the question we have: t2>t4 and therefore t6
View more...

Comments

Copyright ©2017 KUPDF Inc.
SUPPORT KUPDF